You are on page 1of 136

The IBPS RRB PO & Clerk 2021 | SUCCESS GUIDE

1 www.bankersadda.com | www.sscadda.com | www.careerpower.in | Adda247 App


The IBPS RRB PO & Clerk 2021 | SUCCESS GUIDE

Contents of IBPS RRB PO & Clerk 2021 SUCCESS GUIDE


IBPS RRB PO/CLERK Prelims Study Plan | Attempt Daily Quiz on Bankersadda.com & Adda247 App .... 3

IBPS RRB PO/CLERK 2021: Syllabus & Exam Pattern .........................................................................4

IBPS RRB PO/CLERK Syllabus 2021 .................................................................................................. 5

RRB PO PRELIMS | Trend in Exam Analysis | Last 3 Years ................................................................... 7

RRB PO CLERK PRELIMS | Trend in Exam Analysis | Last 3 Years ........................................................9

RRB PO PRELIMS Cut Off Trend: Last 3 Years (2018-2020) ............................................................... 11

RRB CLERK PRELIMS Cut Off Trend: Last 3 Years (2018-2020).......................................................... 13

IBPS RRB PO PRELIMS MEMORY BASED PAPER 2020 .................................................................... 15

IBPS RRB PO PRELIMS MEMORY BASED PAPER 2019 .................................................................... 25

IBPS RRB PO PRELIMS MEMORY BASED PAPER 2018 .................................................................... 35

IBPS RRB PO PRELIMS MEMORY BASED PAPER 2017 .................................................................... 46

IBPS RRB CLERK PRELIMS MEMORY BASED PAPER 2020 ............................................................... 55

IBPS RRB CLERK PRELIMS MEMORY BASED PAPER 2019 ............................................................... 63

IBPS RRB CLERK PRELIMS MEMORY BASED PAPER 2018 ............................................................... 71

IBPS RRB CLERK PRELIMS MEMORY BASED PAPER 2017 ............................................................... 79

Most Important Questions | Quantitative Aptitude | IBPS RRB PO & CLERK PRELIMS ....................... 87

Most Important Questions | Reasoning Ability | IBPS RRB PO & CLERK PRELIMS ............................ 115

2 www.bankersadda.com | www.sscadda.com | www.careerpower.in | Adda247 App


The IBPS RRB PO & Clerk 2021 | SUCCESS GUIDE
IBPS RRB PO/CLERK Prelims Study Plan | Attempt Daily Quiz
on Bankersadda.com & Adda247 App
 IBPS has finally released the official notification for the recruitment of Officer Scale-I and Office Assistant.
Now, the most important thing for all the candidates to follow a great strategy and proper study plan.
 IBPS RRB PO and Clerk Prelims will be held in the month of August 2021. Hence, the Bankersadda team
has launched a proper study plan on the daily basis for all the candidates that will help the students to
improve their accuracy and work on their speed.
 First of all, we will help you to clear your prelims exam so we are providing you the daily topic-wise study
plan of Reasoning Ability and Quantitative Aptitude. The questions will be exactly on the basis of the exam
level.

Mission IBPS RRB PO/Clerk 2021: Study Plan


Dates Reasoning Ability Quantitative Aptitude
9-June Series Simplification
10-June Inequality Missing Series
11-June Coding-Decoding Quadratic Equation
12-June Revision Test Revision Test
13-June Revision Test Revision Test
14-June Syllogism Approximation
15-June Puzzle & Seating Arrangement Wrong Series
16-June Direction Sense Table DI and Bar Graph DI
17-June Blood Relation Line Graph DI and Arithmetic DI
18-June Order & Ranking and Short Puzzles Pie Chart DI and Misc DI
19-June Revision Test Revision Test
20-June Revision Test Revision Test
21-June Coding-Decoding Caselet DI
Number System, Percentage, Ratio &
22-June Inequality
Proportion, Average
23-June Series Partnership, Mixture & Alligation, Ages
24-June Puzzle & Seating Arrangement SI & CI, Time & Work, Pipe & Cistern
Speed, Time & Distance, Boat & Stream, T
25-June Syllogism
rains, Profit & Loss
26-June Revision Test Revision Test
27-June Revision Test Revision Test
Mensuration, Probability, Permutation
28-June Direction Sense
& Combination

3 www.bankersadda.com | www.sscadda.com | www.careerpower.in | Adda247 App


The IBPS RRB PO & Clerk 2021 | SUCCESS GUIDE
IBPS RRB PO/CLERK 2021: Syllabus & Exam Pattern
Read the article to get the detailed information about the IBPS RRB Syllabus 2021 and exam pattern for IBPS
PO, Clerk and other posts.

IBPS RRB PO Exam Pattern 2021


 For IBPS RRB PO, the recruitment will be done based on the prelims, mains, and interview round. All those
students who will qualify for the prelims examination will be called for the mains examination, and the
students who will qualify for the mains examination will be called for the interview round. The detailed
exam pattern is given below:

IBPS RRB PO Prelims Exam Pattern 2021:


Subject No. of Questions Maximum Marks Duration
Reasoning Ability 40 40
45 minutes
Quantitative Aptitude 40 40
Total 80 80

IBPS RRB PO Mains Exam Pattern 2021:


Subjects No. of Questions Maximum Marks Duration
Reasoning 40 50
Quantitative Aptitude 40 50
General Awareness 40 40 2 hours
English/Hindi Language* 40 40
Computer Knowledge 40 20
Total 200 200

IBPS RRB Clerk Exam Pattern 2021


 The IBPS RRB Clerk recruitment will be done based on the prelims and mains examination. All those
students who will qualify for the prelims examination will be called for the mains examination.

IBPS RRB Clerk Prelims Exam Pattern 2021:


Subjects No. of Questions Maximum Marks Duration
Reasoning Ability 40 40
45 minutes
Numerical Ability 40 40
Total 80 80

IBPS RRB Mains Prelims Exam Pattern 2021:


Subjects No. of Questions Maximum Marks Duration
Reasoning Ability 40 50
Numerical Ability 40 50
General Awareness 40 40 2 hours
English/Hindi Language* 40 40
Computer Knowledge 40 20
Total 200 200

4 www.bankersadda.com | www.sscadda.com | www.careerpower.in | Adda247 App


The IBPS RRB PO & Clerk 2021 | SUCCESS GUIDE
IBPS RRB PO/CLERK Syllabus 2021
The sections that will be covered in the IBPS RRB Exam are as follows:
• Quantitative Aptitude
• Logical Reasoning
• English / Hindi Language
• Financial Knowledge
• General Awareness
• Computer Knowledge

IBPS RRB Syllabus: Quantitative Aptitude


 The syllabus for Quantitative Aptitude IBPS RRB 2021 is similar for both the prelims and mains examination
of IBPS RRB PO and Clerk. The Quantitative Aptitude is said to be quite tough and lengthy amongst all, but
if you have prepared well and practiced enough for this section, then you can easily do the same. Check the
detailed syllabus Quantitative aptitude, this will help the students in their preparation.
Number System HCF and LCM Profit and Loss
Decimal Fractions Simple Interest Compound Interest
Time and Work Time and Distance Average
Age Problems Simplification Partnership
Percentage Ratio and Proportion Data Interpretation
Permutation and Combination Probability Quadratic Equations

IBPS RRB Syllabus: Reasoning Ability


 This section covers logical and verbal reasoning. Students must prepare well for this section as well. The
detailed syllabus of IBPS RRB PO and Clerk 2021 Reasoning Syllabus is mentioned below.
Odd man out Analogy Syllogism
Coding-Decoding Blood Relation Alphabet Test
Blood Relation Series Test Ranking and Time
Causes and Effects Direction Test Sitting Arrangements
Decision Making Statement and Assumption Figure Series
Assertion and Reason Statement and Conclusion Word Formation
Statements and Action Courses Inequalities Puzzles

IBPS RRB Syllabus: English Language


 The English Language section is said to be quite scoring if students have prepared well for the same.
 This section comprises topics related to vocabulary grammar etc. Check the detailed syllabus for IBPS RRB
PO and Clerk for the English Language.
Reading Comprehension Cloze Test Fill in the blanks
Rearrangement of Sentences Jumbled Words Error Detection
Phrase Substitution One word substitution Idioms
Antonyms Synonyms
5 www.bankersadda.com | www.sscadda.com | www.careerpower.in | Adda247 App
The IBPS RRB PO & Clerk 2021 | SUCCESS GUIDE
IBPS RRB Syllabus: Computer
 The computer Language section is asked in the mains examination, the detailed syllabus for the Computer
is mentioned in the table below:
Computer Fundamentals Number System and Conversions
History of Computer Shortcut Keys
Software and Hardware Fundamentals Basic Knowledge of the Internet
MS Office Database
Networking Security Tools
Computer Abbreviations Computer Languages
Internet Input and Output Devices

IBPS RRB Syllabus: General Awareness/Financial Awareness.


 The detailed syllabus of general awareness and financial awareness is mentioned in the tables below.
Students must prepare well for this section to boost up their scores.
India and International Current Affairs Banking Awareness
Countries and Currencies National Parks and Wildlife Sanctuaries
Banking Terms and Abbreviations Banking History
RBI Sports
Finance Sports
Books and Authors Agriculture
Fiscal Policies Budget
Government schemes Government policies
Budget and Economic Survey Current Affairs | Past 4 Months

6 www.bankersadda.com | www.sscadda.com | www.careerpower.in | Adda247 App


The IBPS RRB PO & Clerk 2021 | SUCCESS GUIDE
RRB PO PRELIMS | Trend in Exam Analysis | Last 3 Years
 In this article, we will analyze both sections Reasoning and Quantitative Aptitude. We have prepared a chart
and table representation for the last three years 2018, 2019, and 2020 which will help you understand the
exam analysis trend.
 This article shows the topics which were dominating in the particular section and the changing pattern on
types of questions.

IBPS RRB PO Prelims Exam Analysis Trend: Reasoning Section


 The reasoning section contains 40 questions that cover almost all the topics of the Reasoning subject in the
IBPS RRB PO Prelims exam. The most dominating topic of Reasoning is the Puzzle and seating
arrangement section.
 More than 20 questions are asked every year from the puzzles and seating arrangement. Direction sense
was not asked in 2019 but 2018 and 2020. If we discuss the topic’s consistency, then the Alphabetical series
is the only topic that was continuously asked in the exam in 2018, 2019, and 2020.
 The table given below shows the number of questions asked from each topic for 2018, 2019, and 2020.

IBPS RRB PO Prelims Exam Analysis Trend: Reasoning Section


Topics 2020 2019 2018
Puzzle and Seating Arrangement 25 23 27
Direction Sense 3 0 3
Number Based Series 1 0 1
Alphabet Based Series 2 2 1
Inequality 3 5 0
Coding-Decoding 5 5 0
Day based miscellaneous question 1 0 0
Blood Relation 0 5 3
Syllogism 0 0 5
Total 40 40 40

7 www.bankersadda.com | www.sscadda.com | www.careerpower.in | Adda247 App


The IBPS RRB PO & Clerk 2021 | SUCCESS GUIDE
IBPS RRB PO Prelims Exam Analysis Trend: Quantitative Aptitude Section
 The quantitative aptitude section contains 40 questions for the IBPS RRB PO Prelims exam. Half of the
section consists of Data Interpretation and Arithmetic questions.
 Data interpretation weighs approx 10-15 questions every year and arithmetic weighs approx 11-15
questions.

IBPS RRB PO Prelims Exam Analysis Trend: Quantitative Aptitude Section


Topics 2020 2019 2018
Data Interpretation 10 12 15
Quadratic Equation 5 6 5
Missing/Wrong Number Series 5 6 5
Quantity 1 and Quantity 2 based on inequalit
5 0 0
y
Arithmetic Word Problems 15 11 15
Data Sufficiency 0 5 0
Total 40 40 40

8 www.bankersadda.com | www.sscadda.com | www.careerpower.in | Adda247 App


The IBPS RRB PO & Clerk 2021 | SUCCESS GUIDE
RRB PO CLERK PRELIMS | Trend in Exam Analysis | Last 3 Years
This article will give you the analysis comparison for the last three years i.e. 2018, 2019, and 2020. IBPS RRB
Clerk Prelims Exam Analysis Trend will help you to understand the exam structure, types of new questions,
weightage of each topic in each section.

IBPS RRB Clerk Prelims Exam Analysis Trend: Reasoning Section


 40 questions are asked in the Reasoning Ability section in the IBPS RRB Clerk Prelims exam. Half of the
section consists of the puzzles and seating arrangement.
 And remaining section consists of other topics like direction sense, inequality, syllogism, etc.
 The puzzles and seating arrangement weighs approx 20 questions and there is a consistency of
alphanumeric series in all three years. Syllogism questions were also asked in 2019 and 2020.

IBPS RRB Clerk Prelims Exam Analysis Trend: Reasoning Section


Topics 2020 2019 2018
Puzzle and Seating Arrangement 20 20 16
Direction Sense 2 0 5
Alphabet Based Series 5 5 0
Inequality 0 5 5
Coding-Decoding 0 0 5
Miscellaneous question 3 0 4
Blood Relation 0 3 0
Syllogism 4 5 0
Alphanumeric Series 4 2 5
Order & Ranking 2 0 0
Total 40 40 40

9 www.bankersadda.com | www.sscadda.com | www.careerpower.in | Adda247 App


The IBPS RRB PO & Clerk 2021 | SUCCESS GUIDE
IBPS RRB Clerk Prelims Exam Analysis Trend: Quantitative Aptitude Section
 The quantitative Aptitude section consists of 40 questions in the IBPS RRB Clerk Prelims exam. To score
maximum marks in this section, you should focus on Data Interpretation, simplification, and series
questions.
 Minimum 5 questions are always asked from Data interpretation, 5 questions from series, and 10 questions
from simplification. Apart from this, 15 questions were asked from the arithmetic questions in 2020.

IBPS RRB Clerk Prelims Exam Analysis Trend: Quantitative Aptitude Section
Topics 2020 2019 2018
Data Interpretation 10 13 10
Quadratic Equation 0 5 5
Missing/Wrong Number Series 5 5 5
Simplification 10 10 15
Arithmetic Word Problems 15 7 5
Total 40 40 40

10 www.bankersadda.com | www.sscadda.com | www.careerpower.in | Adda247 App


The IBPS RRB PO & Clerk 2021 | SUCCESS GUIDE

RRB PO PRELIMS Cut Off Trend: Last 3 Years (2018-2020)


 In this article we are going to discuss the 1st Stage of IBPS RRB PO 2021 i.e. the Prelims Exam which is
having 2 sections, Maths & Reasoning of a total of 80 marks having a time of 45 minutes.
 To crack the Prelims exam, candidates need to score more than the cut off marks. In this, we are going to
discuss the trend in the IBPS RRB PO Prelims Cut Off for the past 3 years, i.e. 2018-2020.
 This will give all the aspirants an idea about the level of the examination & at least how many questions
they have to attempt in Prelims exam.

IBPS RRB PO Cut Off: Factors that decide the cut-off


The cut off is set up based on the following factors:
1. Number of Vacancies: This factor influences the total number of vacancies released by the IBPS for the
IBPS RRB PO recruitment. An increase or decrease in the vacancies can fluctuate and influence the cut-off.
2. Level Of Difficulty: The second factor is the difficulty level of the examinations. When the exam seems to
be easy, the cut-off increases, and simultaneously the more exam will be tough, the less cut-off will be
released by the recruitment board.
3. Number of Candidates appearing for the exam.

IBPS RRB Cut Off Trend: 2018, 2019, 2020


Let’s discuss the cut-off trends of the past 3 years, which will help you in identifying the level of the
examinations and will understand the level of the competition.

11 www.bankersadda.com | www.sscadda.com | www.careerpower.in | Adda247 App


The IBPS RRB PO & Clerk 2021 | SUCCESS GUIDE
 The above chart highlights the IBPS RRB PO prelims state-wise cut-off trends of the past years.
 As per the trend, the cut-off for the year 2020 was lower than the year 2019 for most of the states. However,
for the year 2018, the overall cut off was on the lower side. It could because of the fact that the paper of
IBPS RRB 2018 was a bit difficult. Even the good attempts that we stated was low in our analysis.
 Similarly, the paper of 2020 was also of moderate level, resulting in a lower cut off in most of the states.

IBPS RRB PO Prelims: Cut Off 2018-2020


 The table below mentioned highlights the state-wise cut-off of IBPS RRB PO prelims for the year 2018 to
2020. In the year 2020, there was an increase in the cut off of the three states which are Gujrat, Odisha and
Rajasthan. Study the table below carefully:

General Cut Off Trend


Name of the States 2018 2019 2020
Andhra Pradesh 52.5 58.5 52.75
Assam – 41.5 41
Bihar 45 58 48
Chhattisgarh 53.5 55.5 43.25
Gujarat 48.25 43.5 59.75
Haryana 57 64.5 60.5
Himachal Pradesh 59.75 59.75 56.5
Jammu & Kashmir 47.25 55.25 52
Jharkhand – 59.5 54.25
Karnataka 44.25 46.25 –
Kerala – 61 –
Maharashtra 47.5 54.7 47.25
Manipur – 56 –
MP - 54.7 44.25
Odisha 50.5 55.75 62.75
Punjab 54.75 63.5 59
Rajasthan 50.5 58.5 66
Tamil Nadu 43.25 55.25 54
Telangana 45.25 54 48.25
Tripura 29.25 37.5 –
Uttar Pradesh 50 58.75 47
Uttarakhand 54 65 61
West Bengal 48.5 55.25 52

12 www.bankersadda.com | www.sscadda.com | www.careerpower.in | Adda247 App


The IBPS RRB PO & Clerk 2021 | SUCCESS GUIDE

RRB CLERK PRELIMS Cut Off Trend: Last 3 Years (2018-2020)


 In this article we are going to provide you the cut-off trends for the IBPS RRB Clerk prelims examination of
the past three years, which will be helpful for all the candidates in identifying the difficulty level, and the
competition of the examination.
 The IBPS RRB Clerk prelims exam will consist of the two sections, which will be of 80 marks containing the
reasoning and quantitative aptitude subjects. Candidates have to qualify for more than the cut-off marks
in order to get the selection. Read the below article, to get detailed information about the IBPS RRB Clerk’s
previous year’s cut-off trends.

IBPS RRB Clerk Cut Off: Factors that decide the Cutoff
As mentioned above, are providing you the cut-off trends of the past three years, which will help the candidates
in identifying the difficulty level and the number of questions they are required to attempt to qualify for the
examination and to score more than the cut-off set up by the board. The cut off trend is based on the two
following factors:
1. Number of Vacancies: This factor influences the total number of vacancies released by the IBPS for the
IBPS RRB Clerk recruitment. An increase or decrease in the vacancies can fluctuate and influence the cut-
off.
2. Level Of Difficulty: The second factor is the difficulty level of the examinations. When the exam seems to
be easy, the cut-off increases, and simultaneously the more exam will be tough, the less cut-off will be
released by the recruitment board.
3. Number of candidates appearing in the examination.

13 www.bankersadda.com | www.sscadda.com | www.careerpower.in | Adda247 App


The IBPS RRB PO & Clerk 2021 | SUCCESS GUIDE
 The above-mentioned chart showcase the cut off trends of IBPS RRB Clerk prelims examination of the year
2020, 2019 and 2018.
 As per the trend, the cut-off for the year 2020 was more than 2019, which implies that the difficulty level of
the exam was less than that of 2019. Similarly, the cut-off for the year 2019 was more than 2018 except for
Himachal Pradesh.

IBPS RRB Clerk Prelims: Cut Off 2018-2020


The table below mentioned the detailed information of the IBPS Clerk Prelims cut-offs for the year 2018-2020
which will give you a clear image of the level of the examinations.

General Cut Off Trend


Name of the States 2018 2019 2020
Andhra Pradesh 72.5 71.5 76.25
Assam – 64.75 69
Bihar 70.25 74.25 75.5
Chhattisgarh 67.75 75.5 70.5
Gujarat 69.75 63.25 78.25
Haryana 76.25 76
Himachal Pradesh 77.5 71 71.25
Jammu & Kashmir 70 – 73.5
Jharkhand 69.75 58.5 –
Karnataka 66.25 65.25 NA
Kerala 73.5 75 NA
Maharashtra 69.75 69.25 67
MP 70.5 68.25 66.75
Odisha 71.25 73.75 79.75
Punjab 74.75 77.5 78.5
Rajasthan 73 75.25 78.75
Tamil Nadu 61.75 68 71
Telangana 67.75 68.5 71.25
Tripura 48.75 71.25 NA
Uttar Pradesh 70.75 74 73
Uttarakhand – 76.75 NA
West Bengal 75.25 74.75 77.75

14 www.bankersadda.com | www.sscadda.com | www.careerpower.in | Adda247 App


The IBPS RRB PO & Clerk 2021 | SUCCESS GUIDE

IBPS RRB PO PRELIMS MEMORY BASED PAPER 2020


REASONING ABILITY

1. If 2 is subtracted from each even digit and 1 is added to 7. What is the code of ‘make’ as per the given code
each odd digit in the given number ‘2145673’, then language?
how many digits will appear more than one in the new (a) zm (b) mn (c) rk
number thus formed? (d) pc (e) None of these
(a) None (b) One (c) Two
8. What is the code of ‘exam’ as per the given code
(d) Three (e) None of these
language?
Directions (2-6): Study the following information (a) rk (b) pc (c) kr
carefully and answer the question given below- (d) oj (e) None of these
Eight persons live in a building of four floors such that
ground floor is numbered 1 and floor above it is 2 and so 9. What is the code of ‘go’ as per the given code language?
on up to 4th floor. Each of the floor consist of 2 flats as flat- (a) pc (b) si (c) kr
P, which is in west of flat Q. Flat-P of floor-2 is immediately (d) either ‘pc’ or ‘kr’ (e) None of these
above flat-P of floor-1 and immediately below flat-P of 10. What is the code of ‘exam today’ as per the given code
floor-3 and in the same way flat-Q of each floor follow same language?
pattern. (a) si oj (b) mn kr (c) lv si
A lives on an even numbered floor. A lives just above the (d) zm oj (e) None of these
flat of E. B lives to the west of E. One floor gap between D
and C. H lives in the east of D. G lives on the 3rd floor. Both 11. If ‘easy to plan’ is coded as ‘mn ly pc’ then what is the
F and C live in the different flats. code of ‘go’ as per the given code language?
(a) pc (b) zm (c) kr
2. Who among the following lives just below the flat in (d) mn (e) None of these
which G lives?
(a) B (b) C (c) A 12. How many pairs of letters are there in the word
(d) Both (b) and (c) (e) None of these “GRANDUAL” each of which have as many letters
between them in the word as they have between them
3. How many floors gap between B and H? in the English alphabetical series??
(a) None (b) Two (c) One (a) Three (b) Four (c) Two
(d) Either (a) or (c) (e) Either (b) or (c) (d) One (e) Five
4. What is the direction of G with respect to E? Directions (13-15): Study the information carefully and
(a) South (b) North-east (c) North answer the questions given below.
(d) East (e) North-west Point P is 26m west of point S. Point G is 52m north of point
5. Which of the following floor does C lives? P. Point M is 39m east of point G and point K is 13m south
(a) Floor-1 (b) Floor-4 (c) Floor-3 of point G. Point H is 39m north of point S.
(d) Floor-2 (e) None of these
6. Which of the following is true regarding H?
(a) Floor 4 – Flat P
(b) Floor3- Flat Q
(c) Floor 2- Flat P
(d) Floor4- Flat Q
(e) Floor1- Flat Q
Direction (7-11): Study the following information
carefully and answer the questions given below:
In a certain code language
‘plan to go exam’ is coded as ‘oj kr mn pc’
‘exam today easy’ is coded as ‘si oj ly’
‘plan your exam today’ is coded as ‘zm oj si mn ’
‘make your plan today’ is coded as ‘zm si mn rk’
15 www.bankersadda.com | www.sscadda.com | www.careerpower.in | Adda247 App
The IBPS RRB PO & Clerk 2021 | SUCCESS GUIDE
13. In which direction point P with respect to point M? born on Friday. Two persons were born between U and
(a) South (b) South-east (c) North-east P. One person was born between R and S. If T was born
(d) East (e) None of these immediate before S, then who among the following
person was born on Wednesday?
14. What is the shortest distance between point K and
(a) U (b) R (c) S
point H?
(d) T (e) None of these
(a) 13m (b) 26m (c) 39m
(d) 25m (e) None of these Directions (22-26): Study the following information
carefully and answer the question given below-
15. If Point Z is 13m north of point H, then what is the
distance between point M and point Z? Ten boxes are placed one above the other. Four boxes are
(a) 13m (b) 26m (c) 39m placed between J and M. Two boxes are placed between J
(d) 25m (e) None of these and k, which placed above of the J. L is placed just below K.
Directions (16-20): Study the following information The number of boxes between L and M is same the number
carefully and answer the question given below- of boxes between M and Q. T is placed just above Q. Y is
placed just above O. X is adjacent to M. P is placed below X.
Seven persons D, G, P, L, J, U and Q are sitting in a row facing
to the north. They all have of different ages. D sits 3rd from 22. How many boxes are placed between O and L?
one of the extreme ends of the row. Q sits 2nd to the right of (a) One (b) More than Five (c) Four
D. The number of persons sit to the left of Q is same as the (d) Three (e) Two
number of persons sit to the right of G, who is 20 years old. 23. Four of the following five are alike in a certain way and
P sits 4th to the left of the one who is 35 years old. Q is not so form a group. Find the one who does not belong to
35 years old. Total age of immediate neighbours of D is 75 that group?
years. J is 30 years old. P is 20 year older than one of his (a) T-Y (b) X-L (c) K-P
immediate neighbours. U sits to the right of L, who sits (d) Q-O (e) M-Q
immediate to the left of the one who is 25 years old. Q is 5
year younger than P. 24. Which of the following statement is true?
(a) L is 3rd from the topmost position
16. The number of persons sit between L and Q is same as (b) Two boxes placed between K and M
the number of persons sit between P and ____? (c) Q is above P
(a) D (b) G (c) U (d) T is placed at bottommost position
(d) Q (e) None of these (e) Three boxes placed between Y and M
17. What is the position of J with respect to Q? 25. What is the position of Y from the bottommost?
(a) 2nd to the left (a) Seven (b) Eight (c) Six
(b) Immediate left (d) Five (e) Three
(c) Immediate right
(d) 4th to the left 26. If T and O interchange their positions then which
(e) 3rd to the right among the following box is placed just below O?
(a) Y (b) X (c) K
18. Four of the following five are alike in a certain way and (d) Q (e) None of these
so form a group. Find the one who does not belong to
that group? Directions (27-31): Study the following information
(a) P (b) G (c) J carefully and answer the question given below-
(d) L (e) U A certain number of persons sit in a row facing to the north
19. Who among the following is 40 years old? direction. L sits 3rd to the left of M. Five persons sit between
(a) L (b) P (c) D M and N. J sits immediate to the right of M. Three persons
(d) U (e) None of these sit between Q and J. Q does not sit next to N. N is 7th from
one of the ends. The number of persons sit to the right of Q
20. Which of the following statement is true? is four more than the persons who sit to the left of N. K sits
(a) J sits to the right of L 2nd from one of the ends and sit to the right of M.
(b) D sits 3 to the right of G
rd

(c) P sits at one of the extreme ends 27. How many persons sit in the above arrangement?
(a) 25 (b) 26 (c) 28
(d) None is true
(d) 24 (e) None of these
(e) Q sits immediate right of the one who is 35 years
old 28. If two persons sit between X and N, then what is the
position of X with respect to L?
21. Study the following information carefully and answer
(a) 4th to the left
the given questions.
(b) 6th to the left
Six persons i.e. P, Q, R, S, T and U was born on different
(c) 5th to the right
days of the same week starting from Monday to
(d) 3rd to the left
Saturday, but not necessarily in the same order. P was
(e) 7th to the left
16 www.bankersadda.com | www.sscadda.com | www.careerpower.in | Adda247 App
The IBPS RRB PO & Clerk 2021 | SUCCESS GUIDE
29. How many persons sit between L and J? Directions (36-40): Study the following information
(a) Five (b) None of these (c) Seven carefully and answer the question given below-
(d) Four (e) Three Nine persons sit around a circular table. Some of them are
facing to the centre while some are facing outside the
30. What is the position of L with respect to Q? centre. C sits 2nd to the right of A, who faces inside. Two
(a) 8th to the right persons sit between C and G. J sits 3rd to the left of G. L sits
(b) 8th to the left 2nd to the left of J, who does not sit next to C. B sits 3rd to the
(c) 6th to the right right of L and is an immediate neighbour of P. K sits 4th to
(d) 5th to the left the right of H, who does not sit near J. Both B and P face
(e) None of these same direction as A. C and G face opposite direction to each
other. K does not face outside.
31. How many persons sit to the right of the one, who sits
immediate left of J? 36. What is the position of P with respect to K?
(a) Ten (b) Seven (c) None of these (a) 3rd to the right
(d) Eight (e) Eleven (b) 2nd to the left
(c) Immediate left
Directions (32-35): In these questions, relationships (d) 3rd to the left
between different elements are shown in the statements. (e) 5th to the right
These statements are followed by two conclusions. Give
37. How many persons sit between J and H, when counted
answer
from the left of H?
(a) If only conclusion I follows.
(a) Five (b) Six (c) Four
(b) If only conclusion II follows. (d) One (e) Three
(c) If either conclusion I or II follows
(d) If neither conclusion I nor II follows. 38. Four of the following five are alike in a certain way and
(e) If both conclusions I and II follow. so form a group. Find the one who does not belong to
that group?
32. Statements: Z > O = G < I  S > P (a) C-H (b) L-K (c) B-J
Conclusions: I. S > O II. P > G (d) H-L (e) B-P
33. Statements: K  M > W  T  Y< Q 39. How many persons sit between G and H, when counted
Conclusions: I. T < Q II. T < K from the left of G?
(a) Three (b) Five (c) Two
34. Statement: J  V < R > M, L> M = I  H (d) Four (e) None of these
Conclusions: I. V  H II. H  M
40. How many persons face outside from the centre?
35. Statement: I = H  B  N < D > L (a) Three (b) Four (c) None of these
Conclusions: I. B < N II. L > H (d) Six (e) Five

Quantitative Aptitude

Directions (41-46): Pie chart shows the percentage 41. Find average number of students appeared in shift I, II
distribution of total students appeared in six different & IV of the exam.
shifts of an exam. Study the pie chart given below and (a) 1040 (b) 900 (c) 720
answer the following questions.
(d) 1140 (e) 880
Total students appeared in exam - 5500
42. Find the central angle for students appeared in shift II
VI I of the examination.
18% 20% (a) 64.2° (b) 48° (c) 57.6°
V (d) 43.6° (e) 52.8°
10% II
16% 43. Find total number of students appeared in shift V & VI
IV
12% together of the examination.
III (a) 1740 (b) 1600 (c) 1820
24% (d) 1960 (e) 1540

17 www.bankersadda.com | www.sscadda.com | www.careerpower.in | Adda247 App


The IBPS RRB PO & Clerk 2021 | SUCCESS GUIDE
44. Students appeared in shift III & IV together of the Directions (52-56): In the given questions, two equations
examination are what percent more or less than (I) & (II) are given. You have to solve both the equations
students appeared in shift I of the examination? and mark the answer accordingly.
(a) 90% (b) 80% (c) 70% 52. I. x 2 + 9x + 20 = 0
(d) 50% (e) 60% II. 8y 2 − 15y + 7 = 0
45. Find ratio of students appeared in shift IV & VI together (a) x < y (b) x > y (c) x ≤ y
of the examination to students appeared in shift II & III (d) x ≥ y (e) x = y or no relation.
together of the examination. 53. I. x 2 − 11x + 30 = 0
(a) 3:4 (b) 5:7 (c) 4:3 II. y 2 + 12y + 36 = 0
(d) 7:5 (e) None of the above. (a) x < y (b) x > y (c) x ≤ y
46. Students appeared in shift I & VI together of the (d) x ≥ y (e) x = y or no relation.
examination are how much more or less than students 54. I. x 2 + 13x + 40 = 0
appeared in shift III & V together of the examination? II. y 2 + 7y + 10 = 0
(a) 330 (b) 150 (c) 360 (a) x < y (b) x > y (c) x ≤ y
(d) 280 (e) 220 (d) x ≥ y (e) x = y or no relation.
47. A vessel contains mixture of milk and water in the ratio 55. I. x 2 − 20x + 91 = 0
of 7:1 respectively. 24 liters mixture is removed from II. y 2 + 16y + 63 = 0
the vessel and if the quantity of remaining milk in the (a) x < y (b) x > y (c) x ≤ y
vessel is 56 liters, then find quantity of water in the (d) x ≥ y (e) x = y or no relation.
vessel initially. 56. I. x 2 − x − 12 = 0
(a) 11 liters (b) 15 liters (c) 12 liters II. y 2 + 5y + 6 = 0
(d) 9 liters (e) 8 liters (a) x < y (b) x > y (c) x ≤ y
48. A & B together can complete a piece of work in 9 days. (d) x ≥ y (e) x = y or no relation.
Time taken by A alone to complete the same work is Directions (57-62): Study the table given below and
7.5 days less than time taken by B alone to complete answer the following questions.
the same work. In how many days B alone will
2 Table gives information about total number of students in
complete 9 of the work? 3 different schools in 1999 & 2000 and also gives
(a) 8 days (b) 6 days (c) 7 days information about total number of girls in these 3 schools
(d) 5 days (e) 4 days in 1999 & 2000.
49. Ratio of ages of A and B, 4 years later is 8:9 Year
respectively. If average of present ages of A & B is 47 1999 2000
School
years, then find difference in present ages of A & B. Total Total Total Total
(a) 5 years (b) 6 years (c) 3 years students Girls students girls
(d) 2 years (e) 4 years A 720 360 900 450
B 360 180 600 180
50. There are 75% boys out of total students (boys + girls) C 450 270 400 120
in a school and 39% of the total students of the school
went on a picnic. If 32% of the total boys went on a Note: Total students in any school in any year = Total (Boys
picnic, then find what percent of total girls went on a + Girls) in that school in that year.
picnic? 57. If average number of students in school A in 1999,
(a) 60% (b) 90% (c) 75% 2000 & 2001 are 700, then find total number of
(d) 80% (e) 50% students in school A in 2001.
51. Number of passed students in an exam in section A & B (a) 540 (b) 480 (c) 420
are 240 & 210 respectively. If in section A 40% of the (d) 600 (e) 360
total students got failed and in section B 30% of the 58. Number of girls in school – A & B together in 2000 are
total students got failed, then find difference between what percent more or less than total number of
total number of students in section A & B. students in school – B & C together in 2000?
(a) 40 (b) 80 (c) 150 (a) 27% (b) 42% (c) 37%
(d) 120 (e) 100 (d) 30% (e) 45%

18 www.bankersadda.com | www.sscadda.com | www.careerpower.in | Adda247 App


The IBPS RRB PO & Clerk 2021 | SUCCESS GUIDE
59. Find total number of boys in school – A, B & C together 65. A man invested Rs. P at 12% p.a. on simple interest for
in 1999. two years.
(a) 720 (b) 640 (c) 680 Quantity I. If at the end of second year he gets
Rs.1200 as interest, then find Rs.P.
(d) 760 (e) 800
Quantity II. Rs.6000
60. Average number of students in school – A, B & C in 66. Ploughing cost of a rectangular field is Rs.288 at the
1999 are what percent of total students in school – B in rate of Rs.3 per square meter. Length of the field is 4
2000? meters more than the width of field.
(a) 95% (b) 85% (c) 75% Quantity I. Length of rectangular field.
(d) 55% (e) 65% Quantity II. 12 meters.

61. Find ratio of number of boys in school – B in 2000 to 67. Quantity I. Sum of present ages of Shivam and
Prashant is 32 years and Shivam is 8
number of boys in school – C in 2000.
years older than Prashant. Find present
(a) 5:4 (b) 4:5 (c) 2:3 age of Prashant.
(d) 3:2 (e) None of the above. Quantity II. 15 years.
62. Total number of girls in school – A, B & C together in 68. ‘A’ invested Rs.4000 and ‘B’ invested Rs.1000 more
1999 are how much more or less than total number of than A. After eight months ‘C’ invested Rs.3000. If at the
girls in school – A, B & C together in 2000? end of the year ‘C’ gets profit of Rs.700, then find the
(a) 140 (b) 60 (c) 180 total profit.
(a) Rs.7000 (b) Rs.8400 (c) Rs.5600
(d) 90 (e) 100
(d) Rs.8800 (e) Rs.6400
Directions (63-67): In the following questions, calculate 69. 440 meters long train passes a platform in 80 seconds.
quantity I and quantity II, compare them and answer If speed of train is increased by 3 m/sec, then it crosses
according to the following options. a pole in 22 seconds. Find the length of platform.
(a) If Quantity I > Quantity II (a) 720m (b) 840m (c) 700m
(b) If Quantity I < Quantity II (d) 920m (e) 900m
(c) If Quantity I ≥ Quantity II 70. Selling price of an article becomes Rs.2160 after giving
(d) if Quantity I ≤ Quantity II two successive discounts of x% and 25% and marked
(e) if Quantity I = Quantity II or no relation can be price of article is Rs.3600. Find the cost price of article
established if there is a profit of x% on selling the article after
giving two successive discounts.
63. Quantity I. Profit earned on selling an article at Rs. (a) Rs. 1720 (b) Rs.1500 (c) Rs.1600
450 at 20% profit (d) Rs.1800 (e) Rs.1900
Quantity II. Cost price of the article which is sold at 71. Three are 5 green balls, 7 blue balls and 3 red balls in a
Rs.84 on 20% profit bag. If 2 balls are chosen randomly from the bag, then
find the probability that at least one ball is green ball.
64. In a village there are 60% males and rest are females. 1 2 3
30% of total male are illiterate and 25% of total female (a) 9 (b) 7 (c) 8
3 4
are illiterate. Number of illiterate males is 1152. (d) 5 (e) 7
Quantity I. Literate females in the village.
72. Speed of boat in still water is six times of speed of
Quantity II. 1940 stream. If boat covers 210 km in upstream in 7 hours,
then find the downstream speed of boat?
(a) 42 km/hr. (b) 36 km/hr. (c) 30 km/hr.
(d) 32 km/hr. (e) 24 km/hr.
73. Length of rectangle ‘A’ is 125% of its breadth and area
of rectangle ‘A’ is 1280 cm². If width of rectangle ‘A’ is
half of the side of a square, then find perimeter of
square.
(a) 72m (b) 64m (c) 84m
(d) 96m (e) 60m
74. The average weight of a class of 45 girls is 53 kg. It was
later found that weight of two girls was read as 49 kg
and 57 kg instead of 45 kg and 52 kg. Find the actual
average weight of the class.
(a) 54 kg (b) 53.40 kg (c) 50.6 kg
(d) 52.80 kg (e) 51.5 kg
19 www.bankersadda.com | www.sscadda.com | www.careerpower.in | Adda247 App
The IBPS RRB PO & Clerk 2021 | SUCCESS GUIDE
Directions (75-80): Find the value of (?) in the following
78. 150, 152, 157, 167, 184, ?
number series.
(a) 229 (b) 245 (c) 232
75. 1.5, 3, 12, 72, 576, ? (d) 210 (e) 206
(a) 5480 (b) 5620 (c) 5580
(d) 5340 (e) 5760 79. 3.5, 2.5, 3, 6, 20, ?
76. 80, 66, 85, 61, 90, ? (a) 95 (b) 80 (c) 65
(a) 50 (b) 56 (c) 64 (d) 75 (e) 90
(d) 60 (e) 63
80. 6300, ?, 525, 105, 17.5, 2.5
77. 163, ?, 43, 23, 13, 8 (a) 2400 (b) 2100 (c) 4200
(a) 92 (b) 83 (c) 78 (d) 5200 (e) 3600
(d) 54 (e) 69

Solutions

REASONING ABILITY

1. (c): 2145673 Solutions (13-15):


0226484
Solutions (2-6):
Floors Flat-P Flat-Q
4 D H
3 G F
2 C A
1 B E
2. (b): 3. (b): 4. (e):
5. (d): 6. (d):
Solutions (7-11): 13. (e):
14. (b):
Words Codes 15. (a)
Plan mn
Solutions (16-20):
To kr/pc
Go pc/kr
Exam oj
Easy ly
Today si 16. (a): 17. (d): 18. (e):
Your zm 19. (a): 20. (e):
Make rk
21. (d):
7. (c): 8. (d): 9. (d): Days Persons
10. (a): 11. (c): Monday Q
Tuesday U
12. (c): Wednesday T
Thursday S
Friday P
Saturday R

20 www.bankersadda.com | www.sscadda.com | www.careerpower.in | Adda247 App


The IBPS RRB PO & Clerk 2021 | SUCCESS GUIDE
Solutions (22-26): 30. (b): 31. (c):
Boxes
32. (a): I. S > O (True) II. P > G(False)
T
Q 33. (e): I. T < Q (True) II. T < K (True)
Y 34. (b): I. V  H(False) II. H  M (True)
O
M 35. (d): I. B < N (False) II. L > H (False)
X Solutions (36-40):
K
L
P
J

22. (d): 23. (e): 24. (c):


25. (b): 26. (d):
Solutions (27-31):

36. (d): 37. (e): 38. (d):


27. (c): 28. (b): 29. (e): 39. (e): 40. (b):

Quantitative Aptitude

1
41. (e): Required average = × (5,500 ×
20+16+12
) = 880 47. (a): ATQ,
3 100
Let quantity of milk and water in the vessel
16
42. (c): Required angle = 100 × 360 ̊ = 57.6 ̊ initially be 7x liters & x liters respectively.
10+18
ATQ,
43. (e): Required number of students = 5,500 × = 7𝑥
100 (7𝑥 − 24 × 8𝑥) = 56
1,540
𝑥 = 11
44. (b): Students appeared in shift III & IV together of the
(24+12) 48. (d): Let time taken by B alone to complete the work
examination = 5,500 × = 1,980
100 be x days.
Students appeared in shift I of the examination = So, time taken by A alone to complete the same
20
5,500 × 100 = 1,100 work = (𝑥 − 7.5) days
1980−1100
Required percentage = × 100 = 80% ATQ,
1100 1 1 1
Or, required percentage =
(24+12)−20
× 100 = + =
𝑥−7.5 𝑥 9
20 45
80% 𝑥 = 3, 2
45. (a): Students appeared in shift IV & VI together of the x cannot be 3 as time taken by A alone cannot be
12+18
examination = 5,500 × 100 = 1,650 negative.
2

Students appeared in shift II & III together of the Required time = 1
9

16+24 45
examination = 5,500 × = 2,200 2
100
1650 = 5 days
Required ratio = = 3:4
2200
(12+18)
Or required ratio = (16+24) = 3 : 4 49. (b): Let ages of A & B, 4 years later be 8x years & 9x
years respectively.
46. (e): Students appeared in shift I & VI together of the ATQ,
20+18 (8𝑥 − 4) + (9𝑥 − 4) = 47 × 2
examination = 5,500 × 100 = 2,090
Students appeared in shift III & V together of the 17𝑥 = 102
10+24 𝑥 = 6 years
examination = 5,500 × 100 = 1,870
Required difference = 9𝑥 − 8𝑥 = 6 years
Required difference = 2090 − 1870 = 220
21 www.bankersadda.com | www.sscadda.com | www.careerpower.in | Adda247 App
The IBPS RRB PO & Clerk 2021 | SUCCESS GUIDE
50. (a): Let total students in the school be 100x. II. 𝑦 2 + 16𝑦 + 63 = 0
So, number of students went on the picnic = 39x 𝑦 2 + 9𝑦 + 7𝑦 + 63 = 0
And, number of boys went on the picnic = 𝑦(𝑦 + 9) + 7(𝑦 + 9) = 0
32
75𝑥 × 100 = 24x (𝑦 + 9)(𝑦 + 7) = 0
So, number of girls went on the picnic = 39𝑥 − 𝑦 = −7, −9
24𝑥 = 15x So, 𝑥 > 𝑦.
15𝑥
Required percentage = × 100 = 60%
25𝑥 56. (e): I. 𝑥 2 − 𝑥 − 12 = 0
51. (e): Total number of students in section A = 𝑥 2 − 4𝑥 + 3𝑥 − 12 = 0
100 𝑥(𝑥 − 4) + 3(𝑥 − 4) = 0
(240 × 60 ) = 400
(𝑥 − 4)(𝑥 + 3) = 0
Total number of students in section B =
100 𝑥 = 4, −3
(210 × 70 ) = 300
II. 𝑦 2 + 5𝑦 + 6 = 0
Required difference = 400 − 300 = 100 𝑦 2 + 3𝑦 + 2𝑦 + 6 = 0
52. (a): I. 𝑥 2 + 9𝑥 + 20 = 0 𝑦(𝑦 + 3) + 2(𝑦 + 3) = 0
𝑥 2 + 5𝑥 + 4𝑥 + 20 = 0 (𝑦 + 3)(𝑦 + 2) = 0
𝑥(𝑥 + 5) + 4(𝑥 + 5) = 0 𝑦 = −2, −3
(𝑥 + 5)(𝑥 + 4) = 0 So, 𝑛𝑜 𝑟𝑒𝑙𝑎𝑡𝑖𝑜𝑛.
𝑥 = −4, −5
II. 8𝑦 2 − 15𝑦 + 7 = 0 57. (b): Required number of students = (700 × 3) −
2
8𝑦 − 8𝑦 − 7𝑦 + 7 = 0 (720 + 900) = 480
8𝑦(𝑦 − 1) − 7(𝑦 − 1) = 0
(𝑦 − 1)(8𝑦 − 7) = 0 58. (c): Number of girls in school – A & B together in
7 2000 = 450 + 180 = 630
𝑦 = 1, 8
Total number of students in school – B & C
So, 𝑥 < 𝑦.
together in 2000 = 600 + 400 = 1000
53. (b): I. 𝑥 2 − 11𝑥 + 30 = 0 Required percentage =
1000−630
× 100 = 37%
2 1000
𝑥 − 6𝑥 − 5𝑥 + 30 = 0
𝑥(𝑥 − 6) − 5(𝑥 − 6) = 0 59. (a): Required number of boys = (720 − 360) +
(𝑥 − 6)(𝑥 − 5) = 0 (360 − 180) + (450 − 270)
𝑥 = 5, 6 = 360 + 180 + 180
II. 𝑦 2 + 12𝑦 + 36 = 0 = 720
𝑦 2 + 6𝑦 + 6𝑦 + 36 = 0
𝑦(𝑦 + 6) + 6(𝑦 + 6) = 0 60. (b): Average number of students in school – A, B & C
(𝑦 + 6)(𝑦 + 6) = 0 1
in 1999 = 3 × (720 + 360 + 450) = 510
𝑦 = −6 510
So, 𝑥 > 𝑦. Required percentage = 600 × 100 = 85%
54. (c): I. 𝑥 2 + 13𝑥 + 40 = 0 600−180
61. (d): Required ratio = 400−120
𝑥 2 + 8𝑥 + 5𝑥 + 40 = 0 420
𝑥(𝑥 + 8) + 5(𝑥 + 8) = 0 = = 3:2
280
(𝑥 + 8)(𝑥 + 5) = 0
𝑥 = −8, −5 62. (b): Total number of girls in school – A, B & C
II. 𝑦 2 + 7𝑦 + 10 = 0 together in 1999 = (360 + 180 + 270) = 810
𝑦 2 + 5𝑦 + 2𝑦 + 10 = 0 Total number of girls in school – A, B & C
𝑦(𝑦 + 5) + 2(𝑦 + 5) = 0 together in 2000 = (450 + 180 + 120) = 750
(𝑦 + 5)(𝑦 + 2) = 0 Required difference = 810 − 750= 60
𝑦 = −2, −5
So, 𝑥 ≤ 𝑦. 63. (a): Quantity I:
20
Required profit = 450 × 120 = Rs.75
55. (b): I. 𝑥 2 − 20𝑥 + 91 = 0
𝑥 2 − 13𝑥 − 7𝑥 + 91 = 0 Quantity II:
100
𝑥(𝑥 − 13) − 7(𝑥 − 13) = 0 Required cost price = 84 × = Rs.70
120
(𝑥 − 13)(𝑥 − 7) = 0
So, Quantity I > Quantity II.
𝑥 = 7, 13
22 www.bankersadda.com | www.sscadda.com | www.careerpower.in | Adda247 App
The IBPS RRB PO & Clerk 2021 | SUCCESS GUIDE
64. (b): Quantity I: l = 1360 – 440
100 40 100−25
Required female = 1152 × × × = l = 920 m
30 60 100
1920 70. (d): ATQ,
Quantity II: 75 (100 − 𝑥)
2160 = 3600 × 100 × 100
1940
So, Quantity I < Quantity II. 2160 = 2700 – 27x
27x = 540
65. (b): Quantity I: x = 20
ATQ, 100
𝑃×12×2 So, required amount = 2160 × 120 = Rs. 1800
100
= 1200
𝑃 = 5,000 Rs. 71. (e): Possible cases = 1 green ball or 2 green balls
Quantity II: Required probability = 1
5𝐶 × 10𝐶1 5𝐶
+ 2
Rs.6,000 15𝐶2 15𝐶2
5 ×10 10 50 10 60 4
So, Quantity I < Quantity II. = 15 + 15 = 105 + 105 = 105 = 7
𝐶2 𝐶2
66. (e): Let breadth of the field be x m.
So, length of the field = (𝑥 + 4) m 72. (a): Let speed of stream be x km/hr.
288 So, speed of boat in still water = 6x km/hr.
Area of a rectangular field = 3 = 96 m2
ATQ,
ATQ, 𝑥(𝑥 + 4) = 96 210
𝑥 2 + 4𝑥 − 96 = 0 7
= (6x - x)
𝑥 2 + 12𝑥 − 8𝑥 − 96 = 0 ⇒ 5x = 30
𝑥(𝑥 + 12) − 8(𝑥 + 12) = 0 x = 6 km/hr
(𝑥 + 12)(𝑥 − 8) = 0 So, required downstream speed of boat = (6x + x)
𝑥 = 8, −12 = 7x = 42 km/hr
Quantity I:
Length of rectangular field = 12m 73. (b): Let width of rectangle A be ‘4x meters’
125
Quantity II: 12 m So, length of rectangle A = 4x × 100 = 5x meters
So, Quantity I = Quantity II. ATQ,
67. (b): Quantity I: 4x × 5x = 1280
Let present age of Prashant be x years. 20x² = 1280
So, present age of Shivam = (𝑥 + 8) years x² = 64
𝑥 + 8 + 𝑥 = 32 x=8
𝑥 = 12 years Hence, side of square = 2 × 8 = 16 cm
Quantity II: Required perimeter = 4 × 16 = 64 cm
15 years [(49+57)–(45+52)]
So, Quantity I < Quantity II. 74. (d): Required average = 53 –
45
9
68. (a): Profit sharing ratio of A, B & C = (4000 × 12) : = 53 –
45
(4000 +1000) × 12 : (3000 × 4) = 52.80 kg
= 48000 : 60000 : 12000
=4:5:1 75. (e): Missing number = 5760
Let total profit be Rs. P Pattern of series –
ATQ, 1.5 × 2 = 3
1
× 𝑃 = 700 3 × 4 = 12
(4 + 5 + 1) 12 × 6 = 72
P = Rs. 7000 72 × 8 = 576
69. (d): Let speed of train be ‘V’ m/sec’ 576 × 10 = 5760
And let length of platform be ‘l meters.
𝑙+ 440 76. (b): Missing number = 56
ATQ, = V … (i) Pattern of series –
80
And, 80 − 14 = 66
440
22
=V+3 66 + 19 = 85
⇒ V = 17 … (ii) 85 − 24 = 61
Put value of (ii) in (i), 61 + 29 = 90
𝑙 + 440
= 17 90 − 34 = 56
80
23 www.bankersadda.com | www.sscadda.com | www.careerpower.in | Adda247 App
The IBPS RRB PO & Clerk 2021 | SUCCESS GUIDE
77. (b): Missing number = 83 79. (a): Missing number = 95
Pattern of series – Pattern of series –
163 − 80 = 83 3.5 × 1 − 1 = 2.5
83 − 40 = 43 2.5 × 2 − 2 = 3
43 − 20 = 23 3×3−3 = 6
23 − 10 = 13 6 × 4 − 4 = 20
13 − 5 = 8 20 × 5 − 5 = 95

78. (d): Missing number = 210 80. (b): Missing number = 2100
Pattern of series – Pattern of series –
?= 6300 ÷ 3 = 2100
2100 ÷ 4 = 525
525 ÷ 5 = 105
105 ÷ 6 = 17.5
17.5 ÷ 7 = 2.5

24 www.bankersadda.com | www.sscadda.com | www.careerpower.in | Adda247 App


The IBPS RRB PO & Clerk 2021 | SUCCESS GUIDE

IBPS RRB PO PRELIMS MEMORY BASED PAPER 2019


REASONING ABILITY

Direction (1-5): Study the following information carefully 6. Statements: Only a few Palace is Home.
and answer the questions given below: All Home is Office.
No Office is Building.
Nine persons i.e. P, Q, R, S, T, U, V, W, X were born on Conclusion I. All Palace is Home is a possibility.
different months i.e. January, March, April, May, July, II. Some Palace is Building.
August, September, October, November but not necessarily
7. Statements: All Men is Women. Some Child is Women.
in same order.
No Men is Boy.
Four persons were born between P and T. P was born Conclusion I. Some Men is Child.
before T. Q was born in the month of 30 days after July. T II. No Men is Child.
was born after Q and before R. There were as many persons 8. Statements: No Professor is Student. Only a few
born before X as after R. one person was born between U Student is Lecturer. All Lecturer is
and V. S was born before U and after W. Principal.
Conclusion I. All Professor is Principal is a
1. How many persons were born between X and V?
possibility.
(a) Two (b) Three (c) One
II. All Student is Lecturer is a possibility.
(d) Four (e) More than four
9. Statements: Only a few Palace is Home.
2. Who among the following was born on August? All Home is Office.
(a) R (b) S (c) T No Office is Building.
(d) P (e) None of these Conclusion I. Some Home is Building.
II. No Home is Building.
3. In which of the following month S was born?
(a) March (b) April (c) June 10. Statements: No Professor is Student.
(d) October (e) None of these Only a few Student is Lecturer.
All Lecturer is Principal.
4. If W is related to April, V is related to July then, P is Conclusion: I. Some Student is Principal.
related to which of the following? II. Some Lecturer is Professor.
(a) March (b) May (c) June
Direction (11-15): Study the following information
(d) August (e) October
carefully and answer the questions given below:
5. Four of the following five are alike in certain way
Eight persons are sitting around a square table. Four
based from a group, find the one which does not persons are sitting at middle of the sides of the square and
belong to that group? all are facing towards inside. Remaining four are sitting at
(a) R (b) S (c) T corners and they face outside.
(d) U (e) V
Two persons are sitting between P and U. R who is an
Direction (6-10): In each of the questions below are given immediate neighbor of P, sits opposite to S. T sits 3rd to the
some statements followed by two conclusions. You have to right of V. W sits immediate right of T. Q faces W.
take the given statements to be true even if they seem to be 11. Who among the following sits opposite to T?
at variance with commonly known facts. Read all the (a) P (b) R (c) S
conclusions and then decide which of the given conclusions (d) W (e) None of these
logically follows from the given statements disregarding
12. How many persons are sitting between P and V when
commonly known facts.
counted from left of P?
(a) If only conclusion I follows. (a) Two
(b) If only conclusion II follows. (b) Three
(c) If either conclusion I or II follows. (c) Four
(d) If neither conclusion I nor II follows. (d) Either (a) or (c)
(e) If both conclusions I and II follow. (e) None of these
25 www.bankersadda.com | www.sscadda.com | www.careerpower.in | Adda247 App
The IBPS RRB PO & Clerk 2021 | SUCCESS GUIDE
13. What is the position of Q with respect to R? Directions (21-25): Read the following information
(a) Immediate right carefully and answer the questions given below:
(b) Immediate left Twelve people are sitting in two parallel rows containing
(c) 2nd to the right six people each in such a way that there is an equal
(d) 2 to the left
nd distance between adjacent persons. In row 1 – P, Q, R, S, T
(e) None of these and U are seated (but not necessarily in the same order)
and all of them are facing south. In row 2 – A, B, C, D, E and
14. Who among the following person sit 3rd to the right of F are seated (but not necessarily in the same order) and all
Q? of them are facing North. Therefore, in the given seating
(a) P (b) U (c) R arrangement each member seated in a row faces another
(d) S (e) None of these member of the other row. P faces D. U does not face A, who
sits left to E but not immediate left. R sit at one of the ends
15. Four of the following five are alike in certain way and diagonally opposite to B. Three persons sit between B
based from a group, find the one which does not and F, who does not face U. C sits immediate left to D but
belong to that group? does not faces S. Two persons sit between Q and U, none of
them sits at the end. The one who faces T sits 2nd right to A.
(a) Q (b) R (c) S
(d) T (e) U 21. Who among the following faces A?
(a) S (b) T (c) Q
Direction (16-17): Study the following information (d) R (e) none of these
carefully and answer the questions given below:
22. How many persons sit to the right of R?
Eight members are living in a family. Q is the only son of P. (a) No One (b) one (c) two
T is wife of U. T is sister of Q and R. V is daughter in law of (d) three (e) four
W. S is son of T. W is the mother of Q. 23. Four of the following five form a group ,who among
the following does not belongs to that group?
16. How is S related to R? (a) U (b) T (c) E
(a) Son (b) Daughter (c) Nephew (d) F (e) A
(d) Niece (e) Can’t be determined
24. If in a certain way R is related to C, T is related to E,
17. How many male members are in the family? then who among the following is related to D?
(a) Four (b) Five (c) Three (a) U (b) T (c) E
(d) Six (e) None of these (d) F (e) Q
25. Who among the following sit 3rd right to U?
18. How many such numerals are there in the number (a) R (b) T (c) P
‘254136987’ which will remain at the Same position (d) S (e) Q
when arranged in ascending order from left to right?
Directions (26-30): Study the following information
(a) one (b) two (c) three
carefully and answer the questions given below:
(d) four (e) None of these In a certain code language
19. How many pairs of letters are there in the word ‘left right centre’ is written as ‘yo vo na’,
‘ahead below behind’ is written as ‘sa ra la’,
‘EDUCATION’, each of which have as many letters
‘above centre right’ is written as ‘ha vo na’, and
between then in the word as they have between them ‘behind below above’ is written as ‘ha ra la’.
in the English alphabet?
26. What is the code for ‘left’?
(a) one (b) two (c) three
(a) sa (b) ha (c) yo
(d) four (e) more than four (d) na (e) None of these
20. If four letter word is formed from 1st, 3rd, 5th and 6th 27. ‘behind’ will be written as?
letter of TRANSLATE then what is the 3rd letter of (a) ra (b) ha (c) la
newly formed word? If more than one meaningful (d) Either (a) or (c) (e) None of these
word is formed, then the answer will be Z. 28. What is the code for ‘ahead’?
(a) L (b) T (c) A (a) sa (b) yo (c) la
(d) S (e) Z (d) ha (e) Can’t be determined

26 www.bankersadda.com | www.sscadda.com | www.careerpower.in | Adda247 App


The IBPS RRB PO & Clerk 2021 | SUCCESS GUIDE
29. What does ‘ha’ stand for? (c) Number of boxes between J and S is four
(a) behind (b) below (c) ahead (d) One of the boxes above J is K
(d) above (e) None of these (e) One box is kept between J and M
30. What is the code for ‘centre’? 34. Number of boxes above K is one less than the number
(a) la (b) yo (c) sa of boxes below ________?
(d) ha (e)Can’t be determined (a) S (b) R (c) F
(d) D (e) None of these
Directions (31-35): Study the following information and
answer the questions given below: 35. How many boxes are there between M and H?
(a) One (b) Two (c) Three
There are eleven boxes placed one above the other. Five (d) None (e) More than three
boxes are placed between F and T. Not more than five
boxes are kept above T. Two boxes are kept between T and Directions (36-40): In each of the question, relationships
M. Three boxes are kept between M and S and M is kept at between some elements are shown in the statements.
one of the positions above S. There are only three boxes These statements are followed by conclusions numbered I
and II. Read the statements and give the answer.
kept above the box J. One box is kept between R and S. Two
(a) If only conclusion I follows.
boxes are kept between R and H. Box D is kept at one of the
(b) If only conclusion II follows.
positions below box K and at one of the positions above box
(c) If either conclusion I or II follows.
C which is not above R. Box E is kept immediately above K.
(d) If neither conclusion I nor II follows.
31. How many boxes are placed between J and R? (e) If both conclusions I and II follow.
(a) 5 (b) 6 (c) 3
36. Statements: C ≤ L = E ≤ R ≤ K = P ≥ O
(d) 4 (e) None of these Conclusions: I. P = C II. C < P
32. Which of the following statement is true regarding C? 37. Statements: W > A = S ≥ H < I ≤ N ≤ G
(a) C is placed at one of the positions above D Conclusions: I. H < W II. G > H
(b) C is placed immediately below F.
(c) R is placed just above C 38. Statements: C < O ≤ D = S > A ≥ P ≥ Q
(d) C is placed at the bottom most position Conclusions: I. Q < D II. C < A
(e) None of these 39. Statements: F ≤ B = I ≤ C = A ≥ S > E
33. Which of the following is not true regarding J? Conclusions: I. S ≥ B II. F > E
(a) J is immediately below box T 40. Statements: I ≥ N = T ≥ E > L ≥ G > M
(b) One of the boxes below J is D Conclusions: I. G < N II. I ≥ L

Quantitative Aptitude

Directions (41-45): Study the table given below and 43. If total no. of employee in E is 25% more than D and
answer the following Question no. of employee in HR dept is same as in company C,
then employee other than HR dept in company E is
what % of other dept employee in company B.
(a) 60% (b) 80% (c) 75%
(d) 50% (e) 55%
44. Find the difference between males of HR dept in
company C and D together and females of HR dept in
41. Find the average no. of Females in HR department company B and C together ?
together ? (a) 36 (b) 42 (c) 48
(a) 54 (b) 46 (c) 49 (d) 40 (e) 30
(d) 50 (e) 52
45. Find the average no. of employee other than HR dept.
42. Females in the HR dept of company C is what % more
than male in HR department of company A ? in A, B and C together ?
(a) 250% (b) 200% (c) 100% (a) 280 (b) 270 (c) 220
(d) 300% (e) 150% (d) 300 (e) 240

27 www.bankersadda.com | www.sscadda.com | www.careerpower.in | Adda247 App


The IBPS RRB PO & Clerk 2021 | SUCCESS GUIDE
46. If there are total 150 females in company C then how 54. If ratio of ages of P and Q before 4 year ago is 5 : 4 and
many female employees are there other than females after 12 years sum of their ages will be 68 years, their
of HR department what was P’s age 2 years ago ?
(a) 90 (b) 100 (c) 80
(d) 110 (e) 120 (a) 24 years (b) 22 years (c) 18 years
(d) 26 years (e) 20 years
Directions (47-51): Find the missing term in the following
number series: 55. If Pipes A and B can fill a tank in 15 min and 20 mins
47. 1864, 1521, 1305, ? , 1116, 1089 respectively and pipe C empties the tank in 12 mins.
(a) 1160 (b) 1180 (c) 1095 what will be the time taken by A, B and C together to
(d) 1205 (e) 1220 fill the tank completely?
48. 18, ?, 9, 18, 72, 576 (a) 25 min (b) 30 min (c) 40 min
(a) 12 (b) 9 (c) 18 (d) 20 min (e)35 min
(d) 10 (e) 6
Directions (56-60): Solve the given quadratic equations
49. 12, 6.5, 7.5, 12.75, 27.5, ?
and mark the correct option based on your answer—
(a) 66.5 (b) 68.75 (c) 63.75
(d) 71.25 (e) None of these (a) x > y
(b) x < y
50. 5 , 15, 50, ?, 1030, 6185
(a) 210 (b) 205 (c) 225 (c) x ≥ y
(d) 200 (e) 195 (d) x ≤ y
(e)x = y or there is no relationship
51. 130, 154, 186 , ? , 274, 330
(a) 216 (b) 220 (c) 240 56. (i) x² = 81 (ii) y² – 18y + 81 = 0
(d) 226 (e) 230
52. If a boat travels 18 km more in downstream than in 57. (i) 4x² - 24x + 32 = 0 (ii) y² - 8y + 15 = 0
upstream in 3 hr. and if the speed of the Boat in still 58. (i) x² - 21x + 108 = 0 (ii) y² – 17y + 72 = 0
water is 20 km/hr. find the distance travelled by boat
in downstream in 4 hr. ? 59. (i) x² – 11x + 30 = 0 (ii) y² - 15y + 56 = 0
(a) 86 (b) 92 (c) 68
(d) 96 (e) None of these 60. (i) x³ = 512 (ii) y² = 64
53. If A invested Rs. 12000 at some rate of interest of S.I 61. If a shopkeeper marks an item 50% above its CP and
and B joined him after 3 months investing 16000 at if 12% discount is given on the marked price and the
same rate of interest if A leaves before 2 month of
shopkeeper makes profit of 256 Rs, then what will be
completion, then what will be the share of B’s profit
after 1 year if total profit is 22000 Rs. ? the actual cost price of the item?
(a) 10000 (b) 14000 (c) 12000 (a) 1000 Rs. (b) 800 Rs. (c) 750 Rs.
(d) 8000 (e) 11000 (d) 1200 Rs. (e) 900 Rs.

Directions (62-67):The line graph shows the data of five seller selling an item(in units) on Monday and Tuesday.
450

400

350

300

250

200

150
A B C D E

Tuesday Monday

28 www.bankersadda.com | www.sscadda.com | www.careerpower.in | Adda247 App


The IBPS RRB PO & Clerk 2021 | SUCCESS GUIDE
62. The no. of item sold by A and C together is how much 70. Total students in art stream in A is what percent more
more or less then items sold by B and D together on than total students in science stream in B?
both days ? (a) 75% (b) 70% (c) 90%
(a) 250 (b) 280 (c) 300 (d) 100% (e) 110%
(d) 320 (e) 350 71. Find the ratio of total students in commerce stream in
63. What is the average no. of items sold by all five sellers B to total students in science stream in A?
(a) 8 : 15 (b) 8 : 17 (c) 8 : 13
on Monday ?
(d) 8 : 11 (e) 8 : 9
(a) 298 (b) 305 (c) 280
(d) 300 (e) 315 72. If in school C total students are720 students and total
students in science stream of school Care 25% more
64. Items sold by B and C on Tuesday together is what % than total students in commerce stream in school B,
more than same sellers on Monday together ? then find total students ofart & commerce stream in
(a) 25% (b) 30% (c) 20% school C is how much less than total students in art
(d) 15% (e) 24% and commerce stream in school A?
(a) 120 (b) 110 (c) 150
65. Find the difference between items sold by B, D, E on
(d) 100 (e) 140
Monday together items sold by B and E on Tuesday
together 73. Find the average number of students in science
(a) 150 (b) 180 (c) 160 stream in school A & B?
(d) 120 (e) 200 (a) 250 (b) 270 (c) 240
(d) 200 (e) 225
66. Item sold On Monday by C and E together is
approximately what percentage of total items sold by 74. If out of total students in art stream of school A & B
A and B together on Tuesday ? ratio of boys to girl is 5 : 3 and 7 : 4 respectively, then
(a) 71% (b) 80% (c) 55% find difference between boys and girls in art stream of
(d) 85% (e)65% school A & Btogether?
(a) 220 (b) 225 (c) 240
67. Find the difference between the average items sold by (d) 248 (e) 224
A and B together on Monday and average of items sold
by B and C together on Tuesday? 75. P invested 60% more than Q and R invested 20%
(a) 45 (b) 35 (c) 25 more than Q. If ratio of investment time-period (P: Q:
(d) 40 (e) 50 R) is 2: 4: 3 and the sum of profit shares of Q and R is
Rs. 8550 then find the profit share of P.
68. If A start from P with speed 60 km/hr at 8:00 am and (a) Rs. 3200 (b) Rs. 4000 (c) Rs. 2400
B starts with speed 70 km/hr. at 8 : 30 am from Q and (d) Rs. 3600 (e) Rs. 3000
total distance between P and Q is 680 km, find at what 76. When a person sold an article, his profit percent is
time they will cross each other? 60% of the selling price. If the cost price is increased
(a) 2 : 30pm (b) 1 : 30pm (c) 12 : 30pm by 75% and the selling price remains the same, then
(d) 3 : 00pm (e) 4 : 00pm find decrement in the profit is what percent of the
69. If a person invested 6000 at T% S.I for 3 year and selling price of the article?
(a) 25% (b) 30% (c) 40%
same amount at (T + 5)% CI for 2 year and difference
(d) 27.5% (e) None of these
between both interest is 60 Rs. then find T ?(in %)
(a) 15 (b) 18 (c) 20
(d) 24 (e) 25
Direction (70−𝟕𝟒): Read the data carefully and answer
the question.
There are 1800 students in two school ‘A’ & ‘B’ and three
3
streams in each school i.e. art, science & commerce.18 4 %
of total students in school A arein commerce stream and 28
4
7
% of total students in school B arein science stream. Sum
of total students in commerce stream in A & science stream
1
in B is 420. 19 % of total students in school B are in
21
commerce stream and 50% of total students in school A are
in Art stream.

29 www.bankersadda.com | www.sscadda.com | www.careerpower.in | Adda247 App


The IBPS RRB PO & Clerk 2021 | SUCCESS GUIDE
77. Area of Istcircle and circumference of IInd circle is 1386 2
79. An article is marked 66 3 % above the cost price and
cm2 and 176 cm respectively. There is a square whose
5 loss incurred on selling that article is 25% of the
side is 35 7 % of twice of sum of the radius of both the discount given on it. Then, find the discount % given?
circles. Find the perimeter of the square (in cm)? 1
(a) 48 3 %
1
(b) 53 3 %
1
(c) 58 3 %
(a) 132 (b) 136 (c) 140 1
(d) 116 (e) 124 (d) 63 3 % (e) 60 %
78. There are 5 red, 6 black and 5 blue balls in a bag. Out
80. A train travelling at 72 km/hr. classes a platform of
of these balls, four balls are picked at random from the
bag. Then, what is the probability that one is red, two 160 m in 18 second and another train travelling at 90
are black and one is blue ball? km/hr crosses the same platform in 15 second. Find
75 75 71 the length of another train?
(a) 362 (b) 364 (c) 362
70 5
(a) 160 m (b) 180 m (c) 140 m
(d) 363 (e) 26 (d) 200 m (e) 215 m

Solutions

REASONING ABILITY
Direction (1-5): 10. (a);
Months Persons
January X
March W
April P
May V Direction (11-15):
July S
August U
September Q
October T
November R
1. (a); 2. (e); 3. (e);
4. (b); 5. (a); 11. (e); 12. (b); 13. (b);
6. (d); 14. (d); 15. (a);
Direction (16-17):

7. (c);

16. (c); 17. (a);

8. (a); 18. (b);

9. (b); 19. (e);

30 www.bankersadda.com | www.sscadda.com | www.careerpower.in | Adda247 App


The IBPS RRB PO & Clerk 2021 | SUCCESS GUIDE
20. (e); Directions (31-35):
Directions (21-25): Boxes
E
K
T
J
H
M
D
R
F
21. (d); 22. (a); 23. (e); S
24. (e); 25. (e); C
Directions (26-30): 31. (c); 32. (d); 33. (c);
Word Code 34. (c); 35. (d);
Right/centre vo/na Direction (36-40):
Left yo 36. (c); I. P = C (False) II. C < P (False)
Below/behind ra/la
Ahead sa 37. (e); I. H < W (True) II. G > H (True)
above ha 38. (a); I. Q < D (True) II. C < A (False)
26. (c); 27. (d); 28. (a); 39. (d); I. S ≥ B (False) II. F > E (False)
29. (d); 30. (e); 40. (a); I. G < N (True) II. I ≥ L (False)

Quantitative Aptitude

41. (c); Average no. of females in HR dept therefore females other than in HR department
75 80 60 60
80×
100
+50× +100× +60×
100 100 100 = 150-60= 90
=
4
=
60+40+60+36
=
196
= 49 47. (b);
4 4
60
42. (b); Females in company C (HR) = 100× 100 = 60
25
Males in company A (HR) = 80 × 100 = 20
Difference = 60 – 20 = 40 48. (b); 18, ?, 9, 18, 72, 576
40
∴ % = 20 × 100 = 200% more 18 × 0.5 = 9
125 9×1=9
43. (c); Total employee in E = 200 × 100 = 250 9 × 2 = 18
∴ employee of HR dept in E = 100 18 × 4 = 72
∴ other employee = 150
100 72 × 8 = 576
∴% of other employee = 150 × 200 = 75%
49. (d); 12 × 0.5 + 0.5 = 6.5
44. (a); Males in HR dept in C and D 6.5 × 1 + 1 = 7.5
40 40
= 100 × 100 + 60 × 100 = 40 + 24 = 64 7.5 × 1.5 + 1.5 = 12.75
Females in HR dept of B and C 12.75 × 2 + 2 = 27.5
80 60
= 50 × 100 + 100 × 100 = 100 27.5 × 2.5 + 2.5 = 71.25
∴ Difference = 100 – 64 = 36
50. (b); 5 × 2 + 5 = 15
220+200+300 720
45. (e); Average of A, B, C = 3
= 3
= 240 15 × 3 + 5 = 50
46. (a); Total females in company C = 150 50 × 4 + 5 = 205
females in HR department in company C 205 × 5 + 5 = 1030
60
= 100× = 60 1030 × 6 + 5 = 6185
100

31 www.bankersadda.com | www.sscadda.com | www.careerpower.in | Adda247 App


The IBPS RRB PO & Clerk 2021 | SUCCESS GUIDE
51. (d); 57. (e); 4x² – 24x + 30 = 0
4x² – 16x – 8x + 32 = 0
4x (x – 4) –8 (x–4) = 0
x = 4, 2
y² – 8y + 15 = 0
52. (b); (Ds –Du ) 3 = 18 km y² – 5y – 3y + 15 = 0
Different in 1 hr. = 6km y(y – 5)–3 (y – 5) = 0
Ds and Du ∴ y = 5, 3
∴ Speed of boat in still water = 20 km/hr. ∴ No relation exists
Ds = 23 km/hr., Du = 17 km/hr.
Distance travelled = 4 × 23 = 92 km 58. (c); x² – 21x + 108 = 0
x² – 9x – 12x + 108 = 0
53. (c); x(x – 9) – 12 (x – 9) = 0
x = 9, 12
y² – 17y + 72 = 0
∴ y² – 8y – 9y + 72 = 0
y (y – 8) – 9 (y – 8) = 0
∴ y = 8, 9
6
∴ B’s share = 22000 × = 12000 ∴x≥y
11
59. (b); x² – 11x + 30 = 0
54. (b); P Q
x² – 6x – 5x + 30 = 0
–4 5 4 ∴ x(x – 6) – 5(x – 6) = 0
+12 P + Q = 68
x = 6, 5
Age increased in 16 year = 32 years
y² – 15y + 56 = 0
Sum of Age of P and Q before 4 years = 36
y² – 7y – 8y + 56 = 0
∴ 5x + 4x = 36 ⇒ X = 4
y (y – 7) – 8 (y – 7) = 0
P’s age 2 years ago = 5x + 2 = 22 years ∴ y = 7, 8
55. (b); ∴x<y
60. (c); x³ = 512
3
x = √512 = 8
y² = 64
∴ tank filled in 1 min = 2 units y = √64 = ± 8
60
Total time = = 30 minutes ∴x≥y
2
61. (b); Let CP = 100 x
56. (d); x² = 81 ∴ marked price = 150x
x=±9 ∴ selling price after giving discount = 132x
Y² – 18y + 81 = 0 ∴ 32x = 256 ⇒ x = 8
(y – 9)² = 0 ∴ CP = Rs 800
∴ y = 9, 9
∴x≤y 62. (b); Item sold by A and C = 550 + 570 = 1120
Item sold by B and D = 750 + 650 = 1400
∴ diff. =1400 – 1120 = 280
300+350+250+380+210 1490
63. (a); Average = = = 298
5 5

64. (c); Item sold by B and C on Monday = 350 + 250


= 600
Item sold by B and C on Tuesday =400+320
= 720
100
∴ % increase = 120 × = 20%
600

65. (d); Items sold on Monday by B, D and E


= 350 + 380 + 210 = 940
Item sold on Tuesday by B and E = 400 + 420
= 820
∴ diff = 940 – 820 = 120
32 www.bankersadda.com | www.sscadda.com | www.careerpower.in | Adda247 App
The IBPS RRB PO & Clerk 2021 | SUCCESS GUIDE
66. (a); Item sold by C and E on Monday And total students in school B in art stream
= 250 + 210 = 460 2
= 840 − × 840 − 160 = 440
7
Item sold by A and B together on Tuesday
= 400 + 250 = 650 Streams A B
100
∴ ? = 460 × 650 Art 480 440
≃ 71% (approx) Commerce 180 160
650 Science 300 240
67. (b); Avg. by A and B on Monday = 2 = 325
480 −240
Avg. of B and C on Tuesday =
720
= 360 70. (d); Required percentage = × 100 = 100%
2 240
Diff. = 360 – 325 = 35 160
71. (a); Required ratio = = 8 : 15
300
68. (b);
72. (e); Total student art & commerce stream in C
125
= 720 – 160 × = 520
100
Required difference = (480 + 180) – 520 = 140
1
Dist travelled by A in 2 hr = 30 km 73. (b); Required average =
300+240
2
Remaining distance to be covered = 680 – 30 540
= = 270
= 650 km 2
Relative speed = 60 + 70 = 130 74. (c); Total boys in art stream of school A & Btogether
650 5 7
∴ time taken = 130 = 5 hr = 480 × 8 + 440 × 11 = 300 + 280 = 580
∴ time = 8 : 30 + 5 hr = 1 : 30 pm Total girls in art stream of school A & B together
3 4
69. (a); By going with the options = 480 × 8 + 440 × 11 = 180 + 160 = 340
Interest received at SI =
6000×3×15
= 2700 Rs Required difference = 580 −340 = 240
100
∴ T + 5 = 20% 75. (d); Let the investment of Q = 100x
6000×44
Interest received after 2 yrs at CI = Investment of P = 160x
100
= 2640 Investment of R = 120x
∴ Difference = 2700 – 2640 = 60 Rs Ratio of profit:
T=15% P Q R
160x × 2 100x × 4 120x × 3
Direction (70−𝟕𝟒): 8 : 10 : 9
Let total students in A = x ATQ,
And, total students in B = y
75 19 unit = Rs. 8550
Total students in school A in commerce stream = x × 4 × 8 unit = 450 × 8 = Rs. 3600
1 3x
=
100 16 76. (b); Let the selling price be 250x
200
Total students in school B in science stream = y × × then, profit = 150x
7
1
=
2y CP=250x − 150x = 100x
100 7 175
3x
Given, 16 +
2y
= 420 … (i) Now, new C.P. = 100x × 100 = 175x
7
And x + y = 1800 … (ii) New S.P. = 250x
So, from (i) and (ii), New profit = 250x – 175x = 75x
150x−75x
Total students in school A = 960 Required % = × 100 = 30%
250x
And total students in school B = 840
Total students in school B in commerce stream 77. (c); Circumference of any circle = 2π × radius
400 1
= × × 840 = 160 Radius of 1st circle = √
1386
= 21 cm
21 100
π
Total students in school A in art stream 176
1
= 2 × 960 = 480 Radius of 2nd circle = 2π
= 28 cm
5
Now, total students in school A in science stream Side of square = 14 × 2 × (21 + 28) = 35 cm
3
= 960− × 960 − 480 = 300 Perimeter of square = 4 × 35 = 140 cm
16

33 www.bankersadda.com | www.sscadda.com | www.careerpower.in | Adda247 App


The IBPS RRB PO & Clerk 2021 | SUCCESS GUIDE
78. (b); Ways to select 4 balls out of 16 balls = 16C 4 3y = 2x
15
Ways to select one red balls = 5C1 Marked price=Rs y
2
Ways to select two black balls = 6C 2 4y
Required discount %=15 × 100 = 53 3 %
1
y
Ways to select one blue balls = 5C1 2
5
∴ Required probability 80. (e); Speed of 1 st train = 72 ×
18
= 20 m/s
5C 1 ×6C 2 ×5C 1
=
75
= 364 ∴ Dist travelled by 1st train = 20 × 18 = 360 m
16C 4
∴ length of train (1st) = 360 – 160 = 200 m
79. (b); Let the cost price be Rs 3x 5
Speed of 2nd train = 90 × 18 = 25 m/s
Then the marked price= Rs 5x
∴ Distance travelled = 25 × 15
And let the discount given be Rs 4y
= 375 m
Then loss incurred= Rs y
∴ length of 2nd train = 375 − 160
ATQ
= 215 m
3x − y = 5x − 4y

34 www.bankersadda.com | www.sscadda.com | www.careerpower.in | Adda247 App


The IBPS RRB PO & Clerk 2021 | SUCCESS GUIDE

IBPS RRB PO PRELIMS MEMORY BASED PAPER 2018


REASONING ABILITY

Direction (1-5): Study the following information carefully 6. If point X is 6m south of point A then which point is at
and answer the given questions: shortest distance from point X?
(a) E (b) A (c) F
Eleven boxes A, B, C, D, E, F, G, H, I, J, K are kept one above (d) B (e) G
the other. Box G is kept at fifth position from the top. Two
boxes are kept between G and H. Box D is kept just above 7. What is the distance of point C from point H?
box H. There are as many boxes above box D as below box (a) 9m (b) 5m (c) 4m
B. Five boxes are kept between box F and box K, which is (d) 6m (e) 7m
kept at one of the positions below box G. Box A is kept at 8. Point B is in which direction with respect to point F?
one of the positions above box F. Only one box is kept (a) South (b) South-east (c) North
between Box G and Box C. Box I is kept above box E but not (d) North-east (e) North-west
just above. Box E is not kept immediately above or Direction (9-13): Study the following information
immediately below box C. carefully and answer the given questions
1. What is the position of box I? Eight persons A, B, C, D, E, F, G, H are sitting around a
(a) 8th from the bottom circular table such that five of them are facing towards the
(b) 7th from the top center and the rest are facing away from the center. Three
persons are sitting between F and H, who is facing center.
(c) 3rd from the top
C is 2nd to the right of F and faces opposite direction to F.
(d) 6th from the bottom
A sits 3rd to the left of C.G is one of the neighbor of E. Two
(e) none of these
persons sit between G and B, who is not neighbor of H.G
2. How many boxes are kept between box E and Box H? does not face C. G and A face same direction but opposite
(a) seven (b)six (c) five to F.
(d) four (e) eight 9. What is the position of E with respect to A?
3. Which among the following statement is true (a) immediate right (b) 5th to the left
regarding box J? (c) 2nd to the right (d) 2nd to the left
(a) it is 7th from the bottom (e) none of these
(b) Box K is placed above box J 10. How many persons are sitting between C and H, wen
(c) only two boxes are kept between box B and box J counted from the left of C?
(d) It is kept just below box H (a) one (b) two (c) three
(e) All are true (d) four (e) none
11. Four of the five are alike in a certain way, which among
4. Which of the following represents the boxes kept the following does not belongs to that group?
between boxes A and I? (a) C (b) B (c) F
(a)C, B (b) A, K (c) F, G (d) D (e) E
(d) J, D (e) none of these
12. Which of the following represents the immediate
5. Which of the following box is kept just above box B? neighbor of G?
(a) C (b) K (c) F (a) C (b) B (c) F
(d) D (e) none of these (d) D (e) A
13. Which of the following is not true regarding F?
Direction (6-8): Study the following information carefully (a) it faces towards the center
and answer the given questions (b) E is immediate left to F
Point B is 14m east of point A. Point C is 9m north of point (c) Two persons sit between F and D, when counted
B. Point D is 12m east of point C. Point E is 15m south of from the right to D
point D. Point F is 30m west of point E. Point G is 10m north (d) All are true
of point F. Point H is 18 m east of point G. (e) no one sits between F and B

35 www.bankersadda.com | www.sscadda.com | www.careerpower.in | Adda247 App


The IBPS RRB PO & Clerk 2021 | SUCCESS GUIDE
Direction (14-18): Study the following information 100 minute and B. Only one movie released after B. B
carefully and answer the given questions. released immediately after 100-minute duration movie.
Movie C released immediately after the one which is of
Certain number of persons are sitting in a row facing north.
130-minute duration. More than two movies released in
M sits 4th to the right of S. Five persons sit between M and
between C and D. The movie which is of 90-minute
X. T sits at one of the positions left to S. The number of
duration released before E. One of movie was of 20 minutes
persons sitting between M and U are same as between S
more duration than E.
and T. Q is 2nd from one of the extreme ends. Four persons
sit between S and U. No one sits to the right of N, who is 21. How many movies were released after E?
immediate right to P. X is 3rd left to P. Not more than two (a) One (b) Two (c) None
persons sit between Q and U. (d) Three (e) More than three
14. How many persons are sitting in the row? 22. Which of the following movie was of 150-minute
(a) 17 (b) 20 (c) 24 duration?
(d) 26 (e) 27 (a) E
15. How many persons are sitting between S and T? (b) A
(a) seven (b)six (c) five (c) There is no such movie
(d) four (e) eight (d) C
(e) D
16. What is the position of U from the left end?
(a) 6th (b) 5th (c) 4th 23. What is the total duration of movie D and E together?
(d) 2nd (e) 3rd (a) 135 (b) 225 (c) 165
(d) 175 (e) 190
17. How many persons are sitting between Q and M? 24. Which of the following statement is true regarding B?
(a) seven (b) eleven (c) ten (a) The movie released after B is of 120-minute
(d) nine (e) eight duration
18. Which of the following represents the person sitting at (b) Two movies released in between A and B
extreme end? (c) Movie B is of 100-minute duration
(a) M (b) U (c) X (d) Total duration of movie B and A is 225 minutes
(d) P (e) T (e) Movie A released after B.
19. If the second, forth, seventh and eighth letter of the 25. Which of the following statement is true?
word “FRACTION” are combined to form a meaningful (a) The movie released before A is of 130-minute
word, then what will be the 3rd letter from the left in duration
the so formed word. If more than one meaningful word (b) Three movies released in between A and E
is formed then the answer is X, if no such word is (c) No movie released in between A and E
formed then answer is Z? (d) Total duration of movie C and A is 230 minutes
(a) O (b) X (c) R (e) Movie C released immediately after E.
(d) Z (e) C
Direction (26-28): Study the following information
20. How many pair of digits have same number of digits carefully and answer the given questions:
between them in the number “573814269” as in the F is the husband of G. K is the mother-in-law of G. H is the
numeric series? Father of F. M is the mother of H, P is the mother of K and
(a) five (b) four (c) six B.
(d) three (e) more than six
26. If Y is the father of H then how is Y related to M?
Direction (21-25): Study the following information (a) Mother (b) Father (c) Sister
carefully and answer the given questions: (d) Brother (e) Husband
Movies of different duration released on different days 27. How is P related to F?
starting from Monday to Friday (starting from Monday). (a) Grandfather (b) Aunt (c) Mother
Movie A was released On Tuesday. No movie released (d) Grandmother (e) Wife
between A and the one which is of 75-minute duration.
Only one movie is released between the one which is of 75- 28. How is B related to H?
minute duration and the one which is of 100-minute (a) Sister (b) Brother (c)Husband
duration. No movie released between the one which is of (d) Can’t be determine (e) Wife

36 www.bankersadda.com | www.sscadda.com | www.careerpower.in | Adda247 App


The IBPS RRB PO & Clerk 2021 | SUCCESS GUIDE
Direction (29-31): Study the following information (a) Neither I nor II follows
carefully and answer the given questions:There are six (b) Only I follows
persons M, N, O, P, Q, R of different heights. N is shorter (c) Either I or II follow
than M but taller than Q. Only two person are taller than M. (d) Both I and II follow
(e) Only II follows
R is taller than Q and O. Q is not the shortest. The one who
is second shortest is 154m. P is not the shortest person. 35. Statements: Some Door are Fan.
No Door is Rose.
29. If M is 19m taller than Q then what is the height of M? No Fan is Shelf.
(a) 190m (b) 181m (c) 175m Conclusions: I. All Door are Shelf is a possibility.
(d) 130m (e) 173m II. All Shelf can be Doors.
(a) Either I or II follows
30. If P is 181m than which of the following is true? (b) Only II follows
I. Only one person is taller than P. (c) Neither I nor II follow
II. The difference between the heights of P and Q is (d) Both I and II follow
27m (e) Only I follows
III. O is the shortest person.
Direction (36-40): Study the following information
(a) Only I (b) Only II and I carefully and answer the given questions:
(c) All are true (d) Only III and II Fourteen persons are sitting in two parallel rows such that
(e) Only III and I seven persons are sitting in each row. A, B, C, D, E, F, G are
31. How many persons are shorter than N? sitting in row-1 facing north while P, Q, R, S, T, U, V are
sitting in row-2 facing south. G sits third to the left of A and
(a) One (b) Two (c) None neither of them sits at an extreme end of the row. The one
(d) Three (e) More than three faces A sits immediate right to T. Only one person sits
Directions (32-35): Question consists of Some statements between T and Q. The one who faces Q sits third to the right
followed by two conclusions. Consider the given of E. S sits to the immediate left of V. S neither faces G nor
statements to be true even if they seem to be at variance E. D is an immediate neighbour of the one who faces S. The
one who faces C sits fifth to the left of P. B sits third to the
with commonly known facts. Read all the conclusions and left of F. U sits at one of position to the right of R.
then decide which of the given conclusions logically follow
from the given statements using all statements together. 36. Four of the following are alike in a certain way so form
a group which of the following does not belong to that
32. Statements: All Grills are Arrow. group?
Some Hat are Grills. (a) U (b) B (c) T
Some Cell are Arrow. (d) C (e) P
Conclusions: I. Some Cell are definitely not 37. How many persons sits between F and C?
Grills. (a) One (b) Two (c) None
II. Some Hat can never be Arrow. (d) Three (e) More than three
(a) Only I follows 38. Which of the following is not true regarding U?
(b) Only II follows (a) No one sits to the right of U
(c) Neither I nor II follow (b) U sits third to the right of Q,
(d) Both I and II follow (c) P is an immediate neighbour of U.
(e) Either I or II follow (d) E is an immediate neighbour of the one who faces
U,
33. Statements: All Grills are Arrow. (e) Only two persons sit between U and S
Some Hat are Grills.
Some Cell are Arrow.
Conclusions: I. Some Hat are Arrow.
II. Some Grills are Cell.
(a) Only II follows
(b) Only I follows
(c) Either I nor II follow
(d) Both I and II follow
(e) Neither I or II follow
34. Statements: Some Door are Fan.
No Door is Rose.
No Fan is Shelf.
Conclusions: I. Some Fan can never be Rose.
II. Some Rose are Shelf is a
possibility.
37 www.bankersadda.com | www.sscadda.com | www.careerpower.in | Adda247 App
The IBPS RRB PO & Clerk 2021 | SUCCESS GUIDE
39. What is the position of C with respect to A? 40. What is the position of B with respect to D?
(a) Second to the left (a) Third to the left
(b) Third to the right (b) Second to the left
(c) Immediate right (c) Forth to the left
(d) Immediate left (d) Third to the right
(e) Second to the right (e) Fifth to the right

QUANTITATIVE APTITUDE
Directions (41-45): Find the wrong number in the 43. 250, 260, 291, 314, 340, 370, 405
following number series ? (a) 370 (b) 314 (c) 260
(d) 405 (e) 250
41. 1, 3, 7, 15, 31, 64, 127
44. 750, 535, 411, 348, 322, 314, 315
(a) 1 (b) 3 (c) 15
(a) 315 (b) 750 (c) 411
(d) 64 (e) 127 (d) 348 (e) 314
42. 1, 15, 119, 475, 949, 947, 473 45. 2, 7, 27, 107, 427, 1708, 6827
(a) 947 (b) 475 (c) 15 (a) 107 (b) 1708 (c) 2
(d) 473 (e) 1 (d) 6827 (e) 7

Directions (46-50): Study the line-graph carefully & answer the question given below.
Line-graph given below shows the total no. of products for (kid + adult) in two different stores P & Q in five different years.
Store P Store Q
80

70
No. of product

60

50

40

30

20
2000 2001 2002 2003 2004

46. What is the difference between total no. of products in 49. What is the average no. of products in all the years
store P in year 2003 & 2004 together and total no. of together in store P?
products in year 2000? (a) 48 (b) 43 (c) 57
(a) None of these (b) 10 (c) 20 (d) None of these (e) 53
(d) 15 (e) 5
50. Total no. of products in store P in year 2003 and in
47. If total products in both the stores in year 2006 is
store Q in year 2004 together is what percent
increased by 20% as compared to year 2004. Then find
more/less than total no. of products in store Q in year
total no. of products in year 2006?
(a) 102 (b) None of these 2000?
(c) 96 (d) 108 (a) 150% (b) 40% (c) 125%
(e) 92 (d) 100% (e) 50%

48. What is the ratio of total products in store Q in year Directions (51-55): Solve the given quadratic equations
2002 & 2003 together to total products in store Q in and mark the correct option based on your answer—
year 2000? (a) x ≥ y (b) x ≤ y (c) x > y
(a) 23 : 12 (b) 23 : 11 (c) 28 : 11 (d) x = y or no relation can be established between x and y.
(d) None of these (e) 27 : 13 (e) x < y
38 www.bankersadda.com | www.sscadda.com | www.careerpower.in | Adda247 App
The IBPS RRB PO & Clerk 2021 | SUCCESS GUIDE
51. (i) 𝑥 2 − 20𝑥 + 96 = 0 (ii) 𝑦 2 = 64 number of students in the college. Number of students
appeared in ‘Y’ exam only is same as number of students
52. (i) 4𝑥 2 − 21𝑥 + 20 = 0 (ii) 3y2 − 19y + 30 = 0
appeared in ‘Y’ and ‘Z’ only.
53. (i) x2 − 11x + 24 = 0 (ii) y2 − 12y + 27 = 0
56. How many students appeared in at least two exams?
54. (i) x2 + 12x + 35 =0 (ii) 5y2 + 33y + 40 =0 (a) 240 (b) 260 (c) 300
55. (i) 4x2 + 9x + 5 =0 (ii) 3y2 + 5y + 2 =0 (d) 360 (e) 500

Directions (56-60): Study the following paragraph 57. How many students appeared in two exams only?
carefully & answer the question given below. (a) 280 (b) 220 (c) 340
(d) 300 (e) 260
There are 1000 students in a college. Out of 1000 students
some appeared in exams ‘X’, ‘Y’ and ‘Z’ while some not. 58. How many students appeared in at most two exams?
Number of student not appeared in any exam is equal to (a) 240 (b) 260 (c) 300
number of students appeared in exam ‘Z’ only. Number of (d) 500 (e) 960
students appeared in exam ‘Y’ is 360. Ratio of number of
students appeared in exam ‘X’ and ‘Y’ only to number of 59. How many students not appeared in exam Y?
students appeared in exam ‘Y’ and ‘Z’ only is 2 : 3. Number (a) 440 (b) 360 (c) 540
of student appeared in exam ‘X’ and ‘Z’ both is half of (d) 640 (e) None of these
number of students appeared in only exam ‘Z’. Number of
students appeared in exam ‘X’ only is 50% more than 60. How many students appeared in exam X or in exam Z?
number of students appeared in ‘Y’ only. Number of (a) 240 (b) 360 (c) 500
students appeared in all the three exam is 4% of the total (d) 680 (e)760
Direction (61-65): Bar chart given below shows Number of tigers in different National Parks i.e. A to D of a country in
two different years. Study the data carefully and answer the following questions

1998 2018
100

80
Number of tigers →

60

40

20

0
A B C D
National Parks→
61. Number of tigers in National Park B and C together in 63. Find the ratio between number of tigers in National
2018 is how much less more/less than Number of Park ‘A’ in 2018 to number of tigers in National Park ‘B’
in 1998?
tigers in National Park A and D together in 1998?
(a) 9 : 10 (b) 10 : 9 (c) 16 : 13
(a) 40 (b) 44 (c) 52 (d) 13 : 16 (e) 3 : 4
(d) 60 (e) 72 64. Number of tigers in National Park ‘E’ in 2018 is 40%
more than number of tigers in National Park ‘D’ in 1998
62. Number of tigers in National Park ‘D’ in both years
while number of tigers in National park ‘E’ in 1998 is
together is what percent of the Number of tigers in 25% less than number of tigers in National Park ‘C’ in
National Park ‘C’ in both years together? 2018. Find total number of tigers in National park ‘E’ in
1998 and 2018 together?
(a) 60% (b) 160% (c) 140%
(a) 148 (b) 84 (c) 172
(d) 120% (e) 180% (d) 160 (e) 136
39 www.bankersadda.com | www.sscadda.com | www.careerpower.in | Adda247 App
The IBPS RRB PO & Clerk 2021 | SUCCESS GUIDE
65. Average number of tigers in all National park in 2018 (a) Rs. 40,000 (b) Rs. 35,000
is how much less/more than average number of tigers (c) Rs. 32,000 (d) Rs. 30,000
in all National park in 1998? (e) Rs. 25,000
(a) 14 (b) 16 (c) 18
(d) 20 (e) 22 73. The sum of four times of an amount ‘x’ and (x – 9.75) is
Rs. 442. Find the approximate value of x.
66. The difference between downstream speed and (a) Rs. 85 (b) Rs. 90 (c) Rs. 100
upstream speed of boat is 6 km/hr and boat travels 72 (d) Rs. 1100 (e) Rs. 75
km from P to Q (downstream) in 4 hours. Then find the
speed of boat in still water? 74. A and B entered into a partnership by investing some
(a) 15 km/hr (b) 18 km/hr (c)20km/hr amounts. The investment of A is twice of the
(d) 16 km/hr (e) 24 km/hr investment of B. Another person C joined them after 4
67. In a vessel, there are two types of liquids A and B in the months. At the end of a year, the profit share of A and
ratio of 5 : 9. 28 lit of the mixture is taken out and 2 lit C is equal. Then find the profit share of B is what
of type B liquid is poured into it, the new ratio(A:B) percent of the profit share of C.
1
thus formed is 1 : 2. Find the initial quantity of mixture (a) 50% (b) 33 % (c) 40%
3
in the vessel? (d) 60% (e) 75%
(a) 84 lit (b) 42 lit (c) 50 lit
(d) 56 lit (e) 70 lit 75. The ratio of age of Ishu 8 years hence and that of Ahana
6 years hence is 5 : 6. The age of Ishu 10 years hence is
68. The average weight of 5 students in a class is 25.8 kg. equal to the age of Ahana 6 years hence. Then, find the
When a new student joined them, the average weight
present age of Ishu.
is increased by 3.9 kg. Then find the approximate
(a) 1.5 yr (b) 2 yr (c) 3 yr
weight of the new student.
(a) 55 kg (b) 49 kg (c) 42 kg (d) 4 yr (e) 5 yr
(d) 44 kg (e) 58 kg 76. What is the difference between 20% of P and 20% of
69. A person has purchased two adjacent plots, one is in (P + 5000).
rectangular shape and other is in square shape and (a) 1500 (b) 1200 (c) 1000
combined them to make a single new plot. The breadth (d) 2000 (e) 1600
of the rectangular plot is equal to the side of the square 77. The ratio of the diameter of base and height of a
plot and the cost of fencing the new plot is Rs. 390 (Rs. cylinder is 2 : 3. Find the radius of the cylinder if the
5/m). Find the side of square if the length of the
approximate volume of cylinder is 3234.01 cm³?
rectangular plot is 15 m. 21 7
(a) 10 m (b) 8 m (c) 12 m (a) cm (b) cm (c) 21 cm
2 2
(d) 9 m (e) 6 m (d) 7 cm (e) 14 cm
70. A shopkeeper marked his article 50% above the cost 78. A train of some length passes the platform of length
price and gives a discount of 20% on it. If he had 524 m in 55 seconds. Find the length of train if the
marked his article 75% above the cost price and gives speed of train is 72 km/hr.
a discount of 20% on it then find the earlier profit is (a) 476 m (b) None of these
what percent of the profit earned latter? (c) 428 m (d) 526 m
1
(a) 50% (b) 60% (c) 333% (e) 576 m
(d) 40% (e) 75%
71. A person invested two equal amounts in two different
schemes. In first scheme, amount is invested at 8% p.a.
on SI for T years and SI received is Rs 2000 while in
second scheme, amount is invested at 10% p.a. for 2
years at CI and the compound interest received is Rs.
1050. Find the value of T.
(a) 4 yr (b) 8 yr (c) 6 yr
(d) 5 yr (e) 3 yr
72. Satish saves 20% of his monthly salary. And of the
1 1
remaining salary 4th and 2th he gives to his mother and
sister respectively and the remaining salary he submits
as his EMI for the payment of his car. If his annual EMI
was Rs. 60,000, then find his monthly salary?
40 www.bankersadda.com | www.sscadda.com | www.careerpower.in | Adda247 App
The IBPS RRB PO & Clerk 2021 | SUCCESS GUIDE
79. Efficiency of B is two times more than efficiency of A. 80. 7 men and 6 women together can complete a piece of
Both started working alternatively, starting with B and work in 8 days and work done by a women in one day
completed the work in total 37 days. If C alone is half the work done by a man in one day. If 8 men and
complete the same work in 50 days then find in how 4 women started working and after 3 days 4 men left
many days A and C together will complete the work? the work and 4 new women joined then, in how many
more days will the work be completed
(a) 24 days (b) 30 days (c) 36 days (a) 7 days (b) 6 days (c) 5.25 days
(d) 48 days (e) 18 days (d) 6.25 days (e) 8.14 days

Solutions

REASONING ABILITY
Direction (1-5): Direction (14-18):
BOX
D
H
A 14. (c); 15. (e); 16. (b);
F 17. (b); 18. (e);
G
I 19. (c); 2nd, 4th,7th and 8th letters are R, C, O, N
C The meaningful word formed is CORN
J
E 20. (e);
K
B
1. (d); 2. (b); 3. (c);
4. (c); 5. (b); Direction (21-25):
Direction (6-8): Days Movies Duration
Monday D 75
Tuesday A 90
Wednesday E 100
Thursday B 130
Friday C 120

21. (b); 22. (c); 23. (d);


24. (a); 25. (c);

6. (c); 7. (b); 8. (d); Direction (26-28):

Direction (9-13):

26. (e); 27. (d); 28. (d);

9. (d); 10. (a); 11. (a); Direction (29-31): R/P > R/P > M > N > Q > O
12. (e); 13. (c); 29. (e); 30. (d); 31. (b);

41 www.bankersadda.com | www.sscadda.com | www.careerpower.in | Adda247 App


The IBPS RRB PO & Clerk 2021 | SUCCESS GUIDE
Directions (32-35): 34. (d);
32. (c);

35. (b);

33. (b);

Direction (36-40):

36. (e); 37. (b); 38. (b);


39. (c); 40. (c);

QUANTITATIVE APTITUDE

41. (d); 48. (c); Required ratio =


80+60
=
140
= 28 : 11
55 55

49. (b); Required Average


25+40+65+55+30 215
= 5
= 5 = 43
42. (a);
50. (d); Required percentage
(55 + 55) − 55
= × 100
55
55
43. (c); = × 100 = 100%
55

51. (a); (i) 𝑥 2 − 20𝑥 + 96 = 0


𝑥 2 − 12𝑥 − 8𝑥 + 96 = 0
𝑥(𝑥 − 12) − 8(𝑥 − 12) = 0
44. (e); (𝑥 − 12)(𝑥 − 8) = 0
𝑥 = 12,8
(ii) 𝑦 2 = 64
𝑦 = ±8
∴ 𝑥≥𝑦
45. (b); 52. (d); (i) 4𝑥 2 − 21𝑥 + 20 = 0
4𝑥 2 − 16𝑥 − 5𝑥 + 20 = 0
4𝑥(𝑥 − 4) − 5(𝑥 − 4) = 0
Alternate, (4𝑥 − 5)(𝑥 − 4) = 0
5
𝑥 = 4,4
(ii) 3𝑦 2 − 19𝑦 + 30 = 0
3𝑦 2 – 9𝑦 − 10𝑦 + 30 = 0
3𝑦(𝑦 − 3) − 10(𝑦 − 3) = 0
46. (e); Required difference (3𝑦 − 10)(𝑦 − 3) = 0
= (55 + 30) – (55 + 25) = 5 10
𝑦 = 3 ,3
47. (a); Total no. of products in year 2006 ∴ No relation can be established between
120
= (55 + 30) × 100 = 102.0 x and y
42 www.bankersadda.com | www.sscadda.com | www.careerpower.in | Adda247 App
The IBPS RRB PO & Clerk 2021 | SUCCESS GUIDE
53. (d); (i) 𝑥 2 − 11𝑥 + 24 = 0
𝑥 2 − 8𝑥 − 3𝑥 + 24 = 0
𝑥(𝑥 − 8) − 3(𝑥 − 8) = 0
(𝑥 − 3)(𝑥 − 8) = 0
𝑥 = 3 ,8
(ii) 𝑦 2 − 12𝑦 + 27 = 0
𝑦 2 – 9𝑦 − 3𝑦 + 27 = 0
𝑦(𝑦 − 9) − 3(𝑦 − 9) = 0
(𝑦 − 9)(𝑦 − 3) = 0
𝑦 = 9,3
∴ No relation can be established between
Now, 2x + 3x + 3x + 40 = 360
x and y
⇒ x = 40
𝑎
54. (b); (i) 𝑥 2 + 12𝑥 + 35 = 0 𝑎𝑛𝑑, 12.5𝑥 + 𝑎 + 2 + 𝑎 = 1000
𝑥 2 + 7𝑥 + 5𝑥 + 35 = 0 5𝑎
= 500
𝑥(𝑥 + 7) + 5(𝑥 + 7) = 0 2

(𝑥 + 7)(𝑥 + 5) = 0 ⇒ a = 200
𝑥 = −7 , −5
(ii) 5𝑦 2 + 33𝑦 + 40 = 0
5𝑦 2 + 25𝑦 + 8𝑦 + 40 = 0
5𝑦(𝑦 + 5) + 8(𝑦 + 5) = 0
(𝑦 + 5)(5𝑦 + 8) = 0
8
𝑦 = − 5 , −5
∴ 𝑦≥𝑥
55. (b); (i) 4𝑥 2 + 9𝑥 + 5 = 0
4𝑥 2 + 4𝑥 + 5𝑥 + 5 = 0
4𝑥(𝑥 + 1) + 5(𝑥 + 1) = 0
(4𝑥 + 5)(𝑥 + 1) = 0 56. (c); Students appeared in atleast two exams
5 = 80 + 60 + 40 + 120 = 300
𝑥 = −1 , −
4
57. (e); Students appeared in two exams only = 80 + 60
(ii) 3𝑦 2 + 5𝑦 + 2 = 0
+ 120 = 260
3𝑦 2 + 3𝑦 + 2𝑦 + 2 = 0
3𝑦(𝑦 + 1) + 2(𝑦 + 1) = 0 58. (e); Students appeared in atmost two exams
(3𝑦 + 2)(𝑦 + 1) = 0 = 180 + 120 + 200 + 60 + 80 + 120 + 200= 960
2
𝑦 = − , −1
3 59. (d); Student not appeared in exam Y
∴ 𝑦≥𝑥 = 1000 – 360 = 640

Solutions (56-60): 60. (d); Students appeared in exam X or in exam Z


Total students = 1000 = 180 + 60 + 40 + 80 + 200 + 120 = 680
Let, students appear in exam Z only = a 61. (d); Number of tigers in National Park B and C
Total students appeared in exam Y = 360 together in 2018 = 52 + 32 = 84
Ratio of number of students appeared in exam X and Y Number of tigers in National Park A and D
only to students appeared in exam Y and Z only together in 1998
=2:3 = 64+80 = 144
Students appeared in exam X and Z both Required difference = 144-84 = 60
= a/2
62. (b); Number of tigers in National Park D in 1998 and
Number of students appeared in all three exams
4
2018 together = 80 + 48 = 128
= 100 × 1000 = 40 Number of tigers in National Park C in 1998 and
Number of students appeared in Y exam only 2018 together = 48 + 32 = 80
128
= No. of students appeared in Y and Z only = 3x Required % = 80 × 100 = 160%
Number of students appeared in exam X and Y only
36 9
2
= × 3𝑥 = 2𝑥 63. (a); Required Ratio = =
3 40 10

43 www.bankersadda.com | www.sscadda.com | www.careerpower.in | Adda247 App


The IBPS RRB PO & Clerk 2021 | SUCCESS GUIDE
64. (e); Number of tigers in National Park E in 2018 = New MP = Rs. 175x
140 80
100
× 80 = 112 New SP = 175x × = Rs. 140x
100
Number of tigers in National Park E in 1998 = New Profit = Rs. 40x
75 20𝑥
100
× 32 = 24 Required % = 40𝑥 × 100 = 50%
Number of tigers in National Park E in 1998 and
71. (d); Let the amount be Rs. x
2018 together 10 × 10
= 112 + 24 = 136 CI at 10% in 2 years = 10 + 10 + 100
= 21%.
65. (b); Total number of tigers in 2018
ATQ,
= 36 + 52 + 32 + 48 = 168 𝑥 × 21
Total number of tigers in 1998 100
= 1050 ⇒ x = Rs. 5000
= 64 + 40 + 48 + 80 = 232 And,
232 168 5000 × 8 × 𝑇
Required difference = − = 2000
4 4
64 100
= = 16 ⇒ T = 5 years.
4

66. (a); Let the speed of boat in still water be x km/hr 72. (e); Let the monthly salary be Rs. 100 x.
and that of stream be y km/hr EMI per month
ATQ, 1 1
= 100x – (20𝑥 + 80𝑥 × 4 + 80𝑥 × 2) = Rs. 20x
(x + y) – (x – y) = 6
⇒ 2y = 6 ⇒ y = 3 km/hr ATQ,
72 20x × 12 = 60,000
Downstream stream = (x + y) = 4 = 18 km/hr
⇒ x = 250
⇒ x = 15 km/hr Monthly Salary = Rs. 25,000
67. (d); Let the initial quantity of mixture in vessel be x 73. (b); ATQ,
lit 4x + x – 9.75 = 442
ATQ, 5x = 451.75
5
𝑥× −10
9
14
=2
1 x = Rs. 90
𝑥× −18+2
14
5𝑥−140 1 74. (a); Let the investment of B be Rs. x
⇒ =
9𝑥−224 2 ∴ investment of A = Rs 2x
⇒ 10x – 280 = 9x – 224 Ratio of profit,
⇒ x = 56 lit 𝐴 : 𝐵 : 𝐶
68. (b); Weight of new student = 6 × (25.8 + 3.9) – 5 × 12 × 2𝑥 : 12 × 𝑥 : 8 × 𝑦
25.8 ATQ,
≈ 49 kg 24x = 8y
69. (c); y = 3x
12 × 𝑥
∴ Required percentage = 8 × 3𝑥 × 100
= 50%
75. (b); Let present age of Ishu & Ahana be x year & y
year respectively
∴ ATQ,
𝑥+8 5
Let the breadth of rectangular plot be y m and =
length = 15 m 𝑦+6 6
ATQ, 6x + 48 = 5y + 30
30 + y + 3y = 390/5 6x – 5y = – 18 … (i)
⇒ 30 + 4y = 78 x + 10 = y + 6
⇒ 4y = 48 ⇒ y = 12 m x–y=–4 … (ii)
∴ x = 2 years
70. (a); Let the CP be Rs. 100x ∴ present age of Ishu is 2 years.
Then, MP = Rs. 150x
20 20
80
SP = 150x × 100 = Rs. 120x 76. (c); quired difference = 100 (𝑃 + 5000) − 100 × 𝑃
Profits = Rs. 20x = 1000

44 www.bankersadda.com | www.sscadda.com | www.careerpower.in | Adda247 App


The IBPS RRB PO & Clerk 2021 | SUCCESS GUIDE
77. (d); Let diameter of base be 2x cm & height of 79. (b); Lets efficiency of A is x unit/day and B’s
cylinder be 3x cm efficiency is 3x unit/day
2𝑥
∴ radius = 2 = 𝑥 cm So, B work for 19 days and A work for 18 days
ATQ—
We know, Total work = 19 × 3x + 18 × x = 75x
Volume of cylinder 75𝑥
𝐸𝑓𝑓𝑖𝑐𝑖𝑒𝑛𝑐𝑦 𝑜𝑓 𝐶 = 50
= 𝜋𝑟 2 ℎ (r→ radius, h → height)
= 1.5𝑥 𝑢𝑛𝑖𝑡/𝑑𝑎𝑦
ATQ, 75𝑥
𝜋𝑟 2 ℎ = 3234 (𝐴 + 𝐶) 𝑡𝑜𝑔𝑒𝑡ℎ𝑒𝑟 =
(𝑥+1.5𝑥)
22 = 30 𝑑𝑎𝑦𝑠
× 𝑥 2 × 3𝑥 = 3234
7 80. (d); One day work of women = half of work done by
x = 7cm
men in one day
Radius = 7cm
Let efficiency of one women = w unit/day
78. (e); Speed of train in m/s. Man’s efficiency = 2w unit/day
5
= 72 × 18 = 20 m/s Total work = (7 × 2w + 6 × w) × 8 =160w unit
Let length of train be x m 8 men and 4 women start work for 3 days
ATQ, Total work done = (8 × 2w + 4 × w) × 3 = 60w
524 + 𝑥 4 women replace 4 man
= = 20 = (4 × 2w + 8 × w) =16w
55 100𝑤
x = 1100 – 524 = 576m Days required = = 6.25 days
16𝑤

45 www.bankersadda.com | www.sscadda.com | www.careerpower.in | Adda247 App


The IBPS RRB PO & Clerk 2021 | SUCCESS GUIDE

IBPS RRB PO PRELIMS MEMORY BASED PAPER 2017


REASONING ABILITY

1. What should come in place of question mark (?) in the 6. How many boxes are there above box D?
following series based on the above arrangement? (a) 4 (b) 3 (c) 6
ZN XD UG QK ? (d) 2 (e) None of these
(a) LK (b) LO 7. Which of the following boxes is kept at the top?
(c) LP (d) KP (a) B (b) A (c) D
(e) Other than the given options (d) E (e) None of these
8. Choose the odd one out?
2. How many such pair of numbers are there in the given
(a) B (b) G (c) A
number “46579739” (Both backward and forward)
(d) D (e) E
same as far as according to numeric series?
(a) One (b) Two 9. Which of the following boxes is kept between F and A?
(c) Three (d) More than three (a) B (b) G
(e) None of these. (c) C (d) H
(e) None as box F is immediately above box A
3. If it is possible to make only one meaningful word with
10. How many boxes are there between C and A
the 1st ,2nd ,4th and 7th letters of the word ‘ECUADOR’
(a) Less than 2 (b) 4 (c) 5
which would be the second letter of the word from the
(d) 6 (e) None of these
right? If more than one such word can be formed give
‘Y’ as the answer. If no such word can be formed, give Directions (11-15): In these questions, relationships
‘Z’ as your answer. between different elements are shown in the statements.
(a) Y (b) E (c) I These statements are followed by two conclusions. Give
(d) Z (e) M answer
(a) if only conclusion I follows
4. If 1 is subtracted from each odd number and 2 is added (b) if only conclusion II follows
to each even in the number 9436527, then how many (c) if either conclusion I or conclusion II follows
digits will appear twice in the new number thus (d) if neither conclusion I nor conclusion II follows
formed? (e) if both conclusions I and II follow
(a) Only 8 (b) Only 8 and 6 (c) 8, 6 and 4
11. Statement: R ≥ S ≥ T > U > X; T < V < W
(d) 2, 4 and 6 (e) None of these
Conclusions: I. R > X II. X < W
5. How many letter will be remain the same position in
12. Statement: E = F < G < H; G ≥ I
the word ‘MONSTER’ when they arranged in the
Conclusions: I. H > I II. E > I
ascending order from left to right?
(a) One (b) Two (c) Three 13. Statement: A > B > F > C; D > E > C
(d) More than Three (e) None Conclusions: I. C < A II. B > D

Directions (6-10): Read the following information 14. Statement: K ≤ L ≤ M = N; P ≥ O ≥ N


carefully and answer the following questions. Conclusions: I. K < P II. K = P
Eight boxes A, B, C, D, E, F, G and H are place one above the 15. Statement: D < E < F < G; K > F
other in any particular order. Box no. 1 is at the bottom and Conclusions: I. K ≤ G II. K > D
box no. 8 is at the top. Three boxes are placed between A
Directions (16-20): Read the following information
and B. Box H is placed immediately below A. There are two
carefully and answer the following questions.
boxes between H and G. There are as many boxes between
Seven persons A, B, C, D, E, F and G were born on different
C and D as between H and B. Box C is kept above D. Box E months viz. January, February, March, April, June, August
is kept immediately below box D. Three boxes are there and October of the same year, but not necessarily in the
between E and F. same order.
46 www.bankersadda.com | www.sscadda.com | www.careerpower.in | Adda247 App
The IBPS RRB PO & Clerk 2021 | SUCCESS GUIDE
Only three persons were born before E and D is not one of Directions (26-30): Study the following information to
them. F was not born immediately after E. B was born after answer the given questions
F. A was born immediately before the month in which G Twelve people are sitting in a two parallel rows containing
was born. Only two persons were born between G and F. six people each in such a way that there is an equal
16. How many persons were born between C and E? distance between adjacent persons. In row 1 – A, B, P, Q, X
(a) Three (b) Two (c) Four and Y are seated (but not necessarily in the same order)
(d) Five (e) None of these and all of them are facing south. In row 2 – E ,F ,R ,Z ,S and
U are seated (but not necessarily in the same order) and all
17. Who amongst the following is the oldest?
of them are facing North. Therefore in the given seating
(a) A (b) C (c) E
arrangement each member seated in a row faces another
(d) B (e) F
member of the other row. Q sits fourth to the left of A. The
18. Who amongst the following was born between the one facing A sits third to the left of S. Only one person sits
months in which A and D were born? between S and E. E does not sit at any of the extreme ends
(a) E (b) G (c) C of the line The one facing U sits second to the right of B. U
(d) B (e) Both E and G does not sit at any of the extreme ends of the line. Only two
people sit between B and Y. The one facing B sits second to
19. How many persons were born after D?
the left of Z. F is not an immediate neighbour of U. P is not
(a) One (b) Three (c) Four
immediate neighbour of Q.
(d) Two (e) None of these
26. Which of the following groups of people represents the
20. Who amongst the following is the person who was
people sitting at extreme ends of both the rows?
born in the month which has less than 30 days?
(a) Q, Y, Z, R (b) F, Y, F, B (c) S, Y, Z, R
(a) F (b) B (c) G
(d) Q, F, Z, B (e) Q, Y, Z, S
(d) C (e) A
27. Who amongst the following faces, F?
Directions (21-25): Study the following information
(a) Q (b) P (c) A
carefully and answer the given questions:
(d) X (e) B
In a certain code language
‘card win team time’ is written as ‘la ta ja sa’ 28. Which of the following is true with respect to the given
‘fight game play card’ is written as ‘ja pa ra da’ information?
‘in win team fight’ is written as ‘da ta fa la’. (a) B faces one of the immediate neighbours of Z.
(b) F sits exactly between R and E.
21. What is the code for ‘time’?
(c) None of the given options is true
(a) sa (b) da (c) ja
(d) A is an immediate neighbour of B
(d) la (e) None of these
(e) A faces U.
22. ‘card fight in’ can be coded as?
29. Which of the following is true regarding X?
(a) sa ja ra (b) fa ja da
(a) B sits second to the right of X.
(c) da ra ta (d) Can’t be determined
(b) F is an immediate neighbor of the person who faces
(e) None of these
X
23. What is the code for ‘game’? (c) Both P and Y are immediate neighbours of X
(a) ra (d) Only one person sits between X and A
(b) pa (e) None of the given options is true
(c) Either ra or pa 30. Who amongst the following sits second to the right of
(d) da the person who faces P?
(e) None of these (a) F (b) U (c) R
24. Which of the following is the code for ‘in’? (d) E (e) S
(a) ta (b) da (c) la
Directions (31-35): Study the following information
(d) fa (e) None of these
carefully and answer the questions given below:
25. If ‘game in risk’ is coded as ‘Pa fa xa’ than what will be
Eight friends M, N, O, P, Q, R, S and T are sitting around a
the code for ‘risk card fight’? circular table with equal distance between them but not
(a) Ja sa da (b) ja da ra (c) sa da fa necessarily in the same order. Some of them are facing the
(d) xa ja da (e) None of these centre with some face outside (i.e. opposite to centre).
47 www.bankersadda.com | www.sscadda.com | www.careerpower.in | Adda247 App
The IBPS RRB PO & Clerk 2021 | SUCCESS GUIDE
O sits second to the right of R, R faces the centre. Only two even if they seem to be at variance with commonly known
people sit between O and N (either form O’s right or O’s facts. Read all the conclusions and then decide which of the
left). S sits second to the right of O. T sits to the immediate given conclusions logically follows from the given
right of N. S and N face opposite direction (i.e. if N faces the statements, disregarding commonly known facts.
centre then S faces outside and vice versa). Immediate Give answer
neighbor of S face the same direction (i.e. If one neighbor (a) If only conclusion I follows.
faces the centre then the other also faces the centre and (b) If only conclusion II follows.
vice-versa) Only three people sit between P and Q. Neither (c) If either conclusion I or II follows.
P nor M is an immediate neighbor of R. Q sits second to the (d) If neither conclusion I nor II follows.
right of M. Both T and Q face a direction opposite to that of (e) If both conclusions I and II follow.
O (i.e. if O faces the centre then both T and Q faces outside
and vice-versa). 36. Statements: All bags are purses.
31. Who sits exactly between M and P? No purse is black.
(a) N (b) S (c) R All blacks are covers.
(d) Q (e) None of these Conclusions: I. All bags are covers
II. Some covers are purses.
32. How many people in the given arrangement face the
centre? 37. Statements: Some cats are rats.
(a) One (b) Three (c) Five Some rats are fishes.
(d) Four (e) None of these All fishes are birds.
Conclusions: I. Some fishes are rats.
33. Who sits second to the right of T?
II. All cats being birds is a possibility
(a) O (b) Q
(c) S (d) R 38. Statements: Some flowers are roses.
(e) Other than the given options No rose is red.
All red are leaves.
34. Four of the following five are alike in a certain way
Conclusions: I. Some flowers are definitely not red.
based on the given seating arrangement and so form a
II. Some leaves are definitely not roses.
group. Which is the one that does not belong to that
group? 39. Statements: All cards are sheets.
(a) P (b) O (c) T All files are cards.
(d) M (e) Q Some sheets are papers.
35. What is P’s position with respect to R? Conclusions: I. All files being papers is a possibility.
(a) Second to the left (b) Third to the right II. All files are not sheets.
(c) Third to the left (d) Sixth to the right 40. Statements: Some flowers are roses.
(e) Second to the right No rose is red.
Directions (36–40): In each question below are given All red are leaves.
some statements followed by two conclusions numbered I Conclusions: I. Some flowers are not leaves.
and II. You have to take the given statements to be true II. No leave is a red.

QUANTITATIVE APTITUDE

Directions (41-45): What should come in place of the 43. 7, 4, 5, 12, 52, ?
question mark (?) in following number series problems? (a) 424 (b) 428 (c) 318
(d) 440 (e) None of these
41. 190, 94, 46, 22, ? , 4
(a) 12 (b) 14 (c) 10 44. 6, 4, 5, 11, 39, ?
(a)159 (b) 169 (c) 189
(d) 8 (e) None of these
(d)198 (e) None of these
42. 5, 28, 47, 64, 77, ?
45. 89, 88, 85, 78, 63, ?
(a) 84 (b) 86 (c) 89 (a) 30 (b) 34 (c) 36
(d) 88 (e) None of these (d) 32 (e) None of these
48 www.bankersadda.com | www.sscadda.com | www.careerpower.in | Adda247 App
The IBPS RRB PO & Clerk 2021 | SUCCESS GUIDE
46. There are 3 consecutive odd numbers and 3 52. Find the central angle for the book D.
consecutive even numbers. The smallest even number (a) 117.5° (b) 115.2° (c) 112.8°
is 9 more than largest odd number. If the square of (d) 108.5° (e) 118.8°
average of all the 3 given odd number is 507 less than
the square of the average of all the 3 given even 53. If total books of another publisher ‘MNP’ is 20% more
number, what is the smallest odd number. than books of ‘XYZ’ publisher then what will be total
(a) 11 (b) 13 (c) 17 books sold by store A and B for publisher ‘MNP’.
(d) 19 (e) 9
Percentage-distribution for different stores for MNP
47. A can complete a task in 15 days B is 50% more remains same as for ‘XYZ’
efficient than A. Both A and B started working together (a) 200 (b) 178 (c) 181
on the task and after few days B left task and A finished (d) 186 (e) 198
1
the remaining 3 of the given work. For how many days
A and B worked together. 54. What is the ratio of total books sold by store A and C
(a) 3 (b) 5 (c) 4 together to the total books sold by store D and E
(d) 6 (e) 2 together
48. A boat can travel 9.6 km downstream in 36 min. If (a) 17 : 27 (b) 18 : 29 (c) 21 : 28
speed of the water current is 10% of the speed of the (d) 22 : 23 (e) 24 : 29
boat in downstream. How much time will boat take to
travel 19.2 km upstream. 55. What is the difference between average of book sold by
(a) 2 hours (b) 3 hours (c) 1.25 hours store A and E together and average books sold by store
(d) 1.5 hours (e) 1 hour C and D together?
49. A started a business with a initial investment of Rs. (a) 33 (b) 11 (c) 22
1200. ‘X’ month after the start of business, B joined A (d) 44 (e) 20
with on initial investment of Rs. 1500. If total profit
was 1950 at the end of year and B’s share of profit was Directions (56-60): In each of these questions, two
750. Find ‘X’ equations (I) and (II) are given. You have to solve both the
(a) 5 month (b) 6 month (c) 7 month equations and give answer
(d) 8 month (e) 9 month (a) if x>y (b) if x≥y
50. Ratio between curved surface area and total surface (c) if x<y (d) if x ≤y
area of a circular cylinder is 3 : 5. If curved surface area (e) if x = y or no relationship can be established.
is 1848 cm3 then what is the height of cylinder.
(a) 28 (b) 14 (c) 17 56. I. 𝑥 2 + 9𝑥 + 20 = 0 II. 𝑦 2 = 16
(d) 21 (e) 7
57. I. 𝑥 2 − 7𝑥 + 12 = 0 II. 3𝑦 2 − 11𝑦 + 10=0
Directions (51-55): Given below is the pie chart which
shows the percentage distribution of a book ‘XYZ’ 58. I. 𝑥 2 − 8𝑥 + 15 = 0 II. 𝑦 2 − 12𝑦 + 36 = 0
publishes in 5 different stores.
59. I. 2𝑥 2 + 9𝑥 + 7 = 0 II. 𝑦 2 + 4𝑦 + 4 = 0
Total books = 550
60. I. 2𝑥 2 + 15𝑥 + 28 = 0 II. 2𝑦 2 + 13𝑦 + 21=0

A, 18% 61. Train A completely crosses train B which is 205 m long


E, 22% in 16 second. If they are travelling in opposite direction
and sum of speed of both are 25 m/s. then find the
B, 12%
difference (in meter) between lengths of both trains.
D, 32% C, 16% (a) 5 (b) 6 (c) 8
(d) 10 (e) 12
62. A trader mixes 14 kg rice of variety A which costs Rs.
51. If number of female who bought the books in store E 60/kg with 18 kg of quantity of type B rice. He sells the
100
are 21 more than number of males who bought books mixture at Rs. 65/Kg and earns a profit of %. Then
3
from same store then find the number of females who
what was the cost price of type B rice.
bought book in store E.
(a) 75 (b) 78 (c) 71 (a) 30 (b) 20 (c) 40
(d) 68 (e) 73 (d) 50 (e) 45
49 www.bankersadda.com | www.sscadda.com | www.careerpower.in | Adda247 App
The IBPS RRB PO & Clerk 2021 | SUCCESS GUIDE
63. Present age of A is 3 years less than present age of B. 2
(a) 3
4
(b) 5
3
(c) 8
Ratio of B’s age 5 year ago and A’s age 4 year hence is 4 2
3 : 4 then find present age (in years) of A. (d) (e)
7 7
(a) 20 (b) 17 (c) 23 65. Cost price of B is 200 more than cost price of A. B is sold
(d) 26 (e) 29 at 10% profit and A is sold at 40% loss and selling price
64. A bag contains 6 Red, 5 Green and 4 Yellow coloured of A and B are in the ratio 4 : 11. If A is sold at 20% loss
balls. 2 balls are drawn at random after one another then what will be selling price of A.
without replacement then what is the probability that (a) 320 (b) 400 (c) 240
atleat one ball is Green. (d) 160 (e) 360

Directions (66-70): Read the following table carefully and answer the following questions—
No. of students and % of students passed out of those who appeared are given for two subjects from year 2001 to 2005 in
a college XYZ.

66. Find the average number of students who were failed 71. ? % of (5284.89 ÷ 7.08) = 986.01 – 533. 06
in Economics in year 2002 and year 2003 together? (a) 42 (b) 39 (c) 74
(a) 1435 (b) 1565 (c) 1720 (d) 65 (e) 60
(d) 1590 (e) None of these 72. (1041.84 + ?) ÷ 3.02 = 1816.25 ÷ 4.01
67. Number of students failed in Statistics in the year 2003 (a) 442 (b) 337 (c) 385
is what % of the number of students failed in (d) 268 (e) 320
Economics in the same year? 73. 69.3% of 445.12 ÷ 14.06 = 623.08 ÷ ?
(a) 145.75% (b) 150% (c) 156.25% (a) 28 (b) 19 (c) 21
(d) 158.25% (e) None of these (d) 33 (e) 37
68. Find the ratio between the total number of students 74. ?2 + 114.09 – 24.06 × 5.14 = 163.19
appeared in Economics from 2002 to 2004 together (a) 7 (b) 13 (c) 11
and the total number of students appeared in Statistics (d) 15 (e) 19
from year 2003 to 2005 together? 75. 768.16 ÷ 11.87 × √257 – 58.05 = ?
(a) 13 : 14 (b) 14 : 13 (c) 15 : 16 (a) 1033 (b) 1175 (c) 966
(d) 16 : 15 (e) None of these (d) 880 (e) 975
69. Find the difference between the total number of Directions (76-80): Study the following line graph
students passed in Statistics from year 2002 and total carefully and answer the following questions.
number of students failed in Economics from year Number of males and number of females are given. They
2005. are visiting a place from Monday to Friday.
(a) 690 (b) 385 (c) 485 Male
(d) 550 (e) 610 230

70. Find the average number of students appeared in


Economics from year 2001 to 2004 together? 180
(a) 3090 (b) 3015 (c) 3060
(d) 3075 (e) 3850 130
Direction (71-75): What approximate value should come
in place of question mark (?) in the following questions? 80
(Note: You are not expected to calculate the exact value) MON TUE WED THUS FRI

50 www.bankersadda.com | www.sscadda.com | www.careerpower.in | Adda247 App


The IBPS RRB PO & Clerk 2021 | SUCCESS GUIDE
76. Find the ratio of the total number of males visited the (a) 30 (b) 60 (c) 40
place on Tuesday and Thursday together to the total (d) 50 (e) None of these
number of females visited the place on Monday and
Friday together? 79. If on Saturday the number of males and number of
(a) 29 : 30 (b) 30 : 29 (c) 25 : 26 females increased by 25% and 20% respectively as
(d) 26 : 25 (e) None of these compared to that on Friday then find the total number
77. Total number of males and females together visited the of males and females together visited the place on
place on Tuesday are what percent more/less than the Saturday?
total number of male and females together visited the (a) 196 (b) 306 (c) 316
place on Thursday ? (d) 206 (e) 216
12 3 3
(a) 26 13 % (b) 25 13 % (c) 26 13 %
80. Total number of males and females visited the place on
7
(d) 25 % (e) None of these Monday and Tuesday together is how much more than
13

78. Find the difference between the total number of the total number of males and females visited the place
females visited the place from Monday to Wednesday on Thursday and Friday together?
and the total number of males visited the place from (a) 175 (b) 125 (c) 150
Wednesday to Friday? (d) 160 (e)130

Solutions
REASONING ABILITY
1. (c); LP 13. (a); Only conclusion I follows.
2. (d); 14. (c); Either conclusion I or II follows.
15. (b);Only conclusion II follows.
Directions (16-20):
3. (a); Race, Care
Month Person
4. (c); January C
February A
March G
April E
5. (a);
June D
August F
October B
Directions (6-10):
16. (b); 17. (b); 18. (e);
Number Box
8 B 19. (d); 20. (e);
7 C Directions (21-25):
6 G
5 F Word Code
4 A Card ja
3 H Time sa
2 D Win/team la/ta
1 E Fight da
6. (c); 7. (a); 8. (e); Game/Play pa/ra
9. (e); 10. (e); In fa
11. (e); Both conclusion I and II follow.
21. (a); 22. (b); 23. (c);
12. (a); Only conclusion I follows. 24. (d); 25. (d);
51 www.bankersadda.com | www.sscadda.com | www.careerpower.in | Adda247 App
The IBPS RRB PO & Clerk 2021 | SUCCESS GUIDE
Direction (26-30): Directions (36–40):
36. (d);

37. (e);
26. (a); 27. (e); 28. (e);
29. (b); 30. (e);
Direction (31-35):

38. (e);

39. (a);

31. (b); 32. (b); 33. (c); 40. (d);


34. (b); 35. (c);

QUANTITATIVE APTITUDE

41. (c); Series is ÷2–1, ÷2–1 46. (a); Let a consecutive odd numbers
(22÷2)–1=10 = 𝑥 – 2, 𝑥 and 𝑥 + 2
and consecutive even numbers
42. (d); = 𝑦 – 2, 𝑦, 𝑦 + 2
So, 𝑦 – 2 = 9 + 𝑥 + 2
𝑦 – 𝑥 = 13 … (i)
Adding prime No. and
77 + 11 = 88 (𝑥)2 + 507 = (𝑦)2
𝑦 2 − 𝑥 2 = 507
43. (a); (7+1) × 0.5 = 4 (𝑥 + 𝑦)(𝑦 − 𝑥) = 507
(4+1) × 1 = 5 (𝑥 + 𝑦) =
507
⇒ 𝑥 + 𝑦 = 39 … (𝑖)
(5+1) × 2 = 12 13
Solving (i) and (ii) 𝑦 = 26 and 𝑥 = 13
(12+1) × 4 =52
so smallest odd numbers = 𝑥 – 2 = 13 – 2 = 11
(52 +1)× 8 = 424
44. (c); (6×1)–2 = 4 47. (c); A complete work in 15 days.
B will complete work in 10 days.
(4×2)–3 = 5
They together will complete whole work
(5×3) –4 = 11 15×10
(11×4) –5 = 39 = 25 = 6 𝑑𝑎𝑦𝑠
(39×5) –6 = 189 A and B together worked for = 6 × 2⁄3= 4 days
9.6
45. (d); 48. (d);Speed of downstream = 36 𝑘𝑚 ⁄ 𝑚𝑖𝑛 = 16 𝑘𝑚/ℎ𝑟
Speed of current = 1.6 km/hr
Let speed of man in still water = 𝑥
So, 𝑥 = 16 – 1.6 = 14.4 km/hr
19.2
Required time in upstream = = 1.5 hours
63 – 31 = 32 14.4−1.6

52 www.bankersadda.com | www.sscadda.com | www.careerpower.in | Adda247 App


The IBPS RRB PO & Clerk 2021 | SUCCESS GUIDE
49. (b);Ratio of profit of A and B = 1200 : 750 58. (c); I. 𝑥 2 − 8𝑥 + 15 = 0
= 24 : 15 = 8 : 5 𝑥 2 − 3𝑥 − 5𝑥 + 15 = 0
So, 𝑥(𝑥 − 3) − 5(𝑥 − 3) = 0
1200×12 8
=5 (𝑥 − 3)(𝑥 − 5) = 0
1500×𝑦
𝑥 = 3,5
𝑦 = 6 months
II. 𝑦 2 − 12𝑦 + 36 = 0
x = 6 month
𝑦 2 − 6𝑦 − 6𝑦 + 36 = 0
2𝜋𝑟ℎ 3
50. (d);2𝜋𝑟(𝑟+ℎ) = 5 𝑦(𝑦 − 6) − 6(𝑦 − 6) = 0
(𝑦 − 6)(𝑦 − 6) = 0
5h = 3r + 3h
𝑦=6
2h = 3r
∴𝑥<𝑦
and
2πrh = 1848 59. (e); I. 2𝑥 2 + 9𝑥 + 7 = 0
22 2
2 × × ℎ × ℎ = 1848 2𝑥 2 + 7𝑥 + 2𝑥 + 7 = 0
7 3
ℎ = 21 𝑥(2𝑥 + 7) + 1(2𝑥 + 7) = 0
(𝑥 + 1)(2𝑥 + 7) = 0
51. (c); Let male who purchased book from Store E = 𝑥 𝑥 = −1, −
7
Then 2
22 II. 𝑦 2 + 4𝑦 + 4 = 0
𝑥 + 𝑥 + 21 = × 550
100 𝑦 2 + 2𝑦 + 2𝑦 + 4 = 0
𝑥 = 50 𝑦(𝑦 + 2) + 2(𝑦 + 2) = 0
Required number of females = 50 + 21 = 71 (𝑦 + 2)(𝑦 + 2) = 0
18 𝑥 𝑦 = −2, −2
52. (b); =
5 32
18×32
∴ No relation.
𝑥 = 5 = 18 × 6.4 = 115.2
60. (d);I. 2𝑥 2 + 15𝑥 + 28 = 0
120
53. (e); 𝑇𝑜𝑡𝑎𝑙 𝑏𝑜𝑜𝑘𝑠 𝑜𝑓 𝑠𝑡𝑜𝑟𝑒 𝑋𝑌𝑍 = 100 × 550 2𝑥 2 + 8𝑥 + 7𝑥 + 28 = 0
2𝑥(𝑥 + 4) + 7(𝑥 + 4) = 0
= 660 (2𝑥 + 7)(𝑥 + 4) = 0
Total books sold by store A and B 7
= (18% + 12%) of 660 = 198 𝑥 = (− ) , −4
2
II. 2𝑦 2 + 13𝑦 + 21 = 0
54. (a); Required ratio = (18% + 16%) : (32% + 22%)
= 34 : 54 = 17 : 27 2𝑦 2 + 7𝑦 + 6𝑦 + 21 = 0
𝑦(2𝑦 + 7) + 3(2𝑦 + 7) = 0
55. (c); Required difference (𝑦 + 3)(2𝑦 + 7) = 0
1 −7
= 2 [(32% + 16%) − (18% + 22%)]550 𝑦 = −3,
2
1
= × 8% 𝑜𝑓 550 = 4% 𝑜𝑓 550 = 22 𝑥≤𝑦
2

56. (d);I 𝑥 2 + 5𝑥 + 4𝑥 + 20 = 0 61. (d);In 16 second distance covered by both


𝑥(𝑥 + 5) + 4(𝑥 + 5) = 0 = 16 × 25 = 400 m
(𝑥 + 4)(𝑥 + 5) = 0 So length of A = 400 – 205 = 195
𝑥 = −4, −5 Required difference = 10 m
II. 𝑦 2 = 16 62. (c); Let cost price of mixture = 𝑦
𝑦 = ±4 4
𝑆𝑜, 3 𝑦 = 65
∴𝑥 ≤𝑦
𝑦 = 48.75
57. (a); I. 𝑥 2 − 7𝑥 + 12 = 0 From mixture and allegation
𝑥 2 − 4𝑥 − 3𝑥 + 12 = 0
𝑥(𝑥 − 4) − 3(𝑥 − 4) = 0
(𝑥 − 3)(𝑥 − 4) = 0
𝑥 = 3, 4
II. 3𝑦 2 − 11𝑦 + 10 = 0
3𝑦 2 − 6𝑦 − 5𝑦 + 10 = 0
3𝑦(𝑦 − 2) − 5(𝑦 − 2) = 0 7 48.75−𝑥
= 60−48.75
(3𝑦 − 5)(𝑦 − 2) = 0 9
5 78.75 = 438.75 − 9𝑥
𝑦 = 2, 3
360 = 9𝑥
∴𝑥>𝑦 𝑥 = 40 Rs./kg
53 www.bankersadda.com | www.sscadda.com | www.careerpower.in | Adda247 App
The IBPS RRB PO & Clerk 2021 | SUCCESS GUIDE
63. (a); Let B’s age = 𝑥 (1042+?)
72. (e); 3.02 = 454 ⇒ ? = 320
So A’s age = 𝑥 − 3
𝑥−5 3 310 625
𝑥+1
= 4 ⇒ 𝑥 = 23 73. (a); 14 = ? ⇒ ? ≈ 28
A’s age = 23−3 = 20 𝑦𝑒𝑎𝑟𝑠
74. (b);?2 = 170 ⇒ ? ≈ 13
64. (d);Probability that no ball is green
10𝐶 × 9𝐶 90 3
75. (c); ≈ 64 × 16 – 58 ≈ 966
1 1
= =
15×14 15×14 7
3 4
76. (a); Total no. of males visited on Tuesday and
Required probability = 1 − = Thursday = 140 + 150 = 290
7 7
Total no. of females visited on Monday and Friday
65. (a); Let C.P. of A = 𝑥
= 170 + 130 = 300
So C.P. of B = 200 + 𝑥
Required ratio = 290: 300 = 29: 30
According to question
110
100
(𝑥+200) 11 𝑥+200 1 77. (a); Total no. of males and females together on Tuesday
60 = ⇒ = ⇒ 𝑥 = 400 = 140 + 190 = 330
𝑥 4 6𝑥 4
100
If it is sold at 20% loss then selling price Total no. of males and females together on
80 Thursday = 150 + 110 = 260
= × 400 = 320 330−260 12
100 Required % = 260 × 100 = 26 13 %
66. (b);No. of students failed in Economics in year 2002
(100−45) 78. (d);Total no. of females visited from Monday to
= 100 × 3800 = 2090
Wednesday = 170 + 190 + 140 = 500
No. of students failed in Economics in year 2003 Total no. of males visited from Wednesday to
(100−60)
= 100 × 2600 = 1040 Friday = 180 + 150 + 120 = 450
2090+1040 Required difference = 500 – 450 = 50
𝑅𝑒𝑞𝑢𝑖𝑟𝑒𝑑 𝑎𝑣𝑒𝑟𝑎𝑔𝑒 = 2
= 1565
55×38+40×26 79. (b);On Saturday —
𝑆ℎ𝑜𝑟𝑡 𝑡𝑟𝑖𝑐𝑘 = = 1565
2 Total no. of males visited the place
125
67. (c); No. of students failed in Statistics in year 2003 = 100 × 120 = 150
100−35
=
100
× 2500 = 1625 Total no. of females visited the place
120
No. of students failed in Economics in year 2003 = 100 × 130 = 156
100−60
=
100
× 2600 = 1040 Required males and females = 150 + 156 = 306
1625
𝑅𝑒𝑞𝑢𝑖𝑟𝑒𝑑 % =
1040
× 100 = 156.25% 80. (c); Total males and females visited the place on
𝑆ℎ𝑜𝑟𝑡 𝑡𝑟𝑖𝑐𝑘 =
65×25
× 100 = 156.25% Monday and Tuesday together
40×26 = 160 + 140 + 170 + 190 = 660
68. (d);Total no. of students appeared in Economics from Total males and females visited the place on
2002 to 2004 = 3800 + 2600 + 4800 = 11200 Thursday and Friday together
Total no. of students appeared in Statistics from = 150 + 120 + 110 + 130 = 510
2003 to 2005 = 2500 + 3200 + 4800 = 10500 Required no. of persons = 660 – 510 = 150
Required ratio = 11,200 : 10,500 = 16 : 15
69. (b);Total no. of students passed in Statistics in year
55
2002 = × 2700 = 1485
100
Total no. of students failed in Economics in year
50
2005 = 100 × 2200 = 1100
Required difference = 1485 – 1100 = 385
Short trick = 55 × 27 – 50 × 22 = 385
70. (e); Average no. of students appeared in Economics
from year 2001 to 2004 together
4200+3800+2600+4800 15400
= 4
= 4 = 3850
?
71. (e); 100 × 750 = 450 ⇒ ? ≈ 60

54 www.bankersadda.com | www.sscadda.com | www.careerpower.in | Adda247 App


The IBPS RRB PO & Clerk 2021 | SUCCESS GUIDE

IBPS RRB CLERK PRELIMS MEMORY BASED PAPER 2020


REASONING ABILITY

Directions (1-5): Study the following information 8. If all the consonants are removed from the given series,
carefully and answer the questions given below: then which among the following element is ninth from
the left end?
Eight persons i.e. D, Q, G, H, K, S, E and W are sitting around (a) 2 (b) O (c) 3
a square table in such a way that four persons sit at the (d) 7 (e) None of these
corner side of the table and other four persons sit in the
middle side of the table. The persons who sit at the corner 9. Which among the following element is fifth to the left
side are facing away from the center and the persons who of twelfth element from the left end?
(a) F (b) 8 (c) 9
sit in the middle side are facing towards the center. D sits
(d) 1 (e) None of these
at the corner side of the table. One person sits between D
and G. Q sits third to the right of G. Three persons sit Directions (10-12): Study the following information
between Q and E. W sits second to the right of E. W is not carefully and answer the questions given below:
an immediate neighbour of D. H sits second to the right of Five persons A, B, C, D and E have different heights. Less
K. than two persons are shorter than D. As many persons are
1. Who among the following sits second to the right of S? taller than D as shorter than C. A is taller than B but shorter
(a) Q (b) E (c) D than E. B is not the shortest person. The height of third
(d) W (e) None of these tallest person is 86 cm.
10. If the height of shortest person is 68 cm, then what may
2. How many persons sit between D and W when counted
be the height of B?
from the right of D? (a) 69 cm (b) 81 cm (c) 78 cm
(a) Three (b) Four (c) One (d) All the given heights (e) None of these
(d) Two (e) None of these
11. How many persons are taller than C?
3. Who among the following sits second to the left of G? (a) None (b) One (c) Two
(a) D (b) K (c) H (d) Three (e) None of these
(d) S (e) None of these
12. Who among the following is just shorter than E?
4. Who among the following are the immediate (a) None (b) C (c) A
neighbours to each other? (d) D (e) None of these
(a) K, E (b) Q, D (c) G, H Directions (13-16): In each of the questions below are
(d) W, S (e) None of these given some statements followed by some conclusions. You
5. Four of the following five are alike in a certain way and have to take the given statements to be true even if they
hence form a group. Find the one who does not belong seem to be at variance with commonly known facts. Read
to that group? all the conclusions and then decide which of the given
(a) E (b) D (c) G conclusions logically follows from the given statements
(d) K (e) H disregarding commonly known facts.

Directions (6-9): Study the following sequence of


numbers and alphabets and answer the given questions-
P4SAW58F91RE72O3751B6KGN
6. How many numbers are there which are immediately
preceded by a vowel?
(a) One (b) None (c) Two
(d) Three (e) None of these
7. If all the numbers are removed from the given series,
then which among the following element is seventh
from the right end?
(a) E (b) O (c) F
(d) R (e) None of these

55 www.bankersadda.com | www.sscadda.com | www.careerpower.in | Adda247 App


The IBPS RRB PO & Clerk 2021 | SUCCESS GUIDE
13. Statements: All Greens are Yellows. No Yellows are 18. Who among the following sits third to the right of G?
Black. (a) E (b) C (c) B
Conclusions: I. No Greens are Black. (d) F (e) None of these
II. Some Greens are Black.
(a) If only conclusion I follows. 19. Who among the following persons sit at the extreme
(b) If only conclusion II follows. ends?
(c) If either conclusion I or II follows. (a) F, A (b) A, E (c) G, E
(d) If neither conclusion I nor II follows. (d) F, E (e) None of these
(e) If both conclusions I and II follow.
20. How many persons sit between D and E?
14. Statements: All Chairs are Sofas. Only a few Sofas (a) Two (b) Four (c) Three
are Beds. No Beds are Curtains. (d) One (e) None of these
Conclusions: I. Some Sofas are not Beds.
II. Some Sofas are not Curtains. 21. Who among the following sits to the immediate left of
(a) If only conclusion I follows. E?
(b) If only conclusion II follows. (a) G (b) A (c) B
(c) If either conclusion I or II follows. (d) D (e) None of these
(d) If neither conclusion I nor II follows.
(e) If both conclusions I and II follow. Directions (22-23): Study the following information
carefully and answer the questions given below:
15. Statements: Only a few Coffee are Tea. All Tea is Point D is 10m north of point P. Point Y is 14m east of point
Drinks. Only a few Drinks are Cold D. Point Q is 8m north of point Y. Point S is 20m west of
drinks.
point Q. Point H is 8m south of point S.
Conclusions: I. Some Tea is not Cold drinks.
II. No Coffee are Drinks. 22. What is the shortest distance between point H and
(a) If only conclusion I follows. point D?
(b) If only conclusion II follows. (a) 8m (b) 6m (c) 4m
(c) If either conclusion I or II follows. (d) 10m (e) None of these
(d) If neither conclusion I nor II follows.
(e) If both conclusions I and II follow. 23. In which direction is point P with respect to point Q?
(a) South east (b) North west (c) South west
16. Statements: All Flowers are Trees. Only a few
(d) North east (e) None of these
Trees are Gardens. No Gardens are
Lawns. Directions (24-27): Study the following information
Conclusions: I. All Lawns can never be Trees carefully and answer the questions given below:
II. Some Flowers can be Gardens There are certain number of persons sitting in a row facing
(a) If only conclusion I follows. towards north direction. A sits fifth to the right of B. Two
(b) If only conclusion II follows. persons sit between C and B. D sits fourth to the left of C.
(c) If either conclusion I or II follows.
Four persons sit to the left of D. The number of persons sit
(d) If neither conclusion I nor II follows.
between D and B is same as the number of persons sit
(e) If both conclusions I and II follow.
between B and F. F sits at fourth position from one of the
Directions (17-21): Study the following information extreme ends.
carefully and answer the questions given below:
Seven persons A, B, C, D, E, F and G are sitting in row and 24. How many persons sit between B and D?
all are facing towards north but not necessarily in the same (a) None (b) Six (c) Five
order. B sits second from one of the extreme ends. Three (d) Four (e) None of these
persons sit between D and B. Two persons sit between D
25. How many persons sit in the row?
and A. C sits to the immediate left of A. F sits to left of C but
is not an immediate neighbour of C. More than two persons (a) Twenty (b) Twenty-one (c) Twenty-two
sit between F and E. (d) Nineteen (e) None of these

17. How many persons sit to the left of F? 26. Who among the following sits second to the right of A?
(a) Two (b) None (c) One (a) None (b) C (c) D
(d) Three (e) None of these (d) F (e) None of these
56 www.bankersadda.com | www.sscadda.com | www.careerpower.in | Adda247 App
The IBPS RRB PO & Clerk 2021 | SUCCESS GUIDE
27. How many persons sit to the right of B? Directions (35-39): Study the following information
(a) Nine (b) Ten (c) Eight carefully and answer the questions given below:
(d) Eleven (e) None of these Six people P, Q, R, S, T and U have events on different dates
7th and 12th of different months i.e. January, February and
28. How many such pairs of letters are there in the
March. D has event on even numbered date in the month
meaningful word ‘MATCHES’ each of which has as
having 31 days. The number of persons have event before
many letters between them in the word as in the
D is same as the number of persons have event after A. One
English alphabet (From both backward and forward)?
person has event between A and C. F has event before C. B
(a) Two (b) One (c) More than three
has event just before E.
(d) Three (e) None of these
35. How many persons have event before B?
Directions (29-33): Study the following sequence
(a) Two (b) None (c) Three
carefully and answer the given questions.
(d) One (e) None of these
COT IVY PEA FOX MRU
36. Who among the following has event just after D?
29. If we add ‘L’ after first letter in every word, then how
(a) C (b) None (c) B
many meaningful words will be formed?
(d) F (e) None of these
(a) None (b) Three (c) Two
(d) One (e) None of these 37. How many persons have event between F and D?
(a) One (b) Three (c) None
30. If third letter of each word is replaced by its succeeding
(d) Two (e) None of these
letter according to English alphabetical order, then in
how many words vowels will appear more than once? 38. E has event on which among the following date?
(a) Two (b) One (c) None (a) 12th February(b) 12th March (c) 7th March
(d) Three (e) None of these (d) 7th January (e) None of these
31. If all the words are arranged according to English 39. Four of the following five are alike in a certain way and
alphabetical order from left to right, then which word hence form a group. Which is the one that does not
will appear fourth from the left end? belong to that group?
(a) MRU (b) FOX (c) PEA (a) A, F (b) E, F (c) F, C
(d) IVY (e) None of these (d) E, B (e) C, B
32. If all the letters are arranged according to English 40. Four of the following five are alike in a certain way and
alphabetical order within each word, then in how hence form a group. Which is the one that does not
many words vowel will appear at second position? belong to that group?
(a) One (b) None (c) Two (a) RUY (b) SQO (c) OMK
(d) Three (e) None of these (d) FDB (e) YWU
33. How many letters are there in English alphabetical
series between the first letter of the second word from
the left end and third letter of the third word from the
right end?
(a) Five (b) Six (c) Seven
(d) Four (e) None of these
34. If in the number ‘35982476’, 1 is added to each even
digit and 2 is subtracted from each odd digit, then
which digits will not appear twice in the number thus
obtained?
(a) Only 1 (b) Only 9 (c) Both ‘1’ and ‘9’
(d) Only 5 (e) None of these

57 www.bankersadda.com | www.sscadda.com | www.careerpower.in | Adda247 App


The IBPS RRB PO & Clerk 2021 | SUCCESS GUIDE
Quantitative Aptitude

Directions (41-45): Line graph given below shows number of passengers travelling in five (A, B, C, D & E) different
compartment of a trains. Read the data carefully and answer the questions.

41. Total passengers in E are what percent less than total 49. 72, 79, 65, 93, ?, 149
passengers in A? (a) 36 (b) 31 (c) 33
(a) 6 ¼ % (b) 8 ⅓% (c) 6 ⅓% (d) 37 (e) 35
(d) 6 ⅔% (e) 5%
50. 8, 9, 19, 58, 233, ?
42. Find average number of passengers in A, C & E? (a) 1164 (b) 1166 (c) 1156
(a) 32 (b) 30 (c) 36
(d) 1152 (e) 1158
(d) 33 (e) 27
43. Find the ratio of total passenger in B to that of in D? 51. If the difference between the present age of P and Q is
(a) 7 : 9 (b) 9 : 10 (c) 11 : 9 three years and the ratio between the age of P and Q
(d) 9 : 13 (e) 9 : 11 after two years will be 5 : 4, then find the age of P after
two years (in years)?
44. Total passenger in C and E together are what percent
(a) 15 (b) 13 (c) 18
more than total passenger in A?
(a) 33 ⅓% (b) 66 ⅔% (c) 66 ⅓% (d) 16 (e) 14
(d) 50% (e) 60% 52. A and B both spend 30% of their income together
45. Find total number of passengers traveling in B, C & D which is equal to Rs. 26400. If income of A is 20% more
together? than that of B, then find the income of B (in Rs.)?
(a) 69 (b) 65 (c) 67 (a) 52000 (b) 48000 (c) 40000
(d) 63 (e) 71 (d) 36000 (e) 30000
Directions (46-50): What will come in the place of 53. If a man invests equal sum at the same rate of interest
question (?) mark in following questions. on simple interest for T and T+4 years and the
46. 12, 12, 24, 72, ?, 1440 respective ratio of interest gets by man is 1:2
(a) 256 (b) 288 (c) 284 respectively, then find ‘T’?
(d) 296 (e) 316 (a) 6 (b) 2 (c) 5
47. 16, 17.8, 21.4, 28.6, 43, ? (d) 3 (e) 4
(a) 69.8 (b) 72.8 (c) 73.8 54. 12 women can complete a work in 64 day, then find
(d) 70.8 (e) 71.8
how many women will be required to complete 2/3 rd
48. 12, 7, 8, 13, ?, 68.5 of the same work in 16 days?
(a) 28 (b) 27 (c) 26 (a) 28 (b) 24 (c) 36
(d) 27.5 (e) 26.5 (d) 32 (e) 48

58 www.bankersadda.com | www.sscadda.com | www.careerpower.in | Adda247 App


The IBPS RRB PO & Clerk 2021 | SUCCESS GUIDE
55. A train running at the speed of 72 kmph crosses a pole 62. Find total orders (all three items) received by R is what
in 30 seconds. Find the time taken by the same train to percent more than that of total orders (all three items)
cross the pole with the speed of 54 kmph (in sec)? received by Q?
(a) 42 (b) 48 (c) 54 (a) 5% (b) 12.5% (c) 10%
(d) 45 (e) 40 (d) 15% (e) 20%
56. The upstream speed and downstream speed of a boat 63. Find ratio of total orders of item A & B received by P to
is 10 kmph and 14 kmph respectively and boat total orders of item B & C received by Q?
travelled for T hours & 6 hours in upstream and (a) 7 : 9 (b) 8 : 7 (c) 4 : 7
downstream respectively. If the distance travelled in (d) 5 : 6 (e) 7 : 8
downstream is 44 km more than upstream, then find 64. Find average number of orders of item B received by Q
the value of ‘T’ & R is what percent of total orders of item A received
(a) 4 (b) 3 (c) 6 by P?
(d) 5 (e) 8 (a) 104 ¼ % (b) 106 ¼ % (c) 108 ¼ %
57. An article was marked up by 50% above cost price and (d) 102 ¼ % (e) 110 ¼ %
allowed Rs 50 discount on marked price. If shopkeeper 65. Find total orders of item A, B & C received by P?
still made a profit of Rs. 50, then find the selling price (a) 210 (b) 220 (c) 190
of the article (in Rs.)? (d) 180 (e) 200
(a) 350 Rs. (b) 300 Rs. (c) 250 Rs. Directions (66-80): What should come in place of
(d) 200 Rs. (e) 150 Rs. question mark (?) in following questions?
58. A & B invested Rs. X and Rs. (X + 800) for same period 66. (48% of 625) ÷ 0.75 = ?
of time in a business. If A gets Rs. 3200 as profit share (a) 800 (b) None of these (c) 40
out of total profit of Rs. 6800, then find ‘X’? (d) 4000 (e) 400
(a) 7800 (b) 6000 (c) 8400
(d) 7200 (e) 6400 67.
59. A vessel contains mixture of milk and water in the (a) 1 (b) 2 (c) 4
ration of 3 : 1 respectively. If 20 liters mixture taken (d) 5 (e) 3
out from the vessel and now the difference between
milk and water in the remaining mixture is 70 liters, 68.
then find initial mixture in vessel (in liters)?
(a) 240 (b) 160 (c) 120 (a) 2 (b) 3 (c) 4
(d) 80 (e) 180 (d) 1 (e) None of these

60. Perimeter of a rectangle is 2 cm more than 69.


circumference of a circle and area of circle is 616 cm². (a) 6 (b) 12 (c) 8
If breath of rectangle is equal to radius of circle, then (d) 4 (e) 16
find length of rectangle (in cm)? 70. ? + 432 – 205 = 550
(a) 35 (b) 33 (c) 31 (a) 384 (b) 244 (c) 224
(d) 21 (e) 27 (d) 276 (e) 324
Directions (61-65): Table given below shows number of 71.
orders received by three (P, Q & R) companies of their (a) 10 (b) 12 (c) 8
three (A, B & C) items. Read the data carefully and answer (d) 6 (e) 9
the questions.
72. 40% of 400 + ? % of 300 = 250
(a) 40 (b) 36 (c) 25
(d) 30 (e) 20

73.
(a) 185 (b) 183 (c) 187
(d) 184 (e) 182
61. Total orders of item A & B received by R is how much
more than total orders of item B & C received by Q? 74.
(a) 50 (b) 10 (c) 40 (a) 1 (b) 4 (c) 5
(d) 20 (e) 30 (d) 2 (e) 3

59 www.bankersadda.com | www.sscadda.com | www.careerpower.in | Adda247 App


The IBPS RRB PO & Clerk 2021 | SUCCESS GUIDE
75. 78.
(a) 10 (b) 13 (c) 17 (a) 3 (b) 4 (c) 2
(d) 8 (e) 16 (d) 1 (e) 5
76. 12% (? + 100) = 18 79.
(a) 40 (b) 50 (c) 30
(d) 100 (e) 60 (a) 4 (b) 4 ½ (c) 5
(d) 5⅙ (e) 6
77.
80.
(a) 19 (b) 18 (c) 17 (a) 2 (b) 3 (c) 0
(d) 16 (e) 15 (d) 6 (e) 7

Solutions
REASONING ABILITY
Directions (1-5): 16. (b):

Directions (17-21):

1. (b): 2. (d): 3. (c):


17. (b): 18. (c): 19. (d):
4. (b): 5. (a): 6. (c):
20. (b): 21. (c):
7. (d): 8. (a): 9. (b):
Directions (22-23):
Directions (10-12): 22. (b):
C > E > A (86) > B > D
10. (d):
11. (a):
12. (c):
Directions (13-16):
13. (a):

23. (c):

S14. (e):

15. (d):

60 www.bankersadda.com | www.sscadda.com | www.careerpower.in | Adda247 App


The IBPS RRB PO & Clerk 2021 | SUCCESS GUIDE
Directions (24-27): 32. (d): 33. (c):
34. (c): 3 5 9 8 2 4 7 6
24. (b): 25. (c): 26. (d): 13793557
27. (b): Directions (35-39):
28. (c): Months 7th 12th
January A F
February C B
March E D

Directions (29-33): 35. (c): 36. (b): 37. (b):


29. (c): 30. (b): 31. (a): 38. (c): 39. (b): 40. (a):

Quantitative Aptitude

41. (b): Required percentage =


36−33
× 100 50. (b): Pattern of series –
36
3 1 8×1+1=9
= 36 × 100 = 8 3 % 9 × 2 + 1 = 19
36+27+33 19 × 3 + 1 = 58
42. (a): Required average = = 32
3 58 × 4 + 1 = 233
43. (e): Required ratio = 18 : 22 = 9 : 11 233 × 5 + 1 = 1166

44. (b): Total passenger in C and E = 27 + 33 = 60 51. (a): Let present age of Q = t years
60−36 So, present age of P = (t + 3) years
Required percentage = × 100
24 2
36 ATQ –
= × 100 = 66 % 𝑡+2
=
4
36 3 (𝑡+3)+2 5
45. (c): Required number of passengers t = 10 years
= 18 + 27 + 22 = 67 So, Age of P after two years
= (10 + 3) + 2 = 15 year
46. (b): Pattern of series –
12 × 1 = 12 52. (c): Let total income of B = 100x Rs.
12 × 2 = 24 So, total income of A
24 × 3 = 72 20
= 100x × (1 + ) = 120𝑥 𝑅𝑠.
? = 72 × 4 = 288 100
ATQ –
288 × 5 = 1440 30
(100x + 120x) × 100 = 26400
47. (e): Pattern of series –
16 + 1.8 = 17.8 66x = 26400
17.8 + 3.6 = 21.4 x = 400 Rs.
21.4 + 7.2 = 28.6 So, income of B = 400 × 100 = 40000 𝑅𝑠.
28.6 + 14.4 = 43 53. (e): Let sum invested by man = Rs. X
? = 43 + 28.8 = 71.8 And, rate of interest = r%
𝑋 ×𝑟×𝑇 1
48. (b): Pattern of series – ATQ- 𝑋×𝑟×(𝑇+4) = 2
12 × 0.5 + 1 = 7 𝑇 1
= ⇒T= 4
7×1+1 = 8 (𝑇+4) 2
8 × 1.5 + 1 = 13 54. (d): Let total work = 12 × 64 = 768 units
? = 13 × 2 + 1 = 27 2 1
Required women = 768 × × = 32
27 × 2.5 + 1 = 68.5 3 16
55. (e): Let length of train be ‘l’ meters
49. (d): Pattern of series – ATQ –
72 + 7 = 79 5 𝑙
79 – 14 = 65 72 × 18 = 30
65 + 28 = 93 l = 600 meters
600
? = 93 – 56 = 37 Required time = 5 = 40 sec
54×
37 + 112 = 149 18

61 www.bankersadda.com | www.sscadda.com | www.careerpower.in | Adda247 App


The IBPS RRB PO & Clerk 2021 | SUCCESS GUIDE
56. (a): ATQ – 64. (b): Average number of orders of item B received by
14 × 6 − 10 × 𝑇 = 44 Q&R=
70+100
= 85
2
10T = 40 ⇒ T = 4 85 1
Required percentage = 80 × 100 = 106 4 %
57. (c): Let cost price of article = 100x Rs.
So, marked price of article 65. (c): Required sum = 80 + 60 + 50 = 190
50
= 100x × (1 + 100) = 150𝑥 Rs. 48
66. (e): 100 × 625 × 3 =? ⇒ ? = 400
4

And, selling price of article = (150x – 50) Rs. 64+324


ATQ – 67. (c): 97
=?
(150x – 50) – 100x = 50 ?= 4
50x = 100 ⇒ x = 2 Rs. 256
So, selling price of article 68. (b): 4? × 2 = 2
= (150 × 2 − 50) = 250 𝑅𝑠. 4? = 64
4? = (4)3 ⇒ ? = 3
58. (e): ATQ –
𝑋 3200
= (6800−3200) 69. (c): 4 × ? = 512 − 480
(𝑋+800) 32
X = 6400 ?= 4 ⇒?=8

59. (b): Let total initial mixture in vessel = 4x 70. (e): ? + 432 – 206 = 550
So, milk in vessel = 3x ? = 550 – 226
And water in vessel = x ? = 324
ATQ – 71. (a): (?)2 = 196 – 96
3𝑥 𝑥
(3x – 20 × ) − (𝑥 − 20 × ) = 70 ?2 = 100 ⇒ ? = 10
4𝑥 4𝑥
(3x – 15) – (x – 5) = 70 40 300
72. (d): × 400 + ×? = 250
2x = 80 ⇒ x = 40 100 100
So, initial mixture in vessel 160 + 3 × ? = 250
90
= 4x = 4 × 40 = 160 liters ? = 3 = 30
60. (c): Let radius of circle be ‘r’ cm 73. (b): ÷ 7 = ? −180 ⇒ ? = 183
ATQ –
22 74. (e): 24 – 12 + 27 = 36 + ?
× 𝑟 × 𝑟 = 616 ?= 3
7
r = 14 cm = breath of rectangle
Let length of rectangle be ‘l’ cm 75. (b): 119 + 41 + 9 =?2
Perimeter of rectangle = circumference of a circle ? = 13
+2 76. (b):
12
× (? +100) = 18
22 100
2(14 + l) = 2 × × 14 + 2 ? = 150 – 100 ⇒ ? = 50
7
2(14 + l) = 90 ⇒ l = 31 cm 11
77. (c): + 9+? = 27
11
61. (d): Required difference = (80 + 100) – (70 + 90) = 20
1 + 9 + ? = 27 ⇒ ? = 17
62. (a): Total orders (all three items) received by R
78. (b): ?2 + 20 = 36
= (80 + 100 + 30) = 210
?2 = 16 ⇒ ? = 4
Total orders (all three items) received by Q
1 1 1 1
= (40 + 70 + 90) = 200 79. (d): ? = 9 3 + 7 2 − 5 6 − 6 3
210−200
Required percentage = 200 × 100 = 5% ?= 9 + 7 − 5 − 6 ( + − − )
1 1 1 1
3 2 6 3
1
63. (e): Total orders of item A & B received by P ?= 5
6
= 80 + 60 = 140
34 ×37×2
Total orders of item B & C received by Q 80. (c): = 3?
36×3
= 70 + 90 = 160 3? = 34+14-18
Required ratio = 140 : 160 = 7 : 8 3 ? = 30 ⇒ ? = 0

62 www.bankersadda.com | www.sscadda.com | www.careerpower.in | Adda247 App


The IBPS RRB PO & Clerk 2021 | SUCCESS GUIDE

IBPS RRB CLERK PRELIMS MEMORY BASED PAPER 2019


REASONING ABILITY

Directions (1-4): In each of the question, relationships 6. Who among the following lives on the top floor?
between some elements are shown in the statements. (a) E (b) B (c) F
These statements are followed by conclusions numbered I (d) D (e) None of these
and II. Read the statements and give the answer. 7. Number of persons lives above F is same as the number
(a) If only conclusion I follows. of persons below __ ?
(b) If only conclusion II follows. (a) B (b) D (c) C
(c) If either conclusion I or II follows. (d) G (e) None of these
(d) If neither conclusion I nor II follows. 8. How many floors are there above the floor on which G
(e) If both conclusions I and II follow. lives?
1. Statements: P < R ≤ M = L > O ≤ V > Y (a) One (b) Two (c) Three
Conclusions: I. L > P II. O > R (d) More than Four (e) Four
9. Who lives immediately below A?
2. Statements: A ≥ B > D = F < E ≤ C
(a) D (b) E (c) F
Conclusions: I. B > E II. D < C (d) C (e) None of these
3. Statements: A = E ≥ D ≥ C < F ≤ B Directions (10-14): Study the following sequence and
Conclusions: I. C < A II. A = C answer the given questions.
4. Statements: F ≥ N = O > P ≤ K > T A@3%4 ENM$8&6LDS♠986QYZ17%ROG
Conclusions: I. K < F II. N < K 2IB2U&
Direction (5-9): Study the following information carefully 10. Which of the following element is twelfth to the left of
and answer the question given below- the twentieth element from the left end of the given
arrangement?
Seven people viz. A, B, C, D, E, F and G lives in a building on (a) 6 (b) & (c) M
seven different floors such as ground floor is numbered 1, (d) $ (e) None of these
the floor just above is numbered 2 and so on till top floor
numbered as seven but not necessarily in the same order. 11. If all the symbols are dropped from the series, which
There are less than three floors above A. Only one person element will be fourth to the right of the one which is
lives between C and A. G lives immediately below D. D lives twelfth from the right end?
(a) 9 (b) O (c) R
on an even number floor. B lives immediately above A. F (d) 7 (e) None of these
lives above E. F does not lives on the 5th floor. F does not
lives on an even number floor. 12. How many such numbers are there in the given series
which are immediately preceded by a symbol and
5. Four of the following five belongs to a group find the followed by a letter?
one that does not belongs to that group? (a) None (b) One (c) Two
(a) CD (b) EC (c) FB (d) Three (e) Four
(d) AB (e) GC 13. Four of the following five are alike in a certain way and
forms a group find the one that does not belongs to that
group?
(a) 3E% (b) R2 (c) M&$
(d) D9S (e) Y7Z
14. What should come in place of question mark (?) in the
following series based on the above arrangement?
34% N$M 6DL 8Q6 ?
(a) %OR (b) 7Z% (c) O%R
(d) R%O (e) R%7
Direction (15-19): Study the following information
carefully and answer the question given below-
Seven people viz. P, Q, R, S, T, U and V are sitting around a
circular table having equal distance between them. All of
them are facing inside.
63 www.bankersadda.com | www.sscadda.com | www.careerpower.in | Adda247 App
The IBPS RRB PO & Clerk 2021 | SUCCESS GUIDE
P sits immediate right of Q. Only one person sits between P 22. Statements: Only a few wells are mats.
and S (either from left or right). U sits third to the right of All pillows are mats.
S. T is an immediate neighbor of U. R sits second to the left Conclusions: I. At least some pillows are wells.
of V. II. All wells can never be pillow.
15. If all the persons are arranged according to the (a) Both I and II follow
alphabetical order in anticlockwise direction starting (b) Either I or II follows
from P, then how many persons position will remain (c) Only II follows.
unchanged (except P)? (d) Only I follow.
(a) Three (b) One (c) Two (e) Neither I nor II follows
(d) None (e) None of these Direction (23-27): Study the following information
16. How many persons sits between Q and U, if counted carefully and answer the question given below-
from the left of Q? There are ten persons are sitting in two parallel row such
(a) One (b) Two (c) Three that five persons are sitting in each row. A, B, C, D and E are
(d) None (e) None of these sitting in row 1 and faces north and M, N, O, P and R are
17. Who sits second to the right of T? sitting in row 2 and faces south such that persons sitting in
(a) P (b) Q (c) R row 1 faces the persons sitting row 2.
(d) S (e) None of these B sits immediate right of A. Neither A nor B sits at the
18. Four of the following five belongs to a group find the extreme ends. Two person sits between P and N. B faces the
one that does not belongs to that group? one who sits on the immediate left of P. M sits on the
(a) VQ (b) PV (c) RT immediate right of R. C sits at the end of the row. D sits on
(d) SU (e) TQ the left of E. D does not face R.
19. Who among the following sits second to the left of the 23. Four of the following five belongs to a group find the
one who sits 4th to the right of V? one that does not belongs to that group?
(a) U (b) T (c) R (a) O (b) C (c) D
(d) S (e) None of these (d) P (e) N
Directions (20-22): In each of the questions below are 24. Who among the following sits second to the left of the
given some statements followed by two conclusions. You one who faces B?
have to take the given statements to be true even if they (a) R (b) N (c) O
seem to be at variance with commonly known facts. Read (d) M (e) None of these
all the conclusions and then decide which of the given
25. How many persons sits on the left of N?
conclusions logically follows from the given statements,
(a) One (b) Two (c) No one
disregarding commonly known facts. Give answer
(d) Three (e) None of these
20. Statements: Only a few lamps are bottles.
No bottle is ship. 26. How many persons sits between D and C?
Conclusions I. Some ships are definitely not (a) One (b) Two (c) Three
lamps. (d) No One (e) Can’t be determined
II. All lamps can never be ships. 27. Who among the following faces A?
(a) Both I and II follow (a) M (b) N (c) O
(b) Either I or II follows (d) R (e) None of these
(c) Only II follows.
(d) Only I follow. 28. IF ‘He will Say’ is coded as ’1 3 9’ and ‘Say To Him’ is
(e) Neither I nor II follows coded as ‘3 5 2’ and ‘He May Do’ is coded as ‘8 7 9’ then
what will be the code of ‘will’?
21. Statements: All bamboos are sticks (a) 3 (b) 1 (c) 9
No bamboos is a fish. (d) 8 (e) Can’t be determined
Conclusions: I. Some sticks are fish.
II. No sticks are fish. 29. How many pairs of letters are there in the word
(a) Both I and II follow “MINUTE” each of which have as many letters between
(b) Either I or II follows them in the word as they have between them in the
(c) Only II follows. English alphabetical series?
(d) Only I follows. (a) Three (b) One (c) Two
(e) Neither I nor II follows (d) More than three (e) None

64 www.bankersadda.com | www.sscadda.com | www.careerpower.in | Adda247 App


The IBPS RRB PO & Clerk 2021 | SUCCESS GUIDE
Direction (30-33): Study the following information 34. How many persons are sitting in the row?
carefully and answer the question given below- (a) 21 (b) 23 (c) 24
There are six persons i.e. A, B, C, D, E and F who all are of (d) 26 (e) Can’t be determined
different weight. No two persons have same weight. Only 35. If G sits 2nd to the right of S, then what is the position
two persons are lighter than A. B is heavier than A but of G from right end?
lighter than C and D. F is heavier than E but lighter than D. (a) 7 (b) 9 (c) 8
D is not the heaviest. The weight of 2nd heaviest person is (d) 6 (e) None of these
115 kg and the weight of lightest is 56 kg.
36. What is the position of F with respect to E?
30. How many persons are heavier than F? (a) Second to the right
(a) One (b) Two (c) Three (b) Second to the left
(d) Four (e) None of these
(c) Third to the left
31. If the sum of weight of E and A is 131 and the sum of (d) Fifth to the Right
weight of D and B is 213, then what is the sum of weight (e) None of these
of A and B?
Direction (37-39): Study the following information
(a) 172 (b) 173 (c) 174
(d) 175 (e) None of these carefully and answer the question given below-
Point C is 12m west of point A. Point B is 18m north of point
32. Which among the following person is the 2nd heaviest? A. Point E is 9m south of point D. Point F is 14m west of
(a) A (b) B (c) C point E. Point D is 28m east of point B. F is 13m south of
(d) D (e) None of these point G.
33. Which of the following statement is true? 37. Four of the following five belongs to a group find the
I. Only two persons are heavier than B. one that does not belongs to that group?
II. Sum of weight of D and E is 171 Kg.
(a) CB (b) AD (c) AE
III. Weight of E is 58 Kg.
(d) BG (e) FB
(a) Only II (b) Both I and II
(c) Both III and II (d) All are True (e) Only III 38. In which direction point A with respect to point G?
(a) North-west (b) South-east (c) South-west
Direction (34-36): Study the following information
carefully and answer the question given below- (d) North (e) North-east
Uncertain number of persons are sitting in a linear row 39. If Point S is 4m south of point G then what is the
facing north. B sits fifth to the left of E. Two persons sits distance between point B and point S?
between B and D. D sits second position from one of the (a) 28m (b) 9m (c) 8m
extreme end. Five persons sits between S and E. S is not an (d) 14m (e) None of these
immediate neighbor of B. As many as persons sits between
E and S as between S and C. As many as persons sits 40. Find the odd one out?
between D and B as between B and F. C sits third position (a) PSRQ (b) MONL (c) ADCB
from the extreme end. (d) VYXW (e) ILKJ

Quantitative Aptitude

41. 1, 2, 5, 16, 65, 328, 1957 44. 3, 5, 8, 17, 33, 58, 94


(a) 5 (b) 328 (c) 16 (a) 8 (b) 94 (c) 58
(d)1957 (e) 65 (d) 3 (e) 5
45. A boat covers 36 km in upstream in 2 hours and 66 km
42. 4, 11, 25, 46, 74, 129, 151
in downstream in 3 hours. Find the speed of boat in
(a) 129 (b) 11 (c) 151 still water?
(d) 4 (e) 46 (a) 21km/h
(b) 19 km/h
43. 84, 96, 83, 95, 80, 94, 81 (c) 20.5 km/h
(a) 95 (b) 81 (c) 83 (d) 20 km/h
(d) 80 (e) 84 (e) 19.5 km/h

65 www.bankersadda.com | www.sscadda.com | www.careerpower.in | Adda247 App


The IBPS RRB PO & Clerk 2021 | SUCCESS GUIDE
46. Two inlet taps A and B can fill a tank in 36 minutes and water is added again then ratio of water to that of the
60 minutes respectively. Find the time taken by both milk becomes 1:2. Find value of x?
1
the taps together to fill 6 𝑡ℎ of the tank? (a) 10 liter (b) 8 liter (c) 12 liter
3 1 (d) 6 liter (e) 9 liter
(a) 3 minutes (b) 3 minutes (c)3 minutes
1 1
4 2 49. Total cost of x pens and (x-2) pencils is Rs 424. If one
(d) 3 minutes (e) 2 minutes pencil and one pen costs Rs 4 and Rs 20 respectively
3 3
then find x?
47. If circumference of first circle is 132 cm and
(a) 16 (b) 18 (c) 15
circumference of second circle is 110 cm then find the
(d) 20 (e) 21
difference between area of both the circle?
(a) 423.5 cm2 (b) 412.5 cm2 (c) 420 cm2 50. A is 6 years younger than B and ratio of present age of
(d) 422.4 cm2 (e) 419.8 cm2 B to C is 12:5. If ratio of present age of A to C is 2:1
then find present age of B?
48. In 64 liter of pure milk, 20 liter of water is mixed and (a) 20 years (b) 30 years (c) 24 years
1
then 4th of the mixture is taken out. When x liter of (d) 18 years (e) None of these
Directions (51-55): Given bar graph shows the data of two types of school buses X and Y for three schools A, B and C.
Study the chart carefully and answer the following questions.

Bus Type X Bus Type Y


60

50
Number of buses

40

30

20

10

0
A B C
Schools
51. What is the average number of X type buses from 55. Which school has maximum number of buses?
school B and school C together? (a) School B
(a) 40 (b) 70 (c) 30 (b) School C
(d) 59 (e) 54 (c) School A & School C
(d) School A & School B
52. X type buses from school A are how much more than (e) School A
that of X type buses from school B? Directions (56-60): Given below are two equations in
(a) 555/19% (b) 25% (c) 55/9% each question, which you have to solve and give answer
(d) 45 /6%
5 (e) 33 /3%
1
(a) if𝑥 > 𝑦
(b) if𝑥 ≥ 𝑦
53. What is the average number of all the buses from
(c) if𝑦 > 𝑥
school B? (d) if𝑦 ≥ 𝑥
(a) 43 (b) 39 (c) 31 (e) if𝑥 = 𝑦 or no relation can be established
(d) 54 (e) 59
56. I.2𝑥 2 − 5𝑥 + 2 = 0 II.2𝑦 2 − 9𝑦 + 7 = 0
54. What is the difference of average number of all buses 57. I.3𝑥 2 + 7𝑥 + 4 = 0 II. 𝑦 2 + 9𝑦 + 20 = 0
from school A and average number of all buses from
school C? 58. I.𝑥 2 − 7𝑥 + 10 = 0 I.𝑦 2 − 14𝑦 + 45 = 0
(a) 16 (b) 4 (c) 8 59. I. x² – 3x = 4 II. y² + 6y + 8 = 0
(d) 24 (e) 12 60. I. x² – 3x = 10 II. y² + 7y + 10 = 0
66 www.bankersadda.com | www.sscadda.com | www.careerpower.in | Adda247 App
The IBPS RRB PO & Clerk 2021 | SUCCESS GUIDE
Directions (61-65): Following are the details of three 69. A started a business by investing Rs. 50,000. After 6
shopkeepers and numbers of items sold by them on three months B joined him by investing Rs. 75,000. After
different days another 6 months C joined with Rs. 1,25,000. What is
the ratio of profit shared after 2 years among A, B and
Shopkeepers Monday Tuesday Wednesday
C?
A 160 240 210
(a) 4 : 5 : 6 (b) 8 : 9 : 10 (c) 8 : 9 : 12
B 200 180 320
(d) 4 : 5 : 8 (e) None of these
C 150 330 280
61. Find the ratio of items sold by A and B on Monday to 70. At what rate will a sum of Rs. 1000 amounts to Rs.
items sold by B and C on Wednesday? 1102.50 in 2 years at compound interest?
(a) 5 : 3 (b) 3 : 5 (c) 3 : 4 (a) 6.5% (b) 6% (c) 5%
(d) 4 : 7 (e) 5 : 8 (d) 5.5% (e) None of these

62. Find the average number of items sold by all 3 Directions (71-80): What should come in place of
shopkeepers on Wednesday? question mark (?) in the following questions?
(a) 280 (b) 290 (c) 270 71. ?² = 40% of 11 of 352
5

(d) 250 (e) 260


(a) 12 (b) 16 (c) 6
63. Items sold by A and B together on Tuesday is what (d) 4 (e) 8
percentage of items sold by B and C on Wednesday?
(a) 70% (b) 75% (c) 60% 72. ?2 = (√1444 + √676) ÷ 4
(d) 65% (e) 80% (a) 6 (b) 16 (c) 8
(d) 2 (e) 4
64. Find the difference of number of items sold by B on
?−0.5 120
Monday and Tuesday together and items sold by A on 73. ( 0.2
)= 2
Tuesday and Wednesday? (a) 30 (b) 12.5 (c) 25
(a) 80 (b) 60 (c) 50 (d)17.5 (e) 22.5
(d) 70 (e) 100
74. 60% 𝑜𝑓 ? −√324 = 222
65. Find the ratio of items sold by B on all 3 days together (a) 600 (b) 250 (c) 200
to the items sold by C on all 3 days? (d) 400 (e) 350
(a) 35 : 38 (b) 38 : 35 (c) 30 : 34
23 ×32
(d) 30 : 38 (e) 35 : 41 75. = √64
(90÷?)
66. Marked price of an article is Rs 250 more than cost (a) 15 (b) 12 (c) 10
price of that article and it is sold at a discount of 15% (d) 11 (e) 16
on marked price. Find the cost price of the article if the 160
profit percent earned is 27.5%? 76. √4 ×? = 10
(a) Rs 600 (b) Rs 550 (c) Rs 500 (a) 64 (b) 60 (c) 68
(d) Rs 750 (e) Rs 900 (d) 56 (e) 72
67. In year 2016, ratio of boys to girls in a school is 36:19. 77. √5929 + √8464 = (? )2
And in year 2017, number of boys is increased by (a) 17 (b) 21 (c) 15
1440 and number of girls is increased by 15%. If in (d) 13 (e) 11
2017, there were total increase in the number of 1 1 50
students is 1725 then find the increased number of 78. 7 2 − 2 2 = ?
boys in the school. (a) 8 (b) 5 (c) 15
(a) 7240 (b) 5440 (c) 6040 (d)12 (e) 10
(d) 4440 (e) 5040 1
79. [(2 × 4) + 4] × 8 =?× 10
68. If ratio of salary of A to that of B is 1:3 and each spends
(a) 4.8 (b) 3.6 (c) 2.4
15% of his salary on house rent. Find the house rent
(d) 3.2 (e) 4.2
paid by A if remaining amount with A and B together
is Rs 42500. 80. 80% 𝑜𝑓 (1.5 × 4+? ) = 24
(a) Rs 1800 (b) Rs 1845 (c) Rs 1785 (a) 30 (b) 36 (c) 24
(d) Rs 1760 (e) Rs 1875 (d) 28 (e) 42

67 www.bankersadda.com | www.sscadda.com | www.careerpower.in | Adda247 App


The IBPS RRB PO & Clerk 2021 | SUCCESS GUIDE
Solutions

REASONING ABILITY

Directions (1-4): 21. (b);

1. (a); 2. (b) 3. (c)

4. (d);
22. (c)
Direction (5-9):

Floors Person
7 F
6 B
5 A Direction (23-27):
4 E
3 C
2 D
1 G

5. (e); 6. (c); 7. (d);


23. (d); 24. (b); 25. (c);
8. (d); 9. (b);
26. (c); 27. (e); 28. (b);
Directions (10-14):
29. (c);
10. (c); 11. (c); 12. (d);

13. (b); 14. (a) Direction (30-33):


C > D (115kg) > B > A > F > E (56kg)
Direction (15-19):
30. (d); 31. (b); 32. (d);
33. (b);

Direction (34-36):

34. (c); 35. (a) 36. (b)


Direction (37-39):

15. (c); 16. (d); 17. (b);

18. (e); 19. (a);

Directions (20-22):

20. (c);

37. (e); 38. (c) 39. (d)


40 (b);

68 www.bankersadda.com | www.sscadda.com | www.careerpower.in | Adda247 App


The IBPS RRB PO & Clerk 2021 | SUCCESS GUIDE
Quantitative Aptitude

41. (b); The wrong no. is 328 49. (b); ATQ


1×1+1=2 20𝑥 + 4 × (𝑥 − 2) = 424
2×2+1=5 𝟐4𝑥 = 432
5 × 3 + 1 = 16 𝑥 = 18
16 × 4 + 1 = 65
65 × 5 + 1 = 326 50. (e); Let present age of B and C be 12x years and 5x
326 × 6 + 1 = 1957 years respectively.
So, there should be 326 instead of 328 Then present age of A=10𝑥 years
ATQ
42. (a); The wrong no is 129 12𝑥 − 10𝑥 = 6
𝑥=3
Present age of B=36 years
So, there should be 109 instead of 129
51. (a); Average number of X type buses from school B
43. (d); The wrong no. is 80 36+44
and school C together = 2
= 40

52. (e); X type buses of school A = 48


So, there should be 82 instead of 80 X type buses of school B = 36
48−36 1
44. (e); The wrong no. is 5 Required value = X 100 = 33 %
36 3

53. (c); Average number of all the buses from school B


36+26
= = 31
2
So, there should be 4 instead of 5. 54. (b); Average number of all the buses from school A
48+38
45. (d); Upstream speed of boat=18 km/hr = = 43
2
Downstream speed of boat=22 km/hr Average number of all the buses from school C
18+22
Speed of boat in still water= = 20 𝑘𝑚/ℎ =
44+34
= 39
2
2
46. (b); Let the capacity of the tank be 180 units (LCM Required difference = 43 – 39 = 4
of 36 and 60)
55. (e); Total buses from school A = 48 + 38 = 86
Efficiency of tap A=5 units/ minute
Total buses from school B = 36 + 26 = 62
Efficiency of tap B=3 units/minute
1th Total buses from school C = 44 + 34 = 78
6
of the tank= 30 units Clearly, School A has maximum number of
30 3
Required time=5+3 = 3 4 minutes buses.
132×7
47. (a); Radius of first circle= 2×22 = 21 𝑐𝑚 56. (e)
22
Area of first circle= 7
× 21 × 21 = 1386 cm2
110×7
Radius of second circle= 2×22 = 17.5 cm
22
Area of second circle= × 17.5 × 17.5 = 962.5
7
cm2 57. (a)
Required difference=423.5 cm2
48. (e); Ratio of milk to that of water in the initial
mixture=16:5
1
4
𝑡ℎ of the mixture=21 liter 58. (d)
16
64−21× 2
21
5 =1
20−21× +𝑥
21
𝑥 = 9 liter

69 www.bankersadda.com | www.sscadda.com | www.careerpower.in | Adda247 App


The IBPS RRB PO & Clerk 2021 | SUCCESS GUIDE
59. (a); I. x² – 3x – 4=0 67. (e); Let the number of students in the exam be 55x
𝑥 2 − 4𝑥 + 𝑥 − 4 = 0 Then number of boys= 36x
(x – 4) (x + 1) = 0 Number of girls=19x
x = 4, –1 ATQ
II. y² + 6y + 8 = 0 55𝑥 + 1725 = (36𝑥 + 1440) + 19𝑥 × 1.15
y² + 2y + 4y + 8 = 0 𝑥 = 100
(y + 2) (y + 4) = 0 Increased number of boys=3600+1440=5040
y = –2, –4
68. (e); Let the salary of A and B be Rs 100x and Rs
⇒x>y
300x respectively
60. (b); I. x² – 3x = 10 ATQ
𝑥 2 − 3𝑥 − 10 = 0 85𝑥 + 255𝑥 = 42500 ⇒ 𝑥 = 125
x² – 5x + 2x – 10 = 0 House rent paid by A=Rs 1875
(x – 5) (x + 2) = 0
69. (b);
x = –2, 5
II. y² + 7y + 10 = 0
y² + 5y + 2y + 10 = 0
(y + 5) (y + 2) = 0
y = – 2, –5 Required ratio = 8:9:10
⇒x≥y 1102.50 𝑟 2
70. (c); ATQ, 1000
= (1 + 100)
61. (b); items sold by A and B on Monday = 200 + 160 𝑟 2 105 2
= 360 or, (1 + ) = (100)
100
Item sold by B and C on Wednesday = 320 + 280 𝑟 2 5 2
= 600 or, (1 + ) = (1 + )
100 100
∴ Required ratio =
360 6
= =
3 Thus, on comparing, 𝑟 = 5%
600 10 5
5
62. (c); Average of items sold by A, B, C on Wednesday 71. (e); ?2 = 40% of × 352
11
210+320+280 810 2 2 5
= 3
= 3 = 270 ? = ×
5 11
× 352 = 64 ⇒ ? = 8

63. (a); items sold by A and B on Tuesday 72. (e); ?2 =


(√1444+√676)
=
38+26
=
64
=16 ⇒? = 4
= 240 + 180 = 420 4 4 4

Items sold by B and C on Wednesday 73. (b); (? −0.5) = 60 × 0.2


= 320 + 280 = 600 ? = 12 + 0.5 = 12.5
420×100
∴ Required percentage = 600 = 70% 60
74. (d); ×? −18 = 222
100
64. (d); items sold by B on Monday and Tuesday = 200 + 60
×? = 240
180 = 380 100
240×100
Items sold by A on Tuesday and Wednesday = ?= ⇒ ? = 400
60
240 + 210 = 450 8×9×?
∴ Required difference = 450 – 380 =70 75. (c); =8
90
90×8
65 (a); Item sold by B on all 3 days = 200 +180 + 320 ?= 8×9
= 10 ⇒ ? = 10
= 700
76. (a); √4 ×? = 16
Items sold by C on all 3 days = 150 + 330 + 280
4 × ? = 256
= 760
700 35 ? = 64
Required ratio = 760 = 38
77. (d); 77 + 92 = ?2
66. (c); Let the marked price be Rs 100x 169 = ?² ⇒ ? = 13
Then selling price= Rs 85x 50
200 78. (e); 5 = ⇒? = 10
Cost price=𝑅𝑠 3 𝑥 ?
9
ATQ 79. (b); × 8 =?× 10 ⇒? = 3.6
200 2
100𝑥 − 3 𝑥 = 250 80
𝑥 = 7.5 80. (c); × (6+? ) = 24
100
Cost price=Rs 500 6+? = 30 ⇒ ? = 24

70 www.bankersadda.com | www.sscadda.com | www.careerpower.in | Adda247 App


The IBPS RRB PO & Clerk 2021 | SUCCESS GUIDE

IBPS RRB CLERK PRELIMS MEMORY BASED PAPER 2018


REASONING ABILITY

Directions (1-5): Read the following information carefully 7. Statements: Some logic are answers.
and answer the questions given below. All keys are answers.
Six persons J, P, Q, R, V, Z are sitting in a row. Some of them Conclusions: I. All keys are logic.
are facing north while some of them are facing south. J sits II. No keys are logic.
second from one of the extreme end of the row. P sits third (a) Only I follows
to the right of J. R is not an immediate neighbor of P and Z. (b) Only II follows
Both the immediate neighbors of V faces opposite (c) Either I or II follows
direction. Both the Immediate neighbors of Z faces same (d) Neither I nor II follows
direction. V sits second to the left of P. Q sits to the right of (e) Both I and II follow
R. R faces north. Q faces same direction as Z.
8. Statement: All numbers are digits.
1. Four of the following five are alike in a certain way, and
Some numbers are points.
so form a group. Which of the following does not
Some points are marks.
belong to the group?
Conclusions: I. Some points are digits.
(a) R, V (b) V, P (c) J, P
(d) V, Q (e) J, R II. All marks being numbers is a
possibility.
2. What is the position of Q with respect to Z? (a) Only I follows
(a) Second to the left (b) Third to the right (b) Only II follows
(c) Third to the left (d) Fifth to the right (c) Either I or II follows
(e) Second to the right (d) Neither I nor II follows
3. Who amongst the following sits exactly between Z and (e) Both I and II follow
J?
Directions (9-13): Read the following information
(a) R (b) P (c) Q
carefully and answer the questions given below.
(d) Both V and Q (e) V
Seven boxes M, N, O, P, Q, R, S are arranged one above
4. How many persons in the given arrangement are another. Only two boxes are placed above box P. Only one
facing North? box is placed between box S and P. As many as boxes are
(a) More than four (b) Four placed between box S and Q as between box Q and M. Three
(c) One (d) Three (e) Two boxes are placed between box N and O. N is placed above
5. Who is sitting 4th to the right of Q? O.
(a) R (b) Z (c) P 9. How many total numbers of boxes are placed in
(d) J (e) None of these
between box S and Q?
Directions (6-8): Some statements are given followed by (a) Two (b) One (c) Three
two conclusions. You have to consider the statements to be (d) More than three (e) None
true even if they seem to be at variance from commonly
known facts. You have to decide which of the following
conclusions follow from the given statements:
6. Statements: No symbol is letter.
All expression are letter.
Some symbols are word.
Conclusions: I. No word is letter.
II. Some symbols being expression is
possibility.
(a) Only I follows
(b) Only II follows
(c) Either I or II follows
(d) Neither I nor II follows
(e) Both I and II follow

71 www.bankersadda.com | www.sscadda.com | www.careerpower.in | Adda247 App


The IBPS RRB PO & Clerk 2021 | SUCCESS GUIDE
10. Which of the following is true regarding Box N? 20. How many persons were born between A and Q?
(a) Three boxes are placed between box Q and N (a) One (b) Three (c) Four
(b) Box N is placed below Q (d) Two (e) None of these
(c) Box N is placed at top 21. How many persons born were before R?
(d) Only one box is placed above box N (a) One (b) Three (c) Four
(e) No box is placed between box N and R (d) Two (e) None of these
11. Which box is placed at top? 22. Who among the following is the oldest?
(a) S (b) N (c) Q (a) C (b) A (c) Q
(d) R (e) M (d) T (e) Y
12. Which box is placed immediately above box Q? 23. Which of the following is not true regarding Y?
(a) M (b) P (c) S (a) Four persons born between C and Y
(d) N (e) R (b) R was born before Y
(c) Q is born immediately after Y
13. How many boxes are placed in between R and M? (d) Only Q was born between Y and R
(a) Two (b) One (c) Three (e) No one was born after Y
(d) More than three (e) None
24. A family consists of five members A, P, R, T, H. P is wife
Directions (14-18): Answer these questions based on the of A. R is the daughter of A. R has only one brother T. H
following information. is daughter-in-law of P. How is H related to R?
In a certain code: (a) mother (b) sister-in-law
“arrange things in order” is coded as - “po gb ik mn” (c) daughter (d) daughter-in-law
“order for new things” is coded as - “po gb fc bv” (e) none of these
“new places to order” is coded as - “gb cq bv ra”
“places in unknown country” is coded as – “ de ra lf ik” 25. If the digits of the number “46752983” are arranged in
increasing order form left to right within the number,
14. What will be the code for “order”? then how many digits will remain on the same position
(a) gb (b) fc (c) cq after the applied operation?
(d) ik (e) can’t be determined (a) Two (b) One (c) Three
15. What may be the code for “things to vanish”? (d) Four (e) None of these
(a) po cq hx (b) po vm ik 26. How many meaningful words can be made by using
(c) cq fc ik (d) either (a) or (b) letters ‘A’, ‘E’, ‘L’ and ‘T’, keeping L as the first letter of
(e) None of these the word?
16. What will be the code for “ arrange”? (a) One (b) Two (c) Three
(a) gb (b) mn (c) cq (d) Four (e) None of these
(d) ik (e) can’t be determined Directions (27-31): Read the following information
17. What may be the code for “ in country”? carefully and answer the questions given below.
(a) lf ik Point E is 15m east of point B. Point G is 20m north of point
(b) de ik E. Point K is 10m east of point G. Point M is 30m south of
(c) po gb point K. Point P is 20m west of point M. Point L is 10m
(d) either (a) or (b) north of point P.
(e) None of these 27. If Point V is 10m east of point S and Point S is 10m
18. “bv” is the code for? north of point L, then what will be the distance
(a) things (b) new (c) arrange between point E and V?
(d) places (e) None of these (a) 10m (b) 15m (c) 20m
(d) 5m (e) 25m
Directions (19-23): Read the following information
carefully and answer the questions given below. 28. What is the total distance between point B and L?
Six persons A, C, Q, R, T, Y were born in six different months (a) 10m (b) 15m (c) 20m
January, April, May, August, September, December of a (d) 5m (e) 30m
year. Three persons were born in between A and Y. A was 29. If Point Z is 10m north of point M, then point what is
born before Y. No one was born in between C and A. Two the distance between point E and Z?
persons were born in between C and R. T was born before (a) 10m (b) 15m (c) 20m
Q. (d) 25m (e) 30m
19. Who among the following was born in May? 30. Point K is in which direction from point P?
(a) C (b) A (c) Q (a) South (b) South-east (c) North
(d) T (e) Y (d) North-east (e) North-west
72 www.bankersadda.com | www.sscadda.com | www.careerpower.in | Adda247 App
The IBPS RRB PO & Clerk 2021 | SUCCESS GUIDE
31. Four of the following five are alike in a certain way, and 36. Which of the following element is 5th to the right of 10th
so form a group. Which of the following does not from the right end?
belong to the group? (a) 9 (b) 8 (c) 2
(a) P, L (b) P, M (c) G, E (d) 1 (e) 4
(d) L, E (e) G, B
Directions (37-40): Read the following information
Directions (32-36): These questions are based on the carefully and answer the questions given below.
following arrangement. Study it carefully and answer the There are six wallets A, B, C, P, Q and R, each containing
questions below it. different amount of money in it. Wallet B has more money
135345 92872365273812184981247352 than wallet Q but less than wallet P. Only wallet R has more
489824 money than wallet C. Wallet Q does not has the least
amount of money. The wallet containing 3rd highest
32. Which element is exactly midway between the seventh
amount of money has Rs. 3000, which is Rs.1000 more than
element from the left end and sixteenth from the right
the wallet which has 2nd lowest amount of money.
end?
(a) 8 (b) 2 (c) 5 37. Which of the following wallet has the least amount of
(d) 6 (e) 7 money?
(a) A (b) B (c) C
33. How many perfect squares are there to the right of the
(d) Q (e) P
fourteenth element from the right end?
(a) Two (b) One (c) Three 38. What may be the amount of money in wallet C?
(d) Four (e) more than four (a) Rs. 2500 (b) Rs. 2000 (c) Rs. 3500
(d) Rs. 2250 (e) Rs. 2100
34. How many perfect cubes are there in the above
arrangement, each of which is immediately preceded 39. What may be the amount of money in wallet B, if it has
by an odd number and immediately followed by an Rs. 250 less than the wallet P?
even number? (a) Rs. 2500 (b) Rs. 2750 (c) Rs. 3500
(a) None (b) Three (c) Two (d) Rs. 3250 (e) Rs. 2200
(d) One (e) More than three
40. Which of the following is true regarding wallet P?
35. How many such odd digits are there in the given (a) Only wallet A has less money than wallet P
arrangement, each of which is immediately followed (b) Wallet B has more money than wallet P
and preceded by an odd number? (c) Wallet P has 3rd highest amount of money
(a) None (b) One (c) Two (d) Wallet Q has more amount of money than P
(d) Three (e) More than three (e) none of these

QUANTITATIVE APTITUDE
41. The upstream speed of a boat is 18 km/hr which is 44. A train having speed of 72 km/hr crosses a pole in 18
500% more than the speed of stream. Find how much sec and a platform in 33 sec. Find the length of
distance boat will cover in 3 hours while travelling in platform?
downstream. (a) 320 m (b) 300 m (c) 330 m
(a) 66 km (b) 63 km (c) 72 km (d) 360 m (e) 350 m
(d) 75 km (e) 78 km
45. The circumference of a circle is 66 cm. Find the
42. If 𝐴2 – 𝐵2 = 252 and 𝐴 + 𝐵 = 42 then find the value of approximate area of square if the radius of circle is two
‘B’? times of the side of a square.
(a) 18 (b) 16 (c) 14 (a) 18 cm² (b) 32 cm² (c) 25 cm²
(d) 20 (e) 22 (d) 36 cm² (e) 28 cm²
43. A alone can do a work in 40 days. The ratio of time
Directions (46-50): What approximate value should come
taken by A and B to do the same work is 5 : 3. Then, find
in place of question mark (?) in the following questions?
in how many days both will complete the work
together ? 46. √1443.98 ÷ 18.98 + 328.1 =?× 22.01
(a) 18 days (b) 12 days (c) 20 days (a) 10 (b) 12 (c) 18
(d) 15 days (e) 10 days (d) 15 (e) 22

73 www.bankersadda.com | www.sscadda.com | www.careerpower.in | Adda247 App


The IBPS RRB PO & Clerk 2021 | SUCCESS GUIDE
47. 29.98% of 880.001 = ? + 110.9 58. Number of employees in Housing department is how
(a) 144 (b) 153 (c) 158 much more than number of employees in Security
(d) 160 (e) 163 department?
(a) 10 (b) 20 (c) 30
48. (?)² + 255.93 = 49.932% of 800.112 (d) 40 (e) 50
(a) 12 (b) 8 (c) 15
59. In Security department, 40% are female employees
(d) 18 (e) 6
3 then find total male employees working in Security
49. √1728.01+? = 256.01 department?
(a) 230 (b) 235 (c) 238 (a) 16 (b) 40 (c) 32
(d) 241 (e) 244 (d) 8 (e) 24
50. 74.91% of ? = (17.932)² 60. Ratio between total number of male and female
(a) 420 (b) 425 (c) 408 employees in HR department is 2 : 3. Find total number
(d) 432 (e) 444 of female employees working in HR department?
(a) 32 (b) 48 (c) 64
Directions (51-55):-Find the wrong number in the given (d) 40 (e) 56
number series questions.
Directions (61-70): What value should come in place of
51. 100, 118, 136, 149, 160, 167, 172 question mark (?) in the following questions?
(a) 172 (b) 160 (c) 100
61. ?2 = 42 + 82 − 31
(d) 118 (e) 136 (a) 6 (b) 7 (c) 8
52. 1.5, 2.5, 6, 24, 100, 505, 3036 (d) 9 (e) 10
(a) 1.5 (b) 6 (c) 100 62. 13 × 6 +? × 4 = 18 × 7
(d) 3036 (e) 2.5 (a) 6 (b) 8 (c) 10
(d) 12 (e) 14
53. 160, 80, 80, 120, 240, 600, 900
(a) 240 (b) 120 (c) 160 63. 40% 𝑜𝑓 ? = 25% 𝑜𝑓 320 + 75% 𝑜𝑓 160
(d) 900 (e) 600 (a) 500 (b) 400 (c) 300
(d) 200 (e) 100
54. 5040, 2520, 840, 210, 42, 8, 1
(a) 8 (b) 5040 (c) 840 64. 112 + 62 = ? +37
(a) 130 (b) 110 (c) 120
(d) 1 (e) 42
(d) 140 (e) 150
360
55. 15, 17, 26, 151, 200, 929, 1050 65. ? = 12 × 6 − 33
(a) 17 (b) 1050 (c) 15 (a) 9 (b) 5 (c) 6
(d) 929 (e) 26 (d) 7 (e) 8
Direction (56-60): There are total five departments in a 66. √225 + √441 = ?2
company. There are total 90 employees in Finance (a) 3 (b) 4 (c) 5
department which is 25% of total employees in the (d) 6 (e) 8
company. 2/9 of the total employees of the company are
working in HR department. Employees working in Sales 67. 16 × 8−? = 26
department is 25% more than that in HR department. Ratio (a) 64 (b) 32 (c) 128
(d) 192 (e) 96
between employees working in Security and Housing
department is 4 : 5. 68. 16 × 54 ÷ 36 + 6 =?
144
56. Find number of employees working in HR department (a) (b) 30 (c) 20
7
is what percent more than number of employees (d) 24 (e) 16
working in Security department?
69. ? = √6 × 3 × 5 + 50% 𝑜𝑓 620
(a) 250% (b) 200% (c) 150%
(a) 14 (b) 16 (c) 18
(d) 100% (e) 50%
(d) 10 (e) 20
57. Find the average number of employees working in 18×8−?×2
Sales, Finance and Housing department? 70. 62 = 3
(a) 60 (b) 70 (c) 80 (a) 36 (b) 27 (c) 18
(d) 90 (e) 100 (d) 9 (e) 54

74 www.bankersadda.com | www.sscadda.com | www.careerpower.in | Adda247 App


The IBPS RRB PO & Clerk 2021 | SUCCESS GUIDE
Direction (71-75): - Table given below shows marks (a) 64.25% (b) 61.25% (c) 67.25%
obtained by four students in four different subjects in an (d) 70.25% (e) 73.25%
exam. Study the data carefully and answer the following
76. An article was sold at a discount of 20% at Rs. 1020. If
questions
the article was sold at discount of Rs. 199 in place of
Subjects
English Hindi Science Maths 20% discount then find the selling price.
Students (a) Rs. 1066 (b) Rs. 1076 (c) Rs. 1086
Paul 65 60 80 65 (d) Rs. 1096 (e) Rs. 1094
Aditya 75 75 60 75
Neeraj 85 55 95 85 77. The total age of A, B and C four years hence will be 98
Sandy 60 60 65 60 years. Find the age of C four years hence if the present
71. Marks scored by Sandy in English and Maths together age of A and B is 32 years and 23 years respectively.
is what percent of the Marks scored by Aditya and (a) 31 yr. (b) 32 yr. (c) 35 yr.
Neeraj in English together? (d) 37 yr. (e) 33 yr.
(a) 25% (b) 50% (c) 75% 78. A invests Rs. 12,000 for X months while B invests Rs.
(d) 100% (e) 125% 16,000 for 9 months in a scheme. The profit share of B
72. Find the ratio of total marks scored by all four students is Rs. 12,000 out of total profit Rs. 21,000. Then find the
together in Hindi to total marks scored by all four value of X ?
students together in Science? (a) 6 months (b) 9 months (c) 8 months
(a) 5 : 6 (b) 57 : 50 (c) 1 : 1 (d) 7 months (e) 10 months
(d) 20 : 19 (e) 6 : 5 79. A mixture of milk and water contains 60% milk and
73. Total marks scored by Paul are how much more/less remaining water. How much water should be added (in
than total marks scored by Neeraj? percentage) in mixture to reverse the proportion of
(a) 70 (b) 60 (c) 40 milk and water?
(d) 50 (e) 80 (a) 25% (b) 37.5% (c) 62.5%
(d) 75% (e) 50%
74. Find the average of the marks scored by Aditya in
English, Hindi and Science together? 80. The simple interest on a certain sum for 2 years at 8%
(a) 65 (b) 85 (c) 80 per annum is Rs. 225 less than the compound interest
(d) 75 (e) 70 on the same sum for 2 years at 10% per annum. The
sum is:
75. If maximum marks for each subject are 100 then find (a) Rs. 3200 (b) Rs. 4200 (c) Rs. 4000
what percentage of total marks is obtained by Sandy? (d) Rs. 3600 (e) Rs. 4500

Solutions

REASONING ABILITY

Direction (1-5): 7. (d);

1. (d) 2. (a) 3. (e)

4. (b) 5. (d)
8. (e);
Directions (6-8):

6. (d);

75 www.bankersadda.com | www.sscadda.com | www.careerpower.in | Adda247 App


The IBPS RRB PO & Clerk 2021 | SUCCESS GUIDE
Direction (9-13): 19. (d) 20. (d) 21. (b)
Boxes 22. (a) 23. (c)
S
N 24. (b)
P
Q
R
O 25. (a)
M
9. (a) 10. (d) 11. (a)
12. (b) 13. (b) 26. (b);LATE, LEAT
Directions (14-18): Direction (27-31):
Word Code
Places ra
Order gb
New bv
To cq
Things po
For fc
In ik
Arrange mn 27. (a) 28. (d) 29. (a)
Unknown/country de/lf
30. (d) 31. (e) 32. (b);
14. (a) 15. (a) 16. (b) 33. (e);
17. (d) 18. (b) 34. (c); 184,982
Direction (19-23):
35. (d); 735,135,353
Months Persons
36. (b);
January C
April A Directions (37-40):
May T
R > C > P (Rs. 3000) > B > Q (Rs.2000) > A
August R
September Q 37. (a); 38. (c); 39. (b);
December Y 40. (c);

QUANTITATIVE APTITUDE
41. (c); Let the speed of stream be 𝑥 km/hr And 𝐴 + 𝐵 = 42 …(ii)
Then, Solve (i) and (ii), we get
600 𝐵 = 18
Speed of upstream = 𝑥 × = 18
100
⇒ 𝑥 = 3km/hr 43. (d);Let the time taken by A and B be 5𝑥 days and 3𝑥
Speed of boat in still water = 18 + 3 = 21 km/hr days respectively.
Distance covered in 3 hours in downstream = (21 ⇒ 5𝑥 = 40 days
+ 3) × 3 = 72 km ⇒ 𝑥 = 8 days
42. (a); (𝐴 + 𝐵)(𝐴 – 𝐵) = 252 B’s time = 3 × 8 = 24 days
⇒ 42 × (𝐴– 𝐵) = 252 [𝐴 + 𝐵 = 42 𝑔𝑖𝑣𝑒𝑛] Time taken by both together to complete the work
40×24 𝑎×𝑏
⇒ (𝐴– 𝐵) = 6 …(i) = [𝑢𝑠𝑒 for two persons] = 15 days.
40+24 𝑎+𝑏

76 www.bankersadda.com | www.sscadda.com | www.careerpower.in | Adda247 App


The IBPS RRB PO & Clerk 2021 | SUCCESS GUIDE
44. (b);Speed of train = 72 km/hr Solution (56-60): Let total employees in company be 100x
5
= 72 × = 20 m/s ATQ,
18 25
Length of train = 18 × 20 = 360 m 100
× 100𝑥 = 90
Length of (train + platform) ⇒ Total employees in company = 100𝑥 = 360
= 20 × 33 = 660 m Employees working in HR department
2
∴ length of platform = 660 m – 360 m = 300 m = 9 × 360 = 80
45. (e); ATQ, Employees working in Sales department
125
2πr = 66 cm = × 80 = 100
22 100
⇒ 2 × × 𝑟 = 66 cm Remaining employees = 360 − 90 − 80 − 100 = 90
7
⇒𝑟=
66×7 21
= cm Employees working in Security department
44 2 4
21 21 = 9 × 90 = 40
Side of a square = 2×2 = 4 cm
Employees working in Housing department
21 2 5
∴ Area of square = (side)² = ( ) = 9 × 90 = 50
4
441
= 16
≈ 28 𝑐𝑚2

46. (d);√1444 ÷ 19 + 328 = ? × 22 80−40 40


⇒ 2 + 328 = ? × 22 . 56. (d);Required % = × 100 = ×100 = 100%
40 40
330
⇒ ? = 22 = 15 100+90+50 240
57. (c); Required average = = = 80
3 3
47. (b);30% of 880 = ? + 111
30×880 58. (a); Required difference = 50 − 40 = 10
⇒ =? +111
100
59. (e); Total number of male employees working in
⇒ ? = 264 – 111 = 153. 60
50×800 Security department = 100 × 40 = 24
48. (a); (? )2 + 256 =
100
(? )2 + 256 = 400 60. (b);Total number of female employees working in HR
3
⇒ (? )2 = 144 ⇒? = 12 department = 5 × 80 = 48
49. (e); 12 + ? = 256 61. (b); ?2 = 42 + 82 − 31
⇒ ? = 244 ?2 = 16 + 64 − 31 = 80 − 31 = 49
50. (d);
75×?
= (18)2 ⇒
75×?
= 324 ?= 7
100 100
⇒?=
324×100
= 432. 62. (d); 13 × 6 +? × 4 = 18 × 7
75 78 +? × 4 = 126
126−78
51. (d); ? = 4 = 12

63. (a); 40% 𝑜𝑓 ? = 25% 𝑜𝑓 320 + 75% 𝑜𝑓 160


2 25 75
×? = × 320 + × 160
5 100 100
52. (b); 2
5
×? = 80 + 120
5
? = 200 × = 500
2

53. (d); 64. (c); 112 + 62 =? +37


121 + 36 − 37 =? ⇒ ? = 120
360
65. (e); ?
= 12 × 6 − 33
54. (a); 360
= 72 − 27
?
360
?= 45
=8

55. (c); 66. (d);√225 + √441 = ?2


15 + 21 =?2
?2 = 36 ⇒ ? = 6
67. (a); 16 × 8−? = 26
128 − 64 =? ⇒ ? = 64

77 www.bankersadda.com | www.sscadda.com | www.careerpower.in | Adda247 App


The IBPS RRB PO & Clerk 2021 | SUCCESS GUIDE
68. (b);16 × 54 ÷ 36 + 6 =? 77. (c); Sum of present age of A, B and C
54
? = 16 × + 6 = 30 = 98 – 4 × 3
36
= 98 – 12 = 86 yr.
69. (e); ? = √6 × 3 × 5 + 50% 𝑜𝑓 620 Present age of C = 86 – (32 + 23) = 31 yr.
? = √90 + 310 = √400 = 20 Age of C four years hence = 31 + 4 = 35 yr.
78. (b);Profit share ratio of
18×8−?×2
70. (c); 62 = 3
𝐴 𝐵
36 × 3 = 144−?× 2 12000 × 𝑥 16000 × 9
?× 2 = 144 − 108 𝑥 : 12
36 ATQ,
? = 2 = 18 71. (c); Marks scored by Sandy in 𝑥 9000
= 12000
English and Maths together = 60 + 60 = 120 12

Marks scored by Aditya and Neeraj in English ⇒ 𝑥 = 9 months.


together = 75 + 85 = 160 79. (e); Let, total quantity = 100ℓ
120
Required % = 160 × 100 = 75% Quantity of milk = 60 ℓ
And quantity of water = 40 ℓ
60+75+55+60 250 5
72. (a); Required ratio = 80+60+95+65 = 300 = 6 ATQ,
40 60
=
73. (d);Total marks scored by Paul = 65 + 60 + 80 + 65 = 100 100+𝑥
270 2 (100 + x) = 5 × 60
Total marks scored by Neeraj = 85 + 55 + 95 + 200 + 2x = 300
85 = 320 2x = 100
Required difference = 320 − 270 = 50 x = 50 ℓ
50
75+75+60
𝑊ater added in % = 100 × 100 = 50%
74. (e); Required average = 3
= 70
80. (e); 𝐿𝑒𝑡 𝑡ℎ𝑒 𝑠𝑢𝑚 𝑏𝑒 𝑅𝑠. 𝑃.
60+60+65+60
75. (b);𝑅𝑒𝑞𝑢𝑖𝑟𝑒𝑑 % = 400
× 100 = 61.25% 11 11 𝑃×2×8
𝑃 [10 × 10 − 1] – 100 = 225
1020 21 16𝑃
76. (b);MP of article = 80 × 100 = Rs. 1275 ⇒ 𝑃[ ]– = 225
100 100
225×100
Selling price = 1275 – 199 = Rs. 1076 ⇒𝑃= ⇒ P=Rs. 4500
5

78 www.bankersadda.com | www.sscadda.com | www.careerpower.in | Adda247 App


The IBPS RRB PO & Clerk 2021 | SUCCESS GUIDE

IBPS RRB CLERK PRELIMS MEMORY BASED PAPER 2017


REASONING ABILITY

Directions (1-5): In each of the question, relationships 9. Who among the following lives on Top floor?
between some elements are shown in the statements(s). (a) N (b) J (c) K
These statements are followed by conclusions numbered I (d) M (e) None of these
and II. Read the statements and give the answer.
10. Which of the following combination is false?
(a) If only conclusion I follows.
(a) J-7 (b) L-3 (c) G-2
(b) If only conclusion II follows.
(d) H-4 (e) N-1
(c) If either conclusion I or II follows.
(d) If neither conclusion I nor II follows. 11. In a row of children facing North, Rajan is twelfth from
(e) If both conclusions I and II follow. the right end and is fifth to the right of Satyarthi who is
tenth from the left end. How many total number of
1. Statements: A < B >N = M, B ≤ V, M > R
children are there in the row?
Conclusions: I. B > R II. V > A
(a) 29 (b) 28 (c) 26
2. Statements: D <E >F = G > H = I ≤ J (d) 27 (e) None of these
Conclusions: I. F > I II. J ≥ E
12. Raj leaves his home and goes straight 20 meters, then
3. Statements: M < N < O > P, N<E turns right and goes 10 meters. He turns left and goes
Conclusions: I. E < M II. E>O 30 meters and finally turns right and starts walking. If
he is now moving in the north direction, then in which
4. Statements: C ≥ D< E = F ≥ G, C < W
direction did he start his walking?
Conclusions: I. E =G II. G< E
(a) East (b) West (c) North
5. Statements: R < T < S < P > Q, R> X (d) South (e) None of these
Conclusions: I. S < Q II. X < S
Directions (13-17): In each of the questions given below,
Direction (6-10): Study the following information a group of digits/letter is given followed by four
carefully and answer the question given below- combinations of symbols numbered (a), (b), (c) and (d).
Eight people viz. G, H, I, J, K, L, M and N lives in a Building You have to find out which of the four combinations
on different floors from top to bottom (such as ground correctly represents the group of digits/letters based on
floor numbered as 1 and top is numbered as 8) but not the symbol codes and the conditions given below. If none
necessarily in the same order. of the four combinations represents the group of digits
There is a gap of three floors between J and L and both of correctly, give (e) ie ‘None of these’ as the answer.
them lives on odd number of floor. N lives just above H,
who lives on even numbered floor. I lives on floor number
6.Only one person lives between L and M. J lives above I.
Three persons live between K and H. Condition for coding the group elements:
(i) If the first letter is Vowel and the last digit is divisible
6. Who among the following lives on ground floor?
by 2, then both are to be coded as +.
(a) N (b) J (c) K
(ii) If the first as well as the last digit is odd, then both are
(d) M (e) None of these
to be coded by the code of the first digit.
7. Who among the following lives immediately below L? (iii) If the first letter is consonant and the last digit is odd
(a) K (b) I (c) G number, then the code of first and last elements are to
(d) H (e) None of these be interchanged.

8. How many persons lives between I and H? 13. WX6ZF1


(a) One (b) Three (c) Fives (a) ^®$@£∞ (b) ^@$∞<! (c) ^®£@$∞
(d) Two (e) None of these (d) ∞®@>!< (e) None of these

79 www.bankersadda.com | www.sscadda.com | www.careerpower.in | Adda247 App


The IBPS RRB PO & Clerk 2021 | SUCCESS GUIDE
14. FE1XI6 25. If all the odd numbers are dropped from the series,
(a) ∞^@<!£ (b) $<^^£@ (c) $>^®µ£ which number will be eighth to the left of eleventh
(d) $<^@^£ (e) None of these number from the left end?
(a) 2 (b) 8 (c) 6
15. 5L2IA1
(d) 4 (e) None of these
(a) ∆!<µ&∆ (b) ∆!&^<µ (c) ∆!<µ^&
(d) µ&∆!<^ (e) None of these 26. If 1 is subtracted from all odd numbers and 2 is
16. E2ZA6 subtracted from all even numbers in the given number
(a) &>!^@ (b) @<@&! (c) @&<@& series, then which number will be sixteenth from the
(d) +<@&+ (e) None of these right end?
(a) 0 (b) 2 (c) 3
17. IZ2W2 (d) 8 (e) 6
(a) @≠^$& (b) +@<∞+ (c) <∞µ@≠
(d) @≠>!^ (e) None of these 27. If the position of the 1st and the 16th numbers, the 2nd
and the 17th numbers, and so on up to the 15th and the
Directions (18-22): Read the following information 30th numbers, are interchanged, which number will be
carefully and answer the questions given below. 7th to the right of 19th number from the right end?
A, B, C, D, E, F, G and H are eight members standing in a row (a) 5 (b) 9 (c) 8
(not necessarily in the same order) facing north. (d) 4 (e) None of these
C and B have as many members between them as G and C
have between them. D, who is 4th from the extreme left end, 28. How many total even numbers which is immediately
is 2nd to the left of E. G is 3rd place away from one of the preceded by a ‘whole cube’ or ‘immediately preceded
extreme end. Neither B nor C sits any extreme end. F sits by a whole square’ in the above sequence?
immediate right of A. (a) Four (b) Five (c) Three
(d) Six (e) None of these
18. How many persons sit between G and B?
(a) One (b) Three (c) Two Directions (29–33): In each question below are given
(d) Four (e) None of these some statements followed by two conclusions numbered I
and II. You have to take the given statements to be true
19. Who among the following persons sits at extreme even if they seem to be at variance with commonly known
ends? facts. Read all the conclusions and then decide which of the
(a) A, G (b) B, C (c) F, H given conclusions logically follows/follow from the given
(d) H, A (e) None of these statements, disregarding commonly known facts. Give
20. Who sits second to the right of E? answer
(a) B (b) H (c) G (a) If only conclusion I follows.
(d) C (e) None of these (b) If only conclusion II follows.
(c) If either conclusion I or II follows.
21. Who sits third to the left of G? (d) If neither conclusion I nor II follows.
(a) A (b) None (c) F (e) If both conclusions I and II follow.
(d) E (e) B
29. Statements: All shirts are skirts.
22. Who sits immediate left of C? No skirt is top. All tops are kurta.
(a) A (b) H (c) C Conclusions: I. All shirts are kurta
(d) D (e) None of these
II. Some kurta are skirts.
23. Find the odd one out?
30. Statements: Some chocolate are chips.
(a) ACB (b) DFE (c) GIH
Some chips are jelly.
(d) JLK (e) MNO
All jelly are whoppers.
Directions (24-28): Study the following number sequence Conclusions: I. Some jelly are chips.
and answer the questions following it. II. All chocolate being whoppers is a
9324579581506429826359821543 possibility
21
31. Statements: Some frooti are Maaza.
24. How many odd numbers are there in the numeric No Maaza is slice.
series which are immediately preceded by a number, All slice are fanta.
which is a whole square? Conclusions: I. Some frooti are definitely not slice.
(a) One (b) Two (c) Three II. Some fanta are definitely not
(d) More than three (e) None of these Maaza.

80 www.bankersadda.com | www.sscadda.com | www.careerpower.in | Adda247 App


The IBPS RRB PO & Clerk 2021 | SUCCESS GUIDE
32. Statements: All carbon are oxygen. 36. If the third alphabet in each of the words is changed to
All Nitrogen are carbon. the previous alphabet in the English alphabetical
Some oxygen are Sulphur. order, how many words thus formed will be without
Conclusions I. All Nitrogen being Sulphur is a any vowels?
possibility. (a) None (b) One (c) Two
II. All Nitrogen are not oxygen. (d) Three (e) More than three
33. Statements: All September are October. 37. If the position of the first and the third alphabet of each
No October is November. of the words are interchanged, which of the following
No November is December. will form a meaningful word in the new arrangement?
Conclusions: I. Some September are not (a) NOW (b) SAD (c) RAT
Novembers (d) WAF (e) Both (a) and (c)
II. No October is December.
38. If in each of the given words, each of the consonants is
Directions (34-38): Following questions are based on the changed to its previous letter and each vowel is
five words given below, Study the following words and changed to its next letter in the English alphabetical
answer the following questions. series, then how many words thus formed will at least
NOW SAD WAF RAT CAT one vowel appear?
(The new words formed after performing the mentioned (a) None (b) One (c) Two
operations may not necessarily be a meaningful English (d) Three (e) None of these
word.) 39. If in the number 9737132710, positions of the first and
34. If the given words are arranged in the order as they the second digits are interchanged, positions of the
appear in a dictionary from left to right, which of the third and fourth digits are interchanged and so on till
following will be the fourth from the left end? the positions of 9th and 10th digits are interchanged,
(a) WAF (b) NOW (c) SAD then which digit will be 6th from the left end?
(d) CAT (e) RAT (a) 7 (b) 1 (c) 3
(d) 9 (e) None of these
35. How many letters are there in the English alphabetical
series between the second letter of the word which is 40. How many pairs of letters are there in the word”
second from the right end and the third letter of the WORSHIP” which have as many letters between them
word which is second from the left end? in the word as in alphabetical series?
(a) Two (b) Three (c) Four (a) None (b) One (c) Two
(d) Five (e) None of these (d) Three (e) Four

QUANTITATIVE APTITUDE

41. The retail price of a water geyser is Rs. 1265. If the 44. In a mixture of 45 litres, the ratio of milk and water is
manufacturer gains 10%, the wholesale dealer gains 3 : 2. How much water must be added to make the ratio
15% and the retailer gains 25%, then the cost of the 9 : 11?
product is: (a) 10 litres (b) 15 litres (c) 17 litres
(a) Rs. 800 (b) Rs. 900 (c) Rs. 700 (d) 20 litres (e) None of these
(d) Rs. 600 (e) None of these
45. A person can row with the stream at 8 Km per hour and
42. A pipe can fill a cistern in 6 hrs. Due to a leak in its
against the stream at 6 Km an hour. The speed of the
bottom, it is filled in 7 hrs. When the cistern is full, in
current is:
how much time will it be emptied by the leak?
(a) 42 hrs (b) 40 hrs (c) 43 hrs (a) 1 Km/h (b) 2 Km/h (c) 4 Km/h
(d) 45 hrs (e) None of these (d) 5 Km/h (e) None of these

43. Ram travels a certain distance at 3 km/h and reaches 46. A father’s age is three times the sum of the ages of his
15 minutes late. If he travels at 4 km/h, he reaches 15 two children, but 20 years hence his age will be equal
minutes earlier. The distance he has to travel is: to the sum of their ages. Then, the father’s age is:
(a) 4.5 km (b) 6 km (c) 7.2 km (a) 30 years (b) 40 years (c) 35 years
(d) 12 km (e) None of these (d) 45 years (e) None of these

81 www.bankersadda.com | www.sscadda.com | www.careerpower.in | Adda247 App


The IBPS RRB PO & Clerk 2021 | SUCCESS GUIDE
47. A sum was put at simple interest at a certain rate for 3 49. A card is drawn at random from a well-shuffled pack of
years. Had it been put at 1% higher rate, it would have 52 cards. What is the probability of getting a two of
fetched Rs. 5100 more. The sum is: hearts or a two diamonds?
3 2 1
(a) Rs. 170000 (b) Rs. 150000 (c) Rs. 125000 (a) (b) (c)
26 17 26
(d) Rs. 120000 (e) None of these 4
(d) 13 (e) None of these
48. From among 36 teachers in a school, one principal and
50. A sum is invested for 3 years at compound interest at
one vice-principal are to be appointed. In how many
5%, 10% and 20% respectively. In three years, if the
ways can this be done?
sum amounts to Rs. 16,632, then find the sum.
(a) 1260 (b) 1250 (c) 1240
(a) Rs. 11000 (b) Rs. 12000 (c) Rs. 13000
(d) 1800 (e) None of these
(d) Rs. 14000 (e) None of these
Directions (51-55): Table shows the mobile phones sold on different days by different sellers. Read the table carefully
and answer the questions.

51. Find the difference of mobile phones sold by P and R Directions (56-65): What should come in place of
together on Monday to the mobile phones sold by S and question mark (?) in following simplification problems?
T on Wednesday ? 56. 45% 𝑜𝑓 600+? % 𝑜𝑓 480 = 390
(a) 60 (b) 50 (c) 80 (a) 20 (b) 25 (c) 30
(d) 20 (e) None of these (d) 40 (e) None of these
52. Find the ratio of mobile phone sold by Q on Tuesday 2 1
57. 4 3 + 7 6 − 5 9 = ?
2

and Saturday together to the mobile phone sold by R 2 2 11


(a) 6 (b) 6 (c) 6
on Thursday and Sunday together? 3 9 18
7
(a) 7 : 19 (b)19 : 5 (c)19 : 6 (d) 6 (e) None of these
18
(d) 2 : 5 (e) None of these
58. 65% 𝑜𝑓 240+ ? % 𝑜𝑓 150 = 210
53. Mobile phones sold by P and S together on Wednesday (a) 45 (b) 46 (c) 32
is what percent of mobile phone sold by T on Sunday ? (d) 36 (e) None of these
(a) 400% (b) 200% (c) 100% 59.
2 2
𝑜𝑓 1 of 75% of 540 = ?
(d) 50% (e) None of these 3 5
(a) 378 (b) 756 (c) 252
54. What is the average of mobile phone sold by Q on (d) 332 (e) None of these
Wednesday, T on Sunday and S on Monday ? 60. 555.05 + 55.50 + 5.55 + 5 + 0.55 = ?
(a) 24 (b) 36 (c) 30 (a) 621.65 (b) 655.75 (c) 634.85
(d) 28 (e) None of these (d) 647.35 (e) None of these
55. The mobiles sold by P on Thursday are of two types i.e. 61. 1425 + 8560 + 1680 ÷ 200 = ?
Windows phone and Android phone in ratio 3 : 4. Find (a) 58.325 (b) 9973.4 (c) 56.425
the number of Windows phones sold by P on (d) 9939.4 (e) None of these
Thursday? 62. ? % of 800 = 293 – 22% of 750
(a) 14 (b) 24 (c) 16 (a) 14 (b) 18 (c) 12
(d) 12 (e) None of these (d) 16 (e) 20

82 www.bankersadda.com | www.sscadda.com | www.careerpower.in | Adda247 App


The IBPS RRB PO & Clerk 2021 | SUCCESS GUIDE
63. 25.6% of 250 + √? = 119 Directions (71-75): What should come in place of
(a) 4225 (b) 3025 (c) 2025 question mark (?) in following simplification problems?
(d) 5625 (e) None of these
71. 50% 𝑜𝑓 250 + √? = 165
5 5 1 11
64. 4 6 − 5 9 = ? −2 3 + 18 (a) 1700 (b) 1600 (c) 1800
3 1 7 (d) 2000 (e) None of these
(a) 4 (b) 2 18 (c) 1 9
11 72. 140% 𝑜𝑓 56 + 56% 𝑜𝑓 140 = ?
(d) 1 (e) None of these
18 (a) 78.4 (b) 158.6 (c) 156.6
65. [30% 𝑜𝑓 {(80% 𝑜𝑓 850) ÷ 34}] = ? (d) 87.4 (e) None of these
(a) 5 (b) 4 (c) 6 1 5 5 1
(d) 8 (e) 9 73. 1 4 + 1 9 × 1 8 ÷ 6 2 = ?
1 1 1
(a) 17 (b) 27 (c) 42
66. The sides of a triangle are in the ratio of ∶ ∶ . If the (d) 18 (e) None of these
2 3 4
perimeter is 52 cm, then the length of the smallest side 74. 999.09 + 99.90 + 9.99 + 9 + 0.99 =?
is: (a) 1118.97 (b) 1128.97 (c) 1218.97
(a) 9 cm (b) 10 cm (c) 11 cm (d) 1139.97 (e) None of these
(d) 12 cm (e) None of these
75. 20% 𝑜𝑓 [{(220% 𝑜𝑓 40) − 10}]% 𝑜𝑓 500 = ?
67. If A’s salary is 25% higher than B’s salary, then how (a) 58 (b) 68 (c) 98
much per cent is B’s salary lower than A’s?
(d) 78 (e) None of these
(a) 15% (b) 20% (c) 25%
1
(d) 33 3% (e) None of these Directions (Q.76-80): What should come in place of
question mark (?) in following number series ?
1
68. Ravi sells an article at a gain of 12 2 %. If he had sold it 76. 5, 8, 12, 18, 27, ?
at Rs. 22.50 more, he would have gained 25% The cost (a) 39 (b) 40 (c) 41
price of the article is: (d) 42 (e) 43
(a) Rs. 162 (b) Rs. 140 (c) Rs. 196
(d) Rs. 180 (e) None of these 77. 2, 10, 30, 68, 130, ?
(a) 210 (b) 215 (c) 222
69. A certain job was assigned to a group of men to do it in (d) 228 (e) 235
20 days. But 12 men did not turn up for the job and the
remaining men did the job in 32 days. The original 78. 142, 133, 115, 88, ?
number of men in the group was: (a) 50 (b) 53 (c) 55
(a) 32 (b) 34 (c) 36 (d) 51 (e) 52
(d) 40 (e) None of these 79. 3, 8, 18, 38, 78, ?
70. A vessel contains liquid P and Q in the ratio 5 : 3. If 16 (a) 158 (b) 154 (c) 150
litres of the mixture are removed and the same (d) 162 (e) 166
quantity of liquid Q is added, the ratio become 3 : 5. 80. 6, 3, 3, 6, 24, ?
What quantity does the vessel hold? (a) 184 (b) 186 (c) 188
(a) 35 litres (b) 45 litres (c) 40 litres (d) 190 (e) 192
(d) 50 litres (e) None of these

Solutions
REASONING ABILITY
1. (e); I. B > R (True) II. V > A (True) Direction (6-10):
Floors Persons
2. (a); I. F > I (True) II. J ≥ E (False) 8 K
7 J
3. (d);I. E < M (False) II. E>O (False) 6 I
5 N
4. (c); I. E =G (False) II. G< E (False) 4 H
3 L
5. (b);I. S < Q (False) II. X < S (True) 2 G
1 M
83 www.bankersadda.com | www.sscadda.com | www.careerpower.in | Adda247 App
The IBPS RRB PO & Clerk 2021 | SUCCESS GUIDE
6. (d); 7. (c); 8. (a); 28. (b);Five
9. (c); 10. (e); 29. (d);
11. (c); Sathyarthi’s position from left end = 10th
Sathyarthi’s position from right end = 17th
Total number of children in the row
= 10 + 17 – 1 = 26
30. (e);
12. (b);Raj started walking towards west.
13. (c); By using condition (iii) the code of WX6ZF1 will be
^®£@$∞.
14. (c); The code of FE1XI6 will be $>^®µ£. 31. (e);
15. (a); By using condition (ii) the code of 5L2IA1 will be
∆!<µ&∆.
16. (d);By using condition (i) the code of E2ZA6 will be
32. (a);
+<@&+.
17. (b);By using condition (i) the code of IZ2W2 will be
+@<∞+.
Direction (18-22):
33. (a);

18. (b); 19. (d); 20. (b);


21. (b); 22. (d);
34. (c); SAD
23. (e);
35. (a); TWO
So, the odd one out will be MNO. 36. (a); None
24. (d);More than three 37. (e); WON, TAR
25. (b);8 38. (b);One
26. (a); 0 39. (b);1
27. (d);4 40. (d);Three- RS, HI, and PS

QUANTITATIVE APTITUDE

41. (a); Cost price =


100 100 100
× × ×1265=Rs. 800 44. (b);Let, M = 3K, W = 2K
110 115 125
∴ 3K + 2K = 45 ⇒ K = 9
1
42. (a); In one hour, of the cistern can be filled ⇒ Milk = 27 litres and water = 18 litres
6 Now suppose x litres of water is added to the
1
In one hour, only 7 of the cistern can be filled due mixture such that
27 9
to leak in its bottom = ⇒ 162 + 9𝑥 = 297
18+𝑥 11
1 1 1 ⇒ 9𝑥 = 135 ⇒ 𝑥 = 15
∴ In one hour 6 − 7 = 42 of the cistern is empty
∴ The whole cistern will be emptied in 42 hrs 45. (a); Let the speed of the current be x Km/h and speed
of the person in still water be y km/h.
43. (b);Let D be the required distance ∴y+x=8
𝐷
So, −
𝐷
=
15+15
y–x=6
3 4 60
⇒ y = 7, x = 1
Or, D = 6 km
∴ Speed of the current = 1 Km/h.
84 www.bankersadda.com | www.sscadda.com | www.careerpower.in | Adda247 App
The IBPS RRB PO & Clerk 2021 | SUCCESS GUIDE
46. (a); Let the father’s age be x years and age of his 61. (e); ? = 1425 + 8560 + 1680 ÷ 200
children be a and b years = 1425 + 8560 +
1680
𝑥 200
∴ (𝑎 + 𝑏) = 3
= 9985 + 8.4 = 9993.4
And (𝑎 + 𝑏) + 20 + 20 = 𝑥 + 20 800×? 750×22
𝑥 62. (d); = 293 −
⇒ + 20 = 𝑥 ⇒ 𝑥 = 30 years 100 100
3 ⇒ 8 × ? = 293 − 165 = 128
5100 128
47. (a); Simple interest for 1 year = 3 = 𝑅𝑠 1700 ⇒?= = 16
8
1% of sum = 1700 25.6
∴ sum =
1700×100
= 𝑅𝑠 170000 63. (b);250 × 100 + √? = 119
1
⇒ 64 + √? = 119
48. (a); One principal can be appointed in 36 days
One vice-principal appointed in remaining 35 ⇒ √? = 119 − 64 = 55
ways ⇒ ? = 55 × 55 = 3025
∴ Total no. of ways = 36 × 35 = 1260. 5 5 1 11
64. (e); 4 + − 5 − =? −2 − +
6 9 3 18
49. (b);∴ Required probability 5 5 1 11
13 𝐶 + 13 𝐶
2 2 ⇒ ? = 4 − 5 + 2 + (6 − 9 + 3 − 18)
= 52 𝐶
2 15−10+6−11
78+78 156 2 ⇒1+( )=1+0=1
= = = 18
1326 1326 17
Alternately, 30 80
65. (c); ? = [100 × {(100 × 850) ÷ 34}]
Required probability
13 12 13 12 30
= 52 × 51 + 52 × 51 = [ × {680 ÷ 34}]
100
13 12 2 30
= 2 × 52 × 51 =17 = [100 × 20] = 6
50. (b);Let, P be the sum. 1 1 1
5 10 20 66. (d);Sides of a triangle are in ratio 2 ∶ 3 ∶ 4, i.e.,
∴ 16632 = 𝑃 (1 + 100) (1 + 100) (1 + 100)
21 11 6
6 : 4 : 3.
Or, 16632 = 𝑃 × × × Let the sides be 6K, 4K and 3K, respectively.
20 10 5
Or, P = Rs.12,000 ∴ 13K = 52 ⇒ K = 4
51. (b);Required difference = (40 + 80) – (12 + 58) ∴ Sides of the triangle are 24 cm, 16 cm and 12 cm,
= 120 – 70 = 50 respectively.

52. (c); Required ratio = 13+47 =


92+98 190
= 19 ∶ 6. 67. (b);A = B + 25% of B
60 𝐵 5𝐵
48+12 60
⇒𝐴=𝐵+ =
4 4
53. (a); 𝑅equired percentage= 15
× 100 = 15 × 100 4 1
⇒ 𝐵 = 5 𝐴 = 𝐴 − 5 𝐴 = 𝐴 − 20% 𝑜𝑓 𝐴
= 400%
27+15+60 102 1
54. (e); 𝐴𝑣𝑒𝑟𝑎𝑔𝑒 = = = 34. 68. (d);12 2 % = 𝑅𝑠 22.50
3 3
⇒ C.P. = Rs 180.
55. (d);Windows phones sold by P on Thursday
3
= × 28 =12 69. (a); Suppose x = original number of men in the group
7
45 ?
∴ (x- 12) men did the job in 32 days
56. (b); 𝑜𝑓 600 + 𝑜𝑓 480 = 390 ∴ 20x = 32(x – 12)
100 100
⇒ 270 + 4.8 ×? = 390 i.e., x = 32
390−270
∴? = = 25
4.8 70. (c); Let, the quantity of liquid P and Q be 5x and 3x
57. (c); ? =
14
+
43

47
=
84+129−94
=
119
= 6 18
11 litres respectively.
3 6 9 18 18 5
65 ?
Quantity of P removed = 5+3 × 16 = 10 litres
58. (d);100 𝑜𝑓 240 + 100 𝑜𝑓 150 = 210 3
Quantity of Q removed = 5+3 × 16 = 6 litres
⇒ 156 + 1.5 ×? = 210 5𝑥−10 3
210−156
∴ ? = 1.5 = 36 Now, 3𝑥−6+16 = 5
2 7 75
59. (a); ? = 3 𝑜𝑓 5 𝑜𝑓 100 𝑜𝑓 540 = 7 × 54 = 378 ⇒ 25𝑥 − 50 = 9𝑥 + 30
⇒ 16𝑥 = 80 ⇒ 𝑥 = 5
60. (a); ? = 555.05 + 55.50 + 5.55 + 5 + 0.55 = 621.65 ∴ Quantity that vessel hold = 8 ×5 = 40 litres

85 www.bankersadda.com | www.sscadda.com | www.careerpower.in | Adda247 App


The IBPS RRB PO & Clerk 2021 | SUCCESS GUIDE
50
71. (b);100 𝑜𝑓 250 + √? = 165 76. (b);

⇒ 125 + √? = 165
⇒ √? = 40
∴ ? = (40)2 = 1600
140 56 77. (c);
72. (e); 100 𝑜𝑓 56 + 100 𝑜𝑓 140
= 78.4 + 78.4 = 156.8
1 5 5 1 5 14 13 13
73. (e); ? = 1 + 1 × 1 ÷ 6 = + × ÷
4 9 8 2 4 9 8 2
5 14 13 2
= 4+ 9
× 8
×
13 78. (e);
5 7 45+14 59 23
= 4 + 18 = 36
= 36 = 1 36

74. (a); 999.09 + 99.90 + 9.99 + 9 + 0.99


= 1118.97 79. (a);
20 220
75. (d); × [{( × 40) − 10}] % 𝑜𝑓 500 = ?
100 100
1
× [{88 − 10}]% 𝑜𝑓 500 = ? 80. (e);
5
1 78
5
× 100 × 500 = ?
? = 78

86 www.bankersadda.com | www.sscadda.com | www.careerpower.in | Adda247 App


The IBPS RRB PO & Clerk 2021 | SUCCESS GUIDE

Most Important Questions | Quantitative Aptitude | IBPS RRB


PO & CLERK PRELIMS
IBPS RRB PO PRELIMS

Directions (1-15): What should come in place of question 5


12. 45% of 2770 + 4 𝑜𝑓 1824 = 5 ×?
mark (?) in the following questions?
(a) 701.2 (b) 705.3 (c) 709.1
1
1. 37 % of 4200 + (25)² + (10)³ – 700 = (?)² (d) 704.5 (e) 706.3
2
(a) 50 (b) 60 (c) 55 675
13. 3³
+ 112 × 1.5 − 42% 𝑜𝑓 350 =?
(d) 75 (e) 80
(a) 42 (b) 48 (c) 44
?+62.5
2. + 180 × 2 –(12)2 – 20 = (13)2
+ 87 (d) 40 (e) 46
3
(a) 137.5 (b) 125.5 (c) 117.5 1 1 1 2
14. 1 + 2 − 3 =
(d) 112.5 (e) 107.5 3 6 9 ?
1 1 1
(a) 4 3 (b) 5 3 (c) 2 7
3. 22.5 × 12 + (11)² – √? = (19)² 1 1
(a) 800 (b) 750 (c) 825 (d) 5 7 (e) 4 3
(d) 900 (e) 950
15. [(28 × 176) ÷ 16 − 615 × 16 ÷ 240] =? −11
624 440
4. + 3.5 × 6 + 27 = (6)2 + (a) 278 (b) 266 (c) 280
? 2
(a) 5 (b) 3 (c) 4 (d) 267 (e) 279
(d) 8 (e) 7 Direction (16-25): What will come in the place of question
1 1 (?) mark in the following number series:
5. ?% of 625 + 33 3
%
of 750 + 9 11
%
of 5500 – √625 =
(10)³ 16. 25, 26, 28, 37, 101, ?
(a) 44 (b) 46 (c) 56 (a) 524 (b) 529 (c) 625
(d) 54 (e) 64 (d) 726 (e) 728
3
6. 763 + 882 + of 6500 + ? = (60)² 17. 46, 51, 60, 73, 90, 111, ?
13
(a) 465 (b) 445 (c) 425 (a) 136 (b) 140 (c) 132
(d) 455 (e) 545 (d) 130 (e) 134
7. 2450 – 1540 + 1700 – 710 = ?% of 1900 18. 80, 68, 62, 56, ?, 29
(a) 120 (b) 160 (c) 125 (a) 43 (b) 44 (c) 45
(d) 200 (e) 100 (d) 46 (e) 47
1626 1
8. 40% of 1325 + 299 + 6 = 33 3 % of 3300 + ? 19. 70, 72, 77, 88, 105, ?
(a) 10 (b) 20 (c) 0 (a) 121 (b) 127 (c) 126
(d) 15 (e) 25 (d) 128 (e) 135
4750 20. 7, 8, 16, 43, 107, ?
9. 625 ÷ 2.5 + 283 – 157 + √576 + (10)² = ?
(a) 216 (b) 232 (c) 240
(a)9.5 (b)8.5 (c)6.5
(d)8 (e)11.5 (d) 248 (e) 260
21. 59.76, 58.66, 56.46, 52.06, ? , 25.66
10. 2250 + 270 – √400 + 3125 ÷ 25 + ? = (60)²
(a) 875 (b) 945 (c) 935 (a) 42.22 (b) 43.26 (c) 40.34
(d) 975 (e) 925 (d) 38.78 (e) 39.22

11. (9)3 × 6 ÷ 9 + (7)3 + 171 = 100 + (? )3 − 431 22. 36, 157, 301, 470, ?, 891
(a) 12 (b) 9 (c) 13 (a) 692 (b) 722 (c) 682
(d) 10 (e) 11 (d) 666 (e) 625
87 www.bankersadda.com | www.sscadda.com | www.careerpower.in | Adda247 App
The IBPS RRB PO & Clerk 2021 | SUCCESS GUIDE
23. 27, 125, 343, 1331, ?, 4913 29. What is the ratio of number of type E watches
(a) 2089 (b) 2197 (c) 2052 manufactured in 2017 to the number of type D
(d) 2297 (e) 2190 watches manufactured in 2018 if the number of type D
24. 102, 99, 104, 97, 106, ? watches manufactured in 2018 is 30% of all the
(a) 98 (b) 96 (c) 95 watches manufactured in 2018 ?
(d) 92 (e) 97 25 35 35
(a) 38 (b) 48
(c) 78
25. 6, 7, 16, 51, 208, ?, 6276
15
(a) 941 (b) 1024 (c) 1045 (d) (e) none of these
48
(d) 1340 (e) 1145
30. If in 2018, the number of all type of watches except
Directions (26-30): Given below is the pie chart which
type B remained same as that of 2017, then find the
shows the percentage distribution of 5 types of watches
manufactured in company X in year 2017. percentage change in the number of watches of type
Ratio of total watches manufactured in company X in year B in 2018 compared to 2017.( in approximate).
2017 to year 2018 is 7 : 8. (a)79% (b)51% (c)71%
(d)62% (e)48%
A
Direction (31-35): In the following questions, two
E 15%
25% equations numbered I and II are given. You have to solve
both the equations and give answers among the following
options.
B
20% (a) if 𝑥 > 𝑦
D (b) if 𝑥 ≥ 𝑦
12% (c) if 𝑥 < 𝑦
(d) if 𝑥 ≤ 𝑦
C (e) if 𝑥 = 𝑦 or the relationship cannot be established.
28%
31. I. 16x – 15y = 22 II. 4x – 10y = 8
26. If difference between watches manufactured of type B (a) b (b) d (c) a
and type C in year 2017 is 11,200 and type A watches (d) e (e) c
manufactured in year 2018 is 20% more than type D
watches manufactured in year 2017, then find the 32. I. 4x² – 7x =15 II. -33y = (9y² + 18)
number of type A watches manufactured in year 2018 (a) c (b) e (c) a
(a) 21,160 (b) 19,160 (c) 18,160
(d) b (e) d
(d) 20,160 (e) none of these
3 5 3
27. If the average number of type D and type C watches 33. I. 𝑥 + 2𝑥 = 2 II. √𝑦 – =0
√𝑦
manufactured in company X in 2017 is 16,100, then
(a) b (b) a (c) e
find the total number watches manufactured in
company X in 2018. (d) d (e) c
(a)92,000 (b)90,000 (c)88,000
34. I. 𝑥 = √289 II. 𝑦 2 – 34𝑦 + 137 =– 152
(d)96,000 (e)none of these
(a) e (b) c (c) d
28. The number of type B watches manufactured in 2018
(d) a (e) b
is 25,600 which is 25% of the total number of watches
manufactured in 2018. Find the number type D and 35. I. 𝑥 2 + 10𝑥 =– 25 II. 𝑦 2 = 25
type B watches manufactured in 2017 in company X. (a) d (b) e (c) a
(a)23,456 (b)26,754 (c)28,672
(d)24,563 (e)none of these (d) b (e) c

88 www.bankersadda.com | www.sscadda.com | www.careerpower.in | Adda247 App


The IBPS RRB PO & Clerk 2021 | SUCCESS GUIDE
Directions (36-40): Solve the given quadratic equations 36. (i) x² – x – 132 = 0 (ii) y² – 22y + 112 = 0
and mark the correct option based on your answer—
37. (i) x² – 8x + 15 = 0 (ii) y² – 2y – 3 = 0
(a) x > y
(b) x ≥ y 38. (i) x² + 5√3𝑥 + 18 = 0 (ii) 𝑦 2 − √3𝑦 − 18 = 0
(c) x < y
39. (i) 2x² – x – 3 = 0 (ii) 4y² – 24y + 35 = 0
(d) x ≤ y
(e) x = y or no relation can be established between x and y. 40. (i) 2x² – 7x + 6 = 0 (ii) 2y² – y– 1= 0

Directions (41-45): Given bar graph shows percentage distribution of people who like three different fruits i.e., Apple,
Orange and Grapes in 4 cities. Population of P,Q,R and S considered for this survey is in ratio of 6:2:4:5 respectively and
each people from every city likes only one fruits.

100

80

60

40

20

0
P Q R S

Apple Orange Grapes

41. Find ratio of total number of people who like apple 45. Total population from all cities who like oranges are
from cities P and R together to total number of people what percent of total population of cities P and R who
who likes Grapes from these two cities together. are considered for that survey?
(a) 45% (b) 49% (c) 51%
(a) 22 : 25 (b) 10 : 11 (c) 19 : 22
(d) 53% (e) 57%
(d) 31 : 33 (e) 9 : 11
Directions (46-50): Find the wrong number in the
42. People who like oranges from cities Q and R together following number series:
is what percent of people who like this fruit from cities 46. 17, 20, 25, 37, 57, 87, 129
P and S together ? (a) 17 (b) 129 (c) 25
8 5 4
(a) 72 11 % (b) 70 11 % (c) 71 11 % (d) 87 (e) 20
3 9 47. 128, 64, 96, 240, 840, 3800, 20790
(d) 72 11 % (e) 71 11 %
(a) 3800 (b) 128 (c) 20790
43. People who like grapes from city Q is what percent (d) 96 (e)240
more or less than people who like apple from city S ? 48. 14, 20, 40, 82, 154, 264, 450
(a) 50% (b) 66 ⅔% (c) 62 ½% (a) 154 (b) 20 (c) 264
(d) 68 ⅓% (e) 65% (d) 14 (e) 450

44. Number of people who like Grapes from city S is 7200. 49. 64, 56, 65, 49, 74, 38, 87
(a) 87 (b) 64 (c) 38
Then find average population of city P, Q and R
(d) 56 (e) 49
considered for this survey.
(a) 14,400 (b) 14,200 (c) 14,000 50. 2, 5, 11, 35, 143, 719, 4319
(a) 11 (b) 719 (c) 5
(d) 14,140 (e) 14,720
(d) 4319 (e) 2
89 www.bankersadda.com | www.sscadda.com | www.careerpower.in | Adda247 App
The IBPS RRB PO & Clerk 2021 | SUCCESS GUIDE
Directions (51-60): Calculate the approximate value of the (a) 122 (b) 132 (c) 128
given questions : (d) 136 (e) 142
1 3
51. 3027.89 + 671.93 – 39.87% of ? + (9.98)³ = (59.87)² – 56. √16906 ÷ 25.90 + ? × 19.85 × 4.96 = √2198 × 4.88
√9999.98 (a) 21 (b) 15 (c) 11
(a) 4000 (b) 3500 (c) 2000 (d) 28 (e) 8
(d) 2500 (e) 3000 3
57. √21950 ÷ 7.08 + √910 = ? +(13.962 − 106.86 ×
127.5+? 2)
52. 7.98
+ 24.89 × 39.87 – √255.93 = (31.89)2
(a) 26 (b) 35 (c) 39
(a) 172.5 (b) 192.5 (c) 202.5
(d) 69 (e) 52
(d) 162.5 (e) 152.5
58. (27.62)2 + (9.10)3 + √7225 = (? )2
53. √1224.96 + √2024.93 + √3024.97 – √99.87=(?)3 (a) 62 (b) 34 (c) 88
(a)
1 (b) 3 (c) 5 (d) 40 (e) 12
(d) 7 (e) 9
59. 131.9% of 800 – 12.5% of 2048 = 3.98 of ?
54. √1220 × 16.06 + √4897= ? (a) 182 (b) 200 (c) 212
(a) 610 (b) 620 (c) 630 (d) 256 (e) 176
(d) 640 (e) 650 24.96 493.92 1 1
60. 𝑜𝑓 195.984 − × ÷ =?
14 201.05 25 2025
55. 34.96 × 39.89 + √960.89 – √(28.96)2 – 49.96% of ? = (a) 150 (b) 112 (c) 176
(22.87)2 (d) 213 (e) 98
Directions (61-65): Given line graph shows the number of small animals and number of bigger animals (Elephant, Giraffe,
Rhino and Buffalo) found in four different national parks of India. Except these four animals given in bigger animals list,
all are considered as smaller animals. Read the given information carefully and answer following questions.

1800
1700
1600
1500
1400
1300 Smaller animals

1200 Bigger animals

1100
1000
900
800
Hemis Kaziranga Gir Guindy

61. Total animals in Kaziranga National park is 62. Number of Elephant, Giraffe, Rhino and Buffalo in
Hemis national park is in arithmetic progression. Ratio
approximately what percent more or less than number
of number of Elephant to Buffalo is 17 : 19 then number
of smaller animals in Guindy National park? of Rhino in this park is what percent of bigger animal
(a) 20% (b) 28% (c) 10% in Gir national park ?
(a) 25% (b) 20% (c) 30%
(d) 24% (e) 30% 1
(d) 22 % (e) None of these
2

90 www.bankersadda.com | www.sscadda.com | www.careerpower.in | Adda247 App


The IBPS RRB PO & Clerk 2021 | SUCCESS GUIDE
63. Number of Deer in Kaziranga National park is 25% out 68. What is ratio of marks obtained by Z in Physics and
of total smaller animals in this national park. Then, Maths together to marks obtained by X in Chemistry
find the ratio between smaller animals except deer in and Physics together ?
Kaziranga to bigger animals in Guindy national park? (a) 2 : 5 (b) 1 : 2 (c) can’t determined
(a) 1 : 2 (b) 4 : 3 (c) 1 : 1 (d) 4 : 3 (e) 2 : 3
(d) 8 : 9 (e) 11 : 12
69. If Max. marks in each subject is 200. Then find percent
64. Find average of number of animals in Hemis and Gir of marks obtained by Y ?
National park. (a) 61 ⅓% (b) 42 ⅔% (c) 53 ⅓%
(a) 1980 (b) 2070 (c) 2120 (d) None of these (e) 47 ⅔%
(d) 2140 (e) 2170
70. If ratio of marks obtained by Z in Physics and Maths is
65. Total smaller animals in Gir and Guindy national parks 2 : 1, then find average marks obtained by X and Z in
together are how much more or less than total bigger Physics ?
animals in Hemis and Gir National parks together? (a) None of these (b) 150 (c) 160
(a) 320 (b) 380 (c) 410 (d) 120 (e) 140
(d) 360 (e) 300
71. The speed of boat in still water is 5 km/hr more than
Direction (66-70): Study the passage carefully and answer speed of current and the ratio of speed of boat in
the questions based on it. downstream to the speed of boat in still water is 4 : 3.
Marks obtained by X in Physics is 160 which is 40% of total Find downstream distance covered by boat in 3 hours.
marks obtained by him. And marks obtained by Z in (a) 33 km (b) 30 km (c) 36 km
chemistry is ¾th of marks obtained by X in Physics. Ratio (d) 39 km (e) 24 km
of total marks obtained by X to Z is 4 : 3. Marks obtained
72. The length of train A is twice that of train B and speed
by Y in Maths is equal to marks obtained by Z in Chemistry.
of train A is half of that of train B. If train A crosses a
And total marks obtained by Y in Physics and Chemistry
man in 4 sec. then find how long will train B take to
together is half of total marks obtained by X.
cross train A if they go in same direction.
66. If marks obtained by X in Maths is 50% more than (a) 3s (b) 4s (c) 5s
marks obtained by him in Chemistry. Then find (d) 6s (e) None of these
difference of marks obtained by X in Maths and marks
73. While travelling in opposite direction, two trains of
obtained by Y in Maths ?
equal length crosses each other in 5 seconds. If the
(a) 12 (b) 14 (c) 24
speed of trains are 72 km/hr and 40 ms¯¹. Calculate the
(d) 32 (e) 28
length of trains.
67. Marks obtained by Y in Physics and Chemistry (a) 300 meter (b) 150 meter (c) 120 meter
together is what percent of total marks obtained by Z? (d) 90 meter (e) None of these
(a) 66 ⅔% (b) 50% (c) None of these
(d) 33 ⅓% (e) 75% 74. Shyam goes to his office which is 96 km away from his
1
home. He walks 12 2% of total journey and remaining
distance covered by running. He walks at the speed of
8 km/hr and run at the speed of 12 km/hr. Find the time
to cover the whole journey?
(a) 6 hr (b) 8.5 hr (c) 7.5 hr
(d) 8 hr (e) 9 hr
75. Time taken by boat to cover 164 km in upstream is
50% more than time taken by boat while returning.
Find the time taken by man to cover 100 km in still
water if speed of man is equal to speed of boat in still
water and speed of stream is 10 km/hr?
(a) 4 hr (b) 6 hr (c) 5 hr
(d) 2 hr (e) 3 hr

91 www.bankersadda.com | www.sscadda.com | www.careerpower.in | Adda247 App


The IBPS RRB PO & Clerk 2021 | SUCCESS GUIDE
76. A man went to market with X Rs. in his pocket. He (a) 12,000 (b) 13,000 (c) 14,000
spent 50% of X on groceries and Rs. 120 on his lunch (d) 15,000 (e) 16,000
X
and 15 on dessert. Total money left with him was Rs. 84. A, B, C and D started a business with an amount of Rs.
400. Find the amount spent on dessert. 12,000, Rs. 15,000, Rs. 18,000 and Rs. 21,000
(a) Rs. 120 (b) Rs. 60 (c) Rs. 160 respectively. D, C and B left the business after 3
(d) Rs. 80 (e) None of these month, 6 months and 9 months respectively after
77. An amount of Rs. 4004 is to be distributed in the ratio starting of the business. After a year what will be the
4 : 6 : 3. Find difference between share of person who profit of ‘C’ out of total profit of Rs. 800.
got highest and who got lowest. (a) Rs. 178 (b) Rs. 188 (c) Rs. 182
(a) Rs. 924 (b) Rs. 800 (c) Rs. 876 (d) Rs. 192 (e) Rs. 198
(d) Rs. 900 (e) Rs. 1276
85. The cost price of a pencil is Rs 8 less than that of pen.
78. Ten’s digit of a two-digit number is 3 more than the
square of its unit digit. If reverse of that number is 45 When the pen is sold at 50% profit and pencil is sold at
less than that of original, then find the original 150% profit it is found that the selling price of both is
number. same. Find the cost price of pen.
(a) 41 (b) 27 (c) 14 (a) Rs. 20 (b) Rs. 24 (c) Rs. 25
(d) 52 (e) 72 (d) Rs. 16 (e) Rs. 15
79. The average price of three different articles X, Y and Z 86. A is 40% more efficient than B and both complete a
is Rs. 15000. If the price of each article is increased by 3
work together in 16 days. If C & D takes 4 th of time
Rs. 1500 then the ratio of these article becomes 4 : 5 :
6 respectively. Find the price of article ‘Z’ after what A & B take together, then find In how many days
increment in price? B, C & D will complete the work together?
1 1 1
(a) Rs. 19,800 (b) Rs. 18,000 (c) Rs. 23,500 (a) 12 days (b) 10 days (c) 6 days
7 7 7
(d) Rs. 20,000 (e) Rs. 16,500 1 1
(d) 9 7 days (e) 4 7 days
2
80. X is two times of Y, while Z is 16 3 % more than Y and
87. Three pipes P, Q and R together can fill a cistern in 11
U is 20% more than average of X, Y & Z. If difference hours. If pipe P, Q and R opened together but after 6
between X & U is 240, then find the average of X & U? hours ‘Q’ is closed and P and R fills the remaining
(a) 1260 (b) 1300 (c) 1360 cistern in 8 more hours then find in how much time
(d) 1320 (e) 1380
pipe ‘Q’ can fill the cistern alone?
81. Ten years ago, age of Anurag was 8 more than half of 1 1 1
(a) 29 3 hours (b) 28 2 hours (c) 14 2 hours
his present age, while ratio of age of Anurag’ four 1 1
years ago to age of Mohit two years ago was 8 : 7. If (d) 27 3 hrs (e) 25 3 hrs
ratio between age of Mohit to age of Ayush six year
hence will be 3 : 2, then find difference between 88. Sum of perimeter of two rectangles is 216 cm and
present ages of Anurag and Ayush ? length of both rectangle are in ratio 3 : 2. If breadth of
(a) 27 years (b) 35 years (c) 30 years both rectangle are equal to the side of square whose
(d) 25 years (e) 18 years area is 196 cm². Then find sum of are of both
rectangle?
82. A vessel contains 180 l mixture of milk and water in the
(a) 1120 cm² (b) 1210 cm² (c) None of these
ratio of 13 : 5 respectively. 54 l mixture taken out from
the vessels and some quantity of milk and water added (d) 1140 cm² (e) 1080 cm²
in the remaining mixture in the ratio of 1 : 3. If ratio of 89. What is the probability that a number is divisible by 3?
milk and water in resulting mixture is 50 : 31, then find 1 3x+1 3
(a) (b) (c)
the amount of water added in remaining mixture? 3 9x+1 10
1
(a) 18 l (b) 36 l (c) 24 l (d) 2 (e) None of these
(d) 27 l (e) 48 l
90. In a party each person shakes hands exactly once with
83. Sanjay invested Rs. 28000 partially in scheme A which each of the others and total number of handshakes is
offers 15% p.a. at S.I. and remaining in scheme B
78. Then find the number of person ?
which offer 18% p.a. at S.I. If after one year he earns
(a) 12 (b) 11 (c) 13
Rs. 4680 as total interest. Find the amount invested by
him in scheme B? (d) 15 (e) 18

92 www.bankersadda.com | www.sscadda.com | www.careerpower.in | Adda247 App


The IBPS RRB PO & Clerk 2021 | SUCCESS GUIDE
Directions (91-95): In the following questions two (a) Statement I alone is sufficient to answer the question
quantities are given for each question. Compare the but statement II alone is not sufficient to answer the
numeric value of both the quantities and answers questions.
accordingly. (b) Statement II alone is sufficient to answer the question
(a) Quantity I > Quantity II but statement I alone is not sufficient to answer the
(b) Quantity II > Quantity I question
(c) Quantity I ≥ Quantity II (c) Both the statements taken together are necessary to
(d) Quantity II ≥ Quantity I answer the questions, but neither of the statements
(e) Quantity I = Quantity II or relation can’t be established. alone is sufficient to answer the question.
(d) Either statement I or statement II by itself is sufficient
2
91. Quantity I: Sum of the first and second number. 3rd of to answer the question.
first number is equal to the cube of the second number (e) Statements I and II taken together are not sufficient to
and second number is equal to 12% of 100 answer the question.
Quantity II: 2352
96. How many marks did Neeraj obtain in Mathematics?
92. Quantity I: Time taken by the police to catch the thief. (I) Neeraj obtained an average of 65% marks in
A thief robbed a shop and ran in a car at a speed of 60 Mathematics, English and Social science.
km/h at 11:00 am. The Police located the position of (II) Neeraj secured 10% marks more in Mathematics
thief and ran after him at 11:15 a.m. from shop in a car. than the average of Mathematics, English and
Maximum speed of the car of police is 65 km/h Social Science.
Quantity II: 3hr 97. There are four consecutive even number what is the
93. Quantity I: 4 value of smallest number among these?
Quantity II: Value of ‘x’ (I) The average of the four consecutive even number
A, B and C started a business together with Rs 12,000, is the first prime number greater than 8.
Rs 12,000 and Rs 8,000 respectively. B worked only for (II) The difference between the largest and the
‘x’ months while C left the business ‘x’ month before smallest of the number is less than 10.
the completion of year. If out of annual profit of Rs 98. A sum of Rs. 705 is distributed among three persons P,
3200, ‘A’ got Rs 1800. Q and R. Who gets the least?
94. Quantity I: Value of ‘X’ (I) P gets ⅔rd of what (Q + R) are getting.
The time taken by a boat for covering ‘X – 18’ km (II) Q gets ¼th of what (P + R) are getting.
upstream is equal to time taken by it for covering ‘X’ 99. What is length of rectangle?
km downstream. If upstream speed is 6 km/hr less I. Ratio between radius & height of cylinder is 7 : 6
than downstream speed and speed of boat in still and breadth of rectangle is equal to height of
water in 15 kmph. cylinder. Volume of cylinder is 7392 cm3 and
Quantity II: 50 perimeter of rectangle is 80 cm.
II. Length of rectangle is two times of side of square,
95. Quantity I: Price at which P sold watch to Q
of area 196 cm2.
‘P’ sells his watch at 20% profit to Q while Q sells it to
R at a loss of 10%. R pays Rs.2160. 100. A bag contains seven red, ‘y’ blue & ‘x’ yellow balls.
Quantity II: 1600 How many total blue & yellow balls together?
I. One ball is drawn from bag, probability of being
Directions (96-100): The following questions are 1
that ball blue is 4.
accompanied by two statements I and II. You have to
determine which statements(s) is/are sufficient/necessary II. One ball is drawn from bag, probability of being
2
to answer the questions. that ball yellow is 5 .

93 www.bankersadda.com | www.sscadda.com | www.careerpower.in | Adda247 App


The IBPS RRB PO & Clerk 2021 | SUCCESS GUIDE
IBPS RRB CLERK PRELIMS

Directions (1-20): What should come in place of question 13. 75% of 160 + 45% of 300 = ? × 17
mark (?) in the following questions? (a) 17.5 (b) 12.5 (c) 25
1. 7843 – 7777 + 2531– 239 = ? (d) 15 (e) 18
(a) 2358 (b) 1358 (c) 3258 2 8 1 8
14. 28 7 × 5 11 +? = 36 9 × 7 13
(d) 1344 (e) 1323
(a) 125 (b) 118 (c) 113
7525
2. 52
+ 3125 × (5)–4 – 105 × 2 =? (d) 108 (e) 103
(a) 97 (b) 86 (c) 96 255 272 85
(d) 106 (e) 94 15. ? = 102 × 204 ÷ 153

28×25 1398
(a) 7 (b) 4 (c) 5
3. 2 ( )– + 5 × 24 = ? (d) 6 (e) 9
4 3
(a) 5 (b) 7 (c) 14
16. (841 ÷ 29) + (34 × 5) – (23 × 5) =?
(d) 12 (e) 4
(a) 88 (b) 81 (c) 84
232 +372 +46×37
4. 192 +112 +22×19
= (? )2 (d) 78 (e) 72
(a) 1 (b) 120 (c) 2 17. ? ² = 40% of 420 + 44% of 200
(d) 13 (e) 5 (a) 24 (b) 12 (c) 8
82 +42 +22 1 (d) 16 (e) 416
5. =
182 +122 +62 ?
(a) 6 (b) 5 (c) 4 18. 343 + 243 + 512 = 20% of ?
(d) 3 (e) 2 (a) 4590 (b) 5490 (c) 6490
6. 2⁵ + 2¹⁰ = ? × 12 (d) 6140 (e) 5290
(a) 68 (b) 72 (c) 80 19. (0.3)3 × (0.09) × (0.027) = (0.0081)?+2
(d) 88 (e) 96 (a) 0 (b) 1 (c) 2
7. √256 + √784 = ? × √121 (d) 3 (e) 4
(a) 4 (b) 5 (c) 3 3 3
20. √1728 + √784 + √1296 = ? + √512
(d) 6 (e) 2
(a) 86 (b) 68 (c) 74
8. 80% of 350 + 45% of 800 = ? × 256 (d) 72 (e) 64
(a) 1.75 (b) 2 (c) 2.25
(d) 2.5 (e) 3 Directions (21-25): Table given below shows the
percentage distribution of students who have appeared in
9. 115% of 360 + 180% of 270 = ? × 225 AIIMS entrance exam from five different institutes and
(a) 4.5 (b) 3 (c) 3.25
number of girls out of each institute. Read the information
(d) 3.5 (e) 4
carefully and answer the following questions. (Note→ all
10. 9? × 729 = 3⁴ × 9⁶ ÷ 81 students appeared in this exam are from these institutes
(a) 2 (b) 3 (c) 4 only)
(d) 5 (e) 6 Students → Boys + Girls.
1 1 1 1 1
11. 5 4 + 7 3 + 4 2 = 3 6 +? +5 6 Institutes Students Girls
1 1
(a) 8 (b) 8 4 (c) 8 2 A 24% 360
3
(d) 8 (e) 9 B 22% 50%
4
1 1 C 490 60%
12. 37 2 % of 300 + 62 2 % of 460 =?
D 18% 280
(a) 350 (b) 400 (c) 420
(d) 460 (e) 500 E 22% 70%

94 www.bankersadda.com | www.sscadda.com | www.careerpower.in | Adda247 App


The IBPS RRB PO & Clerk 2021 | SUCCESS GUIDE
21. Total number of girls from institute A and C together (a) 200 (b) 120 (c) 160
are what percent of total students from institute A ? (d) 140 (e) 180
6 4
(a) 75% (b) 77 7% (c) 78 7 % Directions (26-30): Find the wrong number in the
1 2
(d) 77 % (e) 78 % following number series:
7 7

22. Find difference of boys from institute D and boys from 26. 31, 53, 105, 182, 280, 391
institute E ? (a) 391 (b) 31 (c) 280
(a) 126 (b) 121 (c) 117 (d) 53 (e) 105
(d) 130 (e) 119 27. 1, 1, 3, 23, 367, 11745
23. Find ratio of number of girls from institute B to (a) 11745 (b) None of these (c) 3
number of boys from institute D ? (d) 23 (e) 367
(a) 11 : 10 (b) 6 : 5 (c) 4 : 3 28. 125, 127, 137, 163, 213, 296
(d) 8 : 7 (e) 15 : 14 (a) 125 (b) 127 (c) 163
(d) 296 (e) 213
24. Find average number of students who have appeared
from institute A and D. 29. 675, 338, 170, 86, 44, 23
(a) 755 (b) 685 (c) 735 (a) 23 (b) 338 (c) 170
(d) 705 (e) 715 (d) 44 (e) 675
25. Total number of girls from B and C together are how 30. 48, 62, 96, 224, 992, 7136
much more or less than total number of girls from D (a) 48 (b) 62 (c) 224
and E together. (d) 992 (e) 7136
Directions (31–35): Given bar graph shows the number of student passed in Xth class from 6 different school.

140

120

100

80

60

40

20

0
P Q R S T U

31. Pass percentage of school S is equal to that of school 33. If ratio between total student who passed to who
Q. Find total strength of school Q is what % more than failed from all school is 7 : 3, then find the total number
that of school S. of failed student from all schools together.
(a) 20% (b) 40% (c) 50% (a) 225 (b) 125 (c) 250
(d) 25% (e) 60% (d) 275 (e) None of these
32. If fail percentage of school P is 65% then, find number 34. Student passed from school P, Q, U and T together is
of student failed from school P is what percentage of how much more than that of school R and S together.
number of student passed from school T.
(a) 30% (b) 100% (c) 120% (a) 220 (b) 250 (c) 190
(d) 130% (e) 70% (d) 220 (e) None of these

95 www.bankersadda.com | www.sscadda.com | www.careerpower.in | Adda247 App


The IBPS RRB PO & Clerk 2021 | SUCCESS GUIDE
35. Failed student of school U is 15 more than that of 1
40. 25% of 2001 + 2001% of 25 – 33 3% 100 = ? + 50
school R. If ratio between total strength of school U to
(a) 917 (b) 906 (c) 910
school R is 3 : 2, then find the total number of failed
(d) 920 (e) 927
student from both schools together.
(a) 57 (b) 23 (c) 45 41. 342.09 + 575.98 − ?2 = (12.09)2− √99.95
(d) 63 (e) 35 (a) 28 (b) 24 (c) 16
Directions (36-45): Find the approximate value of (?) in (d) 32 (e) 36
given questions:
42. 24.09 × ? +11.98 % 𝑜𝑓 224.99 = (19.02)2 + √676.09
36. 119.79% of 119.89 + 70.12% of 79.73 = ?% of 999.89 (a) 5 (b)10 (c) 20
(a) 2.5 (b) 20 (c) 15 (d) 25 (e) 15
(d) 35 (e) 40
43. (12.01)3 + ?2 = 35.98 % of 5200.08
37. 1.823 + 5.126 × 7.01 – 432.001 = ? – 531.99
(a) 8 (b) 12 (c) 16
(a) 155 (b) 126 (c) 137
(d) 120 (e) 145 (d) 24 (e) 20
22
38. 43 × 7 + 20% of 529.7 – 43 =? 44. 472.09 + 548.09 − ?2 = (18.02)2 + √399.98
(a) 26 (b) 29 (c) 33
(a) 177 (b) 185 (c) 212
(d) 205 (e) 195 (d) 36 (e) 40

39. (23.12 × 22.98) + 11.89 × 7.79 = ?² 45. 121.09 + ? = 96.02 % of 499.98


(a) 20 (b) 25 (c) 31 (a) 350 (b) 348 (c) 342
(d) 22 (e) 30 (d) 336 (e) 359

Directions (46-50): Pie-chart given below shows total income of Sandeep in six different months and percentage
distribution in these months. Study the data carefully and answer the following questions.

Total income = 15,000

Jan
Jun 8%
Feb
24%
12%

Mar
16%
May
20%
Apr
20%

46. Income of Sandeep in the month of Jan and April 48. Which month shows the highest percent increment in
together is what percent less than income of Sandeep income as compare to previous month?
in the month of Mar and Jun together?
(a) Feb (b) March (c) April
(a) 20% (b) 30% (c) 40%
(d) 50% (e) 70% (d) May (e) Both (b) and (c)

47. Income of Sandeep in May and Jun together is how 49. Income in the month of March and April together
much more than the income of Sandeep in Feb and makes how much central angle of the total?
March together?
(a) 115.2° (b) 158.4° (c) 144°
(a) 1500 (b) 1800 (c) 1200
(d) 2400 (e) 2700 (d) 100.8° (e) 129.6°

96 www.bankersadda.com | www.sscadda.com | www.careerpower.in | Adda247 App


The IBPS RRB PO & Clerk 2021 | SUCCESS GUIDE
50. Sandeep’s average income in starting four month in 66. 3, 4, 8, 17, 33, ?
the given six months is how much less than Sandeep’s (a)47 (b)45 (c)58
average income in last four months in the given six (d)54 (e) None of these
months?
(a) 300 (b) 600 (c) 900 67. 5, 7, 13, 25, 45, ?
(d) 1200 (e) 1500 (a)45 (b)75 (c)65
(d)86 (e) None of these
Directions (51-55): In each of these questions, two
equations (I) and (II) are given. You have to solve both the 68. 7, 7, 10, 18, 33, ?
equations and give answer (a)34 (b)54 (c)57
(a) if x>y (b) if x≥y (d)65 (e) None of these
(c) if x<y (d) if x ≤y 69. 2, 7, 25, 105, 531, ?
(e) if x = y or no relation can be established between x and (a)2341 (b)3241 (c)3567
y. (d)3193 (e) None of these
51. I. x² + 13x +42 = 0 II. y² + 11y + 30 = 0 70. 10, 13, 18, 26, 38, ?
52. I. x² - 9x -90=0 II. y² + 14y + 48 = 0 (a)43 (b)55 (c)65
(d)45 (e) None of these
53. I. 4x² +16x +15=0 II. 3y² + 4y + 1 = 0
54. I. x² + 2x – 35 = 0 II. y² + 3y – 10 = 0 Directions (71-75): Read the given information carefully
and answer the following questions.
11
55. I. 4x + 8y = 16 II. x + y = Total population of Gurgaon sector -29 is 26,000 and there
4
are three sectors i.e. A, B and C in it. Ratio of population of
Directions (56-60): In each question two equations these sectors (A: B: C) is 21: 16: 15. Number of males in
numbered (I) and (II) are given. Student should solve both sector A is 2500 more than that of females in sector C.
the equations and mark appropriate answer. Ratio of number of females in sector A to that of males in
(a) If 𝑥 = 𝑦 or no relation can be established
sector C is 9: 8. Number of females in sector B is 70% of the
(b) If 𝑥 > 𝑦 (c) If 𝑥 < 𝑦
number of males in sector A.
(d) If 𝑥 ≥ 𝑦 (e) If 𝑥 ≤ 𝑦
71. Find the average of number of males in sector A and B
56. I. 𝑥 2 + 32𝑥 − 273 = 0 II. 3𝑦 2 − 44𝑦 + 161 = 0
is how much more or less than number of females in
57. I. 100𝑥 2 − 20𝑥 + 1 = 0 II. 400𝑦 2 − 4 = 0 sector A ?
58. I. 3𝑥 2 + 2𝑥 − 5 = 0 II. 2𝑦 2 + 𝑦 – 6 = 0 (a) 200 (b) 300 (c) 400
(d) 500 (e) 600
59. I. 2𝑥 2 + 31𝑥 + 120 = 0 II. 2𝑦 2 + 3𝑦 − 90 = 0
72. 68% and 85% of females of sector A and B respectively
60. I. 2𝑥 2 − 13𝑥 + 21 = 0 II. 2𝑦 2 − 7𝑦 + 3 = 0 are working. Then find working females of B is how
Direction (61-70): Find the value of question mark (?) in much percent more or less than that of working
following number series: females of A.
2
61. 12, 12, 15, 23, 38, ? (a) 20 % (b) 15% (c) 147%
(a) 63 (b) 61 (c) 62 (d) 22 ½% (e) 16 ⅔%
(d) 58 (e) 38
62. 15, 33, 103, 417, 2091, ?
(a) 12551 (b) 11542 (c) 14553
(d) 12553 (e) 13555
63. 2, 6, 25, 96, 285, ?
(a) 667 (b) 568 (c) 846
(d) 776 (e) 602
64. 22, 33, 20, 37, 18, ?
(a) 17 (b) 33 (c) 39
(d) 27 (e) 41
65. 17, 26, 30, 39, 43, ?
(a) 52 (b) 47 (c) 49
(d) 59 (e) 51
97 www.bankersadda.com | www.sscadda.com | www.careerpower.in | Adda247 App
The IBPS RRB PO & Clerk 2021 | SUCCESS GUIDE
73. Total male population of sector A and C together is 81. Trains A and B are travelling at x km/hr and (x + 36)
what percent of total population of these two sectors? km/hr respectively. Train B crosses train A when
5 5 1
(a) 55 9 % (b) 52 9 % (c) 50% running in the same direction in 31 2 seconds. Find the
2 4 sum of speed of both the trains if they pass each other
(d) 55 9 % (e) 57 5 %
in 9 seconds while running in opposite direction
74. Find ratio of total number of females in all sectors to (a) 126 km/hr (b) 120 km/hr (c) 116 km/hr
that of total number of male population in all sectors? (d) 136 km/hr (e) 115 km/hr
(a) 21 : 23 (b) 2 : 3 (c) 41 : 46
(d) 61 : 69 (e) 43 : 46 82. Neeraj invested Rs 10,000 at rate of interest 20% per
annum. The interest was compounded yearly for the
75. The average of male population of sector B and C is first two years and in the third year it was compounded
how much more or less than average of female half yearly. What will be the total interest earned at
population of these two sectors. the end of the third year?
(a) 40 (b) 50 (c) 60 (a) Rs 7224 (b) Rs 7324 (c) Rs 7424
(d) 70 (e) 80 (d) Rs 7624 (e) Rs 7824
76. ‘A’ invested Rs. 5,000 more than that of ‘B’, while ratio 83. A natural number when increased by 50% it gives a
of investment period for ‘A’ and ‘B’ is 2 : 1. If out of total natural number. However, when the value of the
profit of Rs. 2450, ‘A’ got Rs. 1750 then find total number is reduced by 75%, the number is still natural.
investment done by ‘A’ and ‘B’ together 2
If on reducing the number by 66 %, number is still
(a) Rs. 20,000 (b) Rs. 45,000 (c) Rs. 25,000 3
(d) Rs. 30,000 (e) Rs. 40,000 natural than least number that could be −
(a) 6 (b) 18 (c) 12
77. Neeraj’s income is 30% more than Veer’s income (d) 24 (e) 30
which is Rs.6000 more than Satish’s income. If ratio
between Neeraj’s income to Satish’s income is 39 : 20, 84. In an infinite G.P. (geometric progression), common
then find Veer’s income ratio is 6.25% of the first term and 5th term is 50% less
(a) Rs. 9,000 (b) Rs. 12,000 (c) Rs. 18,000 than the 4th term then find the sum of given G.P.?
32 34
(d) Rs. 15,000 (e) Rs. 21,000 (a) 12 (b) (c)
3 3
78. The ratio of speed of boat in still water to the speed of (d) 16 (e) Can’t be determined
stream is 5 : 3. A boat takes total 12 hours to go 48 km 85. A sum of Rs. 54.60 is comprised of 153 coins that are
in upstream and same distance in downstream. Find either 20 paise or 50 paise. Find the number of 20 paise
the speed of boat is still water. coins
1
(a) 5 km/h (b) 7 km/h (c) 10 km/h (a) 80 (b) 43 (c) 27
2
(d) 15 km/h
1
(e) 122 km/h (d) 63 (e) None of these
86. A bird covers a distance of 112 km in ‘y’ hours travelling
79. Abhi invested Rs x at the rate of 15% p.a. on CI for two
at the speed of ‘x’ km/hr. Find the speed of bird, given
years and gets total interest of Rs 5805. If Abhi
that x and y are co-primes integers, and both are
invested Rs (x + 14000) for another 2 yrs at an
greater than 1.
additional rate of 15%, then what will be the CI on that
(a) 16 km/hr
investment?
(b) Can’t be determined
(a) Rs 22080 (b) Rs 24000 (c) Rs 25200
(c) 14 km/hr
(d) Rs 26080 (e) Rs 20080
(d) 28 km/hr
80. Three friends P, Q and R have amount in the ratio of 5 (e) 7 km/hr
1
: 6 : 8. If R gives 37 2 % of his amount to P then the 87. A shopkeeper buys rice from wholesaler at 30%
difference of the amount left with R and the average discount on marked price. If shopkeeper gives
of the amount of P and Q is Rs 560. Find the ratio of discount of 20% on marked price while selling the
their initial amount after Rs 1000 is added to each of same rice, find his net profit/loss percentage.
them. 1
(a) 122%
2
(b) 147%
2
(c) 163%
(a) 60 : 66 : 83 (b) 67 : 72 : 81 (c) 60 : 67 : 81 2 2
(d) 60 : 69 : 81 (e) 65 : 71 : 83 (d) 21 % (e) 23 %
7 7

98 www.bankersadda.com | www.sscadda.com | www.careerpower.in | Adda247 App


The IBPS RRB PO & Clerk 2021 | SUCCESS GUIDE
88. By selling 6 dozen bananas, a man is losing selling Directions (96-100): In the following questions two
price of 1 dozen bananas. Find his percentage loss. quantities are given for each question. Compare the
3 1 1 numeric value of both the quantities and answers
(a) 147% (b) 143% (c) 122%
2 2 accordingly.
(d) 147% (e) 143%
96. A certain sum of money amounts to Rs. 2613 in 6 years
89. A person converted his circular field into a square field at 5% per annum.
by removing some area of circular field. Find the area Quantity I: No. of years in which the same amount
of square field if area of the circular field is 288 π m² becomes Rs. 3015 at same rate.
and diameter of circular field is equal to diagonal of Quantity II: 11 years.
square field. (a) Quantity I > Quantity II
(a) 484 m² (b) 576 m² (c) 529 m² (b) Quantity I < Quantity II
(d) 512 m² (e) 578 m² (c) Quantity I ≥ Quantity II
90. If ratio of length, breadth and height of a cuboid is 1 : (d) Quantity I ≤ Quantity II
2 : 3 and its area is 88 cm², then find the volume of a (e) Quantity I = Quantity II or No relation
cube having edge length equal to the breadth of the 97. Quantity I : A sphere of diameter 13.4 cm is melted
cuboid. and cast into a right circular cone of height 26.8 cm.
(a) 68 cm³ (b) 44 cm³ (c) 64 cm³ The radius of the base of the cone is ?
(d) 74 cm³ (e) None of these Quantity II: 5.95 cm.
(a) Quantity I > Quantity II
91. The probability of Ram speaking truth is ⅓ and that of
(b) Quantity II > Quantity I
Shyam is ⅖. A question is asked from both of them.
(c) Quantity I ≥ Quantity II
What is the probability that one of them lies ?
7 6 5 (d) Quantity II ≥ Quantity I
(a) (b) (c) (e) Quantity I = Quantity II or relation can’t be
15 15 15
8
(d) 15 (e) None of these established.

92. In how many difference ways, the letters of the word 98. A, B and C complete a task and receive Rs. 8880. They
‘UBUNTU’ can be arranged so that vowels come work for equal number of days. Five times of A’s
together? efficiency is equal to four times of B’s efficiency which
6! is equal to six times of C’s efficiency.
(a) 4!× 3! (b) 3! (c) 24
Quantity I: 2400
(d) 3!× 3! (e) None of the above Quantity II: Amount received by C.
93. If X men can do a work in (X+16) days and (2X+32) men (a) Quantity I > Quantity II
can do same work in 8 days then find in how man days (b) Quantity I < Quantity II
(X-8) men can do half of the work. (c) Quantity I ≥ Quantity II
(a) 32 days (b) 16 days (c) 64 days (d) Quantity I ≤ Quantity II
(d) 48 days (e) 8 days (e) Quantity I = Quantity II or No relation

94. The product of the age of Bhagat and Rahu is 240. If


twice the age of Rahu is more than Bhagat’s age by 4
years. Then what is the age of Rahu?
(a) 12 years (b) 10 years (c) 15 years
(d) 8 years (e) 20 years
95. Two vessels contain liquid A and liquid B in the ratio 3:7
and 3:2 respectively. In what ratio should the contents
of these two vessels be mixed such that the final ratio
of liquid B and liquid A in the resultant solution
becomes 23:17.
(a) 5 : 3 (b) 4 : 3 (c) 6 : 5
(d) 7 : 5 (e) 7 : 4

99 www.bankersadda.com | www.sscadda.com | www.careerpower.in | Adda247 App


The IBPS RRB PO & Clerk 2021 | SUCCESS GUIDE
99. A solution of acid and water contains 20% acid in it. 100. A person lends ⅓rd of his money at 15% while the rest
Quantity I: Percentage of water that must be at 18% per annum rate at simple interest.
evaporated from solution to get a 50% acid solution. Quantity I: His annual rate of interest on the whole
Quantity II: 80% sum.
(a) Quantity I > Quantity II Quantity II: 16%.
(a) Quantity I > Quantity II
(b) Quantity I < Quantity II
(b) Quantity I < Quantity II
(c) Quantity I ≥ Quantity II
(c) Quantity I ≥ Quantity II
(d) Quantity I ≤ Quantity II (d) Quantity I ≤ Quantity II
(e) Quantity I = Quantity II or No relation (e) Quantity I = Quantity II or No relation

Solutions
IBPS RRB PO PRELIMS
3 4750
1. (a): 8 × 4200 + 625 + 1000 – 700 = (? )2 9. (a): 250 + 283 – 157 + 24 + 100 =
?
(?)² = 1575 + 625 + 1000 – 700 = 2500 4750
?= 500
= 9.5
? = 50
?+62.5 10. (d): 2250 + 270 – 20 + 125 + ? = 3600
2. (c): + 360 – 144 – 20 = 169 + 87
3 ? = 975
? + 62.5 = (169 + 87 + 144 + 20 – 360) × 3
? = 180 – 62.5 = 117.5 11. (e): (9)3−1 × 6 + 343 + 171 = −331 + (? )3
1000 + 331 = (?)³
3. (d): 270 + 121 – √? = 361 ?= 11
√? = 30
45×2770 5
? = 900 12. (b): 100
+4 × 1824 = 5 ×?

4. (b):
624
+ 21 + 27 = 36 + 220 ? = 705.3
?
624 42×350
= 208 13. (e): 25 + 168 − =?
? 100
624
? = 208 = 3 ? = 46

? 1 1 1 1 1 2
5. (a): 100 × 625 + 3 × 750 + 11 × 5500 – 25 = 1000 14. (d): (1 + 2 − 3) + (3 + 6 − 9) = ?
7 2
? × 6.25 + 250 +500 – 25 = 1000 =
18 ?
? × 6.25 = 275 36 1
?= = 5
? = 44 7 7

6. (d): 1645 + 1500 + ? = 3600 15. (a): 308 – 41 = ? – 11


? = 455 ? = 278
? 16. (d):
7. (e): 2450 – 1540 + 1700 – 710 = 100 × 1900
? × 19 = 1900
? = 100
2 1
8. (c): 5 × 1325 + 299 + 271 = 3 × 3300 + ? 17. (a):
530 + 299 + 271 = 1100 + ?
?=0

100 www.bankersadda.com | www.sscadda.com | www.careerpower.in | Adda247 App


The IBPS RRB PO & Clerk 2021 | SUCCESS GUIDE
18. (e): 25. (c): Here,
6× 1+1=7
7 × 2 + 2 = 16
16 × 3 + 3 = 51
51 × 4 + 4 = 208
208 × 5 + 5 = 1045
19. (d): 1045 × 6 + 6 = 6276
Hence, the question mark should be replaced by
1045.
2,5,11,17 and 23 are alternate prime numbers. 26. (d): Let total watches manufactured in year 2017
20. (b): = 700x
So,
(28% - 20%) of 700x = 11200
7 × 8x = 11200
21. (b): 59.76 – 1.1 = 58.66 x = 200
58.66 – 2.2 = 56.46 Number of type A watches manufactured in year
56.46 – 4.4 = 52.06 2018
120 12
52.06 – 8.8 = 43.26 = 100 × 700 × 200 × 100 = 20160
43.26 – 17.6 = 25.66 27. (a): Let, total no. of watches in 2017 be x.
Hence, the question mark should be replaced by 1 (12+28)
Then, 2 [ × x] = 16100
43.26. 100
1 2
⇒ × x = 16100
22. (d): 36 + 121 = 157 2 5
or, x = 16100 × 5
157 + 144 = 301 16100×5
Total no. of watches in 2018 = × 8 = 92000
301 + 169 = 470 7
470 + 196 = 666 28. (c): Total no. of watches in 2018 = 2568 ×
100
25
666 + 225 = 891
= 25600 × 4
Hence, the question mark should be replaced by
= 102400
666. Total no. of watches in 2017 = 89600
23. (b): Here, No. of watches of type B and D in 2017
(12+20)
27 = 33 = × 89600 = 28,672
100
125 = 53
29. (b): Let the total number of watches manufactured in
343 = 73
2017 and 2018 is 7x and 8x respectively
1331 = 113 25
×7𝑥 5×7 35
2197 = 133 Required ratio = 100
30 = 6×8 = 48
×8𝑥
4913 = 173 100

Hence, the question mark should be replaced by 30. (c): Let total no. of watches manufactured in 2017
2197 = 700
Then, total no. of watches manufactured in 2018
24. (c): 102 – 3 = 99
= 800
99 + 5 = 104 No. of all type of watches except type B in 2017
104 – 7 = 97 80
= 100 × 700 = 560
97 + 9 = 106
No. of type B watches in 2018 = 800 – 560 = 240
106 – 11 = 95 (240 –140)
Hence, the question mark should be replaced by Required percentage change = 140
× 100
5
95. = × 100 = 71%
7

101 www.bankersadda.com | www.sscadda.com | www.careerpower.in | Adda247 App


The IBPS RRB PO & Clerk 2021 | SUCCESS GUIDE
31. (c): From (i) and (ii) 37. (b): (i) x² – 3x – 5x + 15 = 0
⇒x=1 (x – 3) (x – 5) = 0
2
⇒y=− x = 3, 5
5
𝑖. 𝑒. x > y (ii) y² – 3y + y – 3 = 0
y(y – 3) + 1 (y – 3) = 0
32. (b): I. 4x2–7x–15=0
⇒ 4x2–12x+5x-15=0 (y + 1) (y – 3) =0
⇒ 4x (x-3)+5(x-3)=0 y = –1, 3
⇒ (4x+5)(x-3)=0 x≥y
5
x=-4 ; x=3 38. (d): (i) 𝑥 2 + 2√3𝑥 + 3√3𝑥 + 18 = 0
II. -33y =(9y2+18) 𝑥(𝑥 + 2√3) + 3√3 (𝑥 + 2√3) = 0
⇒9y2+18+33y=0
(𝑥 + 3√3)(𝑥 + 2√3) = 0
⇒ 9y2+27y+6y+18=0
⇒ (y+3)(9y+6)=0 𝑥 = −3√3, −2√3
−6
Y=–3 ; y= 9 = 3
−2 (ii) 𝑦 2 + 2√3 𝑦 − 3√3 𝑦 − 18 = 0
Relationship can’t be established (𝑦 + 2√3)(𝑦 − 3√3) = 0
3 5 𝑦 = −2√3, 3√3
33. (e): I. x+2𝑥=2 y≥x
2𝑥 2 +3 5
= 39. (c): (i) 2x² – x – 3 = 0
2𝑥 2
4x2+6=10x (x + 1) (2x – 3) = 0
4x2–10x+6=0 3
𝑥 = −1,
4x2–4x–6x+6=0 2
(4x-6) (x-1)=0 (ii) 4y² – 24y + 35 = 0
⇓ ⇓ 4y² – 14y – 10y + 35 = 0
+3 5 7
x= 2 x=1 𝑦= ,
2 2
3
II. √𝑦 – =0⇒y=3 y>x
√𝑦
𝑖. 𝑒. y > x 40. (a): (i) 2x² – 3x – 4x + 6 = 0
34. (a): I. x = √289 x(2x – 3) – 2 (2x – 3) = 0
⇒ x = 17 (x – 2) (2x – 3) = 0
3
II. y2 – 34y + 137 = –152 𝑥 = 2, 2
⇒ y2 – 34y + 289 = 0 (ii) 2y² – 2y + y – 1 = 0
⇒ (y – 17)2 ⇒ y = 17 2y(y – 1) + 1 (y – 1) = 0
i.e. x = y (2y + 1) (y – 1) = 0
35. (a): I. x2 + 10x + 25 = 0 1
y= −2,1
⇒ (x + 5)2 = 0
x>y
⇒ x = -5
II. y2 = 25 41. (d): Let population of city P and R be 6x and 4x
y=+5 respectively.
i. e. x < y People who like apple from cities P and R together
6x×35 4x×25
36. (e): x² – 12x + 11x – 132 = 0 = + 100
100
x(x – 12) + 11 (x – 12) = 0 210x 100x 310x
= 100 + 100 = 100
(x + 11) (x – 12) = 0
x = –11, 12 People who like Grapes from these two cities
(ii) y² – 14y – 8y + 112 = 0 together
35×6𝑥 30×4𝑥 330𝑥
y(y – 14) – 8(y – 14) = 0 = + = 100
100 100
(y – 8) (y – 14) = 0 310𝑥
100
y = 8, 14 Required ratio = 330𝑥 = 31 : 33
No relation can be established between x & y. 100

102 www.bankersadda.com | www.sscadda.com | www.careerpower.in | Adda247 App


The IBPS RRB PO & Clerk 2021 | SUCCESS GUIDE
42. (a): Let the population of cities P, Q, R and S be 6x, 2x, 49. (b):
4x and 5x respectively
30 45
(2𝑥× +4𝑥× )
100 100
Required% = 30 30 × 100
(6𝑥× +5𝑥× )
100 100
240𝑥
8 50. (e):
100
= 330𝑥 × 100= 72 11 %
100

43. (b): Let population of cities Q and S be 2x and 5x


respectively. 51. (e): 3028 + 672 – 40% of ?+ (10)3 = (60)² – √10000
2𝑥×25 50𝑥
People from city Q who like grapes = 100 = 100 40×?
3700 + 1000 + 100 – 3600 =
5𝑥×30 150𝑥 100
People from city S who like apple = 100 = 100 ? = 3000
150𝑥 50𝑥
( – )
100 100 127.5+?
Required % = 150𝑥 × 100 52. (b): + 25 × 40 – √256 = (32)2
100 8
𝑥×100 127.5+?
= 150𝑥
× 100 = 66 ⅔% + 1000 – 16 = 1024
8

44. (a): Let population of city P, Q, R and S be 6x, 2x, 4x 127.5 + ? = 40 × 8


and 5x respectively. ? = 320 – 127.5
ATQ, ? = 192.5
5𝑥×40
= 7200 53. (c): √1225 + √2025 + √3025 – √100 = (? )3
100
⇒ x = 3600 35 + 45 + 55 – 10 = (?)³
1
Required average = [6 × 3600 + 2 × 3600 + (?)³ = 125
3
4 × 3600] ?=5
1
= 3 [43200] 54. (c): 560 + 70 = 630
= 14,400
55. (a): 35 × 40 + √961– (29)² – 50% of ? = (23)²
45. (e): Let population of city P, Q, R and S be 6x, 2x, 4x ?
and 5x respectively 1400 + 31 – 841– 2 = 529
?
Total number of people who like oranges from all 590 – 529 = 2
the cities
6𝑥×30 2𝑥×30 4𝑥×45 5𝑥×30 ? = 61 × 2
= + + + ? = 122
100 100 100 100
570𝑥
= 1 3
100 56. (b): √16906 ÷ 25.90 + ? × 19.85 × 4.96 = √2198 ×
570𝑥
Required % = 100×10𝑥 × 100 = 57%
4.88
1 1
46. (e): 130 × 26 +?× 20 × 5 = 13 × 5
5 + 4? = 65
? ≃ 15
3
57. (e): √21950 ÷ 7.08 + √910 = ? +(13.962 −
106.86 × 2)
1
47. (a): 28 × + 30 =? +(196 − 214)
7
34 = ? – 18
? ≃ 52
48. (e): 58. (d): (27.62)2 + (9.10)3 + √7225 = (? )2
(28)2 + (9)3 + 85 = (? )2
784 + 729 + 85 = (? )2
√1598 ≃?
? ≃ 40

103 www.bankersadda.com | www.sscadda.com | www.careerpower.in | Adda247 App


The IBPS RRB PO & Clerk 2021 | SUCCESS GUIDE
59. (b): 131.9% of 800 – 12.5% of 2048 = 3.98 of ? Total marks obtained by Y = 200 + 120 = 320
132 × 8 − 256 = 4 ×?
800 ≃ 4 ×?
? ≃ 200
24.96 493.92 1 1
60. (a): 14
𝑜𝑓 195.984 − 201.05 × 25 ÷ 2025 =?
25 494 1
× 196 − × × 2025 = ?
14 200 25
350 − 2.5 × 81 ≃ ?
? = 150
61. (d): Total animals in Kaziranga National park 66. (c): Let marks obtained by X in Chemistry be a.
= 1120 + 960 = 2080 ATQ,
150
Smaller animals in Guindy National park = 1680 a + 100 a = 240
2080–1680 a = 96
Required % = × 100
1680
500 Required difference = 96 × 1.5 – 120
= 21 % ≈ 24% = 144 – 120 = 24
62. (a): Let number of Elephant, Giraffe, Rhino and 200
67. (a): Required % = 300 × 100 = 66 ⅔%
buffalo be a–d, a, a+d, a+2d respectively.
ATQ, 68. (c): Cannot be determined
a – d + a + a + d + a + 2d = 1080 320
69. (c): Required % = 600 × 100 = 53 ⅓%
⇒ 4a + 2d = 1080 ⇒ 2a + d = 540 …(i)
and, 70. (e): Marks obtained by Z in Physics = 180 × ⅔ = 120
𝑎–𝑑 17 120+160
= ⇒ 2a = 53d …(ii) Required average = = 140
𝑎+2𝑑 19 2
From (i) and (ii) 71. (b): Let the speed of current be x km/hr.
d = 10 And speed of boat in still water = (x + 5) km/hr
a = 265 ATQ,
275 2𝑥+5 4
Required% = 1100 × 100 = 25% =3
𝑥+5
63. (c): Smaller animals except deer in Kaziranga ⇒ 6x + 15 = 4x + 20
75 ⇒ 2x = 5 ⇒ x = 2.5 km/hr
= 100 × 1120 = 840
Speed of boat in downstream = (2x + 5) km/hr = 10
840
Required ratio = =1 ∶1 km/hr
840
Required distance = 10 × 3 = 30 km/hr
(1280+1080)+(1100+880)
64. (e): Required average = = 2170
2 72. (d): Let, length of train A be 2x m and speed be y m/s.
65. (b): Required difference = (880 + 1680) – (1080 + 1100) Then, length of train B is x m and speed of train B
= 380 is 2y m/s.
ATQ,
Solutions (66-70): Marks obtained X in Physics = 160 2x
160
=4
y
Total marks obtained by X = 40
× 100 = 400 (2x+x) 3x 3×2y
Required time = (2y–y) = = y = 6s.
Marks obtained by X in Chemistry & Maths together y
= 400 – 160 = 240 73. (b): Let the length of train is X m.
400
Total marks obtained by Z = 4
× 3 = 300 When they will travel in opposite directions, total
Marks obtained by Z in Chemistry =
3
× 160 = 120 distance travelled = 2X.
4 and their speed will add up.
Marks obtained by Z in Physics and Maths = 300 – 120 5
Resultant speed = 18 × 72 + 40ms¯¹
= 180
Marks obtained by Y in Maths = 120 = 20 ms¯¹ + 40 ms¯¹ = 60 ms¯¹.
Marks obtained by Y in Physics and Chemistry together ATQ,
2X 300
=
400
= 200 = 5 ⇒ X= = 150 meters
60 2
2
104 www.bankersadda.com | www.sscadda.com | www.careerpower.in | Adda247 App
The IBPS RRB PO & Clerk 2021 | SUCCESS GUIDE
74. (b): Distance covered by walking = 12 km 79. (a): Total price = 15000 × 3 + 1500 × 3
Distance covered by Run = 84 km = Rs. 45000 + 4500
12 84
Required time = 8 + 12 = 8.5 hr = Rs. 49500
6
Price of article ‘Z’ = 15 × 49500
75. (d): Let speed of boat in still water be x km/hr and = Rs. 19,800
speed of stream be y km/hr.
80. (d): Let Y be ‘a’
ATQ,
164 150 164
So. X = 2a
=
(x–y) 100
× (x+y) And Z = 6
7𝑎

2 (x + y) = 3 (x – y) (𝑎+2𝑎+ )
7𝑎
120
6
∴ x = 5y U = 3
× 100
5𝑎
∴ x = 5 × 10 = 50 km/hr U= 3
100 5𝑎
∴ Required time = 50
= 2 hr. 2a − = 240
3
a = 720
76. (d): Total amount spent by man 720×5
X X X 2×720+
3
=2 + 120 + [50% of X = ] Required average =
15 2 2
2640
Money left with him = 400 = 2
= 1320
X X
X– [2 + 120 + 15] = 400 81. (e): Let present age of Anurag is ‘x’ years
15X+2X ATQ—
X –[ ] = 520 x
30
13𝑥 x – 10 = 8 +
2
⇒ = 520 ⇒ x = 1200
30 2x – 36 = x
X 1200
∴ money spent on dessert = = = 80 x = 36 years
15 15
Let Mohit’s age = y
77. (a): The person who got highest share ATQ—
6 36 –4 8
= 13 × 4004 =7
y –2
= 6 × 308 224 = 8y – 16
= Rs. 1848 y = 30 years
The person who got lowest share Let Ayush’s age = z
3
= 13 × 4004 ATQ—
30+6 3
=2
= 3 × 308 = 924 z+6
Required difference = 1848 – 924 = Rs. 924 72 = 3z + 18
z = 18 years
78. (e): Let Original number = 10x + y Required difference = 36 – 18 = 18 years
So reversed = 10y + x 13
82. (d): Total milk in vessel = 180 × 18 = 130𝑙
ATQ,
5
10x + y – (10y + x) = = 45 Total water in vessel = 180 × = 50 𝑙
18
13
9x – 9y = 45 Remaining milk = 130 – 54 × 18 = 91 𝑙
x–y=5 5
Remaining water = 50 – 54 × 18 = 35 𝑙
and
ATQ –
x = 3 + y²
Let milk and water added in resulting mixture be x
y² + 3 – y = 5 and 3x respectively
y² – 2 – y = 0 91+𝑥 50
35+3𝑥
= 31
y = +2, –1
2821 + 31x = 1750 + 150x
So,
119x = 1071
y=2 x=9l
x=7 Amount of water added in remaining mixture
Original Number = 72 = 3 × 9 = 27 𝑙
105 www.bankersadda.com | www.sscadda.com | www.careerpower.in | Adda247 App
The IBPS RRB PO & Clerk 2021 | SUCCESS GUIDE
83. (e): Let amount invested in scheme A = x 88. (a): Let the length of smaller rectangle be ℓ₁ and
ATQ, length of larger rectangle be ℓ₂
15 18
x × 100 + 100 (28000 – x) = 4680 Breadth of both = side of square = √196 = 14 cm
3x ATQ,
⇒ = 5040 – 4680
100 2(ℓ₁ + 14) + 2 (ℓ₂ + 14) = 216
⇒ x = 12000
2(ℓ₁ + 14) +2 ℓ₂ + 28 = 216
Amount invested in scheme B ℓ
= 28000 – 12000 = 16000 2ℓ₁ + 28 + 3 21 × 2 + 28 = 216
5ℓ₁ = 216 – 56
84. (d): Ratio of Profit ⇒ ℓ₁ = 32 cm
32
ℓ₂= 3 × 2
= 48 cm
800 Required sum = 32×14 + 48×14
Profit of ‘C’ = × 12
50 = 14 × (80) = 1120 cm²
= 16 × 12 = Rs. 192
89. (d): A number divisible by 3 leads to 2 possibilities
85. (a): Let the cost price of pen be Rs. x. either it is divisible or not.
Then, cost price of pencil = Rs (x-8) 1
So, Required probability =
ATQ, 2
150 250 2 = Total cases [Either divisible or not]
x × 100 = (x − 8) × 100
1 = Favourable [number is divisible]
⇒ x = 20
C.P. of pen = Rs. 20 90. (c): Total number of handshakes = ⁿc₂
𝑛(𝑛–1)
2
= 78 ⇒ n = 13 person
86. (d): Let efficiency of B = 5x units/day
140
So, efficiency of A = 5x × 100 = 7𝑥 𝑢𝑛𝑖𝑡𝑠/𝑑𝑎𝑦 91. (a): Quantity I.
100×12
Total work = (5x + 7x) × 16 = 192𝑥 𝑢𝑛𝑖𝑡𝑠 Second no. = = 12
100
3 3 3
C & D take together = 16 × 4 = 12𝑑𝑎𝑦𝑠 ∴ first no. = 123 × 2 = 1728 × 2 =2592
192𝑥
Efficiency of C & D = 12 = 16𝑥 𝑢𝑛𝑖𝑡𝑠 ∴ Required sum = 12 + 2592 = 2604
192𝑥 Quantity I > Quantity II
Required days =
(16𝑥+5𝑥)
1 92. (c): Quantity I.
= 9 7 𝑑𝑎𝑦𝑠 15
Distance travelled by thief in 15 min =60 × 60
87. (a): Let P, Q and R takes time p, q and r hours = 15 km
respectively to complete the work alone Time taken by police to catch thief after 11:15 pm
ATQ, 15
1 1 1 1 ≥ 65−60 ≥ 3 hr
𝑝
+ + =
𝑞 𝑟 11
… (𝑖)
Quantity I ≥ Quantity II
After being opened for 6 hours, cistern remaining
1 5 93. (e): Quantity II.
to be filled = 1 − 6 × 11 = 11
Ratio of profit ⇒
Now,
8 8 5
A : B : C
+ 𝑟 = 11
𝑝 12 12 : 12  x : 8  (12 − x)
1 1 5
𝑝
+ = 𝑟 88
..(ii) 36 : 3x : 2(12 − x)
From (i) & (ii) ATQ,
1 1 5 36 1800
𝑞
= 11
− 88 = 3200
60+𝑥
1 3
= ⇒ 60 + x = 64
𝑞 88
88 1 ⇒x=4
Q alone can fill the cistern in = = 29 3 hrs
3 Quantity I = Quantity II

106 www.bankersadda.com | www.sscadda.com | www.careerpower.in | Adda247 App


The IBPS RRB PO & Clerk 2021 | SUCCESS GUIDE
94. (a): Quantity I. II. Let Q gets Rs. 𝑥 and (𝑃 + 𝑅) gets Rs. 4𝑥
Let speed of stream = y km/hr. ⇒ 𝑥 + 4𝑥 = 705
ATQ, ⇒ 𝑥 = 141
x –18 x
= 15+y … (i) P + R = 4 × 141 = Rs. 564
15 –y
Also, From I and II
15 + y – (15 – y) = 6 P = Rs. 282
2y = 6 Q = Rs. 141
y = 3 ...(ii) R = Rs. 564 – 282 = Rs. 282
From (i) and (ii) Q gets the least among all.
x –18
= 18
x So, both statements are necessary to answer the
12
questions.
x = 54 km
Quantity I > Quantity II 99. (d): From I –
Let radius & height of cylinder is 7x and 6x
95. (a): Quantity I:
Let C.P. of watch for P be Rs. 100 respectively
Amount paid by R ATQ –
22
= 120 × 100
90
7
× 49𝑥 2 × 6𝑥 = 7392
= Rs. 108 x = 2 cm
ATQ, Breadth of rectangle = 2 × 6 = 12 𝑐𝑚
108 → 2160 Given, 2(L + 12) = 80
1 → 20 L = 40 − 12
100 → 2000 L = 28 cm
C.P. of watch for P= Rs. 2000 From II –
Required price at which P sold to Q Side of square = a cm
120
= 2000 × Given, a2 = 196
100
= Rs. 2400 a = 14 cm
Quantity I > Quantity II Length of rectangle = 14 × 2 = 28 𝑐𝑚
So, either from statement I or statement II we can
96. (e): No marks are given in numbers.
determine the answer
So, we can’t find the marks obtained in
Mathematics. 100. (c): Total number of balls in the bag = (7 + y + x)
From I –
97. (a): I. Let the four consecutive even integer be 𝑦 1
𝑥, 𝑥 + 2, 𝑥 + 4 and 𝑥 + 6 =
(7 + y + x) 4
𝑥 + 𝑥 + 2 + 𝑥 + 4 + 𝑥 + 6 = 11 × 4 = 44 − 𝑥 + 3𝑦 = 7 --------- (i)
32
⇒𝑥 = =8 From II –
4
𝒙 𝟐
II. we can’t find from statement II =
(7 + y + x) 𝟓
So, statement I alone is sufficient to answer. 3x − 2𝑦 = 14 ------- (ii)
98. (c): I. Let P gets Rs. 2𝑥 and (Q + R) gets Rs. 3𝑥 From (I) & (II) –
⇒ 2𝑥 + 3𝑥 = Rs. 705 x=8,y=5
⇒ 𝑥 = Rs. 141 Total blue & yellow balls in bag = 8 + 5 = 13
P = 2 × 141 = Rs. 282 So, statement I and statement II together are
Q + R = 3 × 141 = Rs. 423 required to answered the question

107 www.bankersadda.com | www.sscadda.com | www.careerpower.in | Adda247 App


The IBPS RRB PO & Clerk 2021 | SUCCESS GUIDE
IBPS RRB CLERK PRELIMS

1. (a): ? = 2358 2 8 1 8
14. (c): 28 7 × 5 11 +? = 36 9 × 7 13
7525 3125 198 63 325 99
2. (c): + 5×5×5×5 – 210 =? 7
× 11 +? = 9
× 13
25
? = 301 + 5 – 210 = 96 ? = 275 – 162
1400 ? = 113
3. (e): 4
– 466 + 120 =?
255×272×153
?=4 15. (d): ? = 102×204×85
=6
(23+37)2 602 16. (c): ? = 29 + 170 – 115 = 84
4. (c): = (19+11)2 = = 4 = 22
302
?=2 40 44
17. (d): ?² = 100 × 420 + 100 × 200
64+16+4 1 = 168 + 88 = 256
5. (a): =
324+144+36 6
⇒ ? = ± 16
?= 6
20
6. (d): 25 + 210 = ? × 12 18. (b): ×? = 1098 ⇒ ? = 5490
100
25 (1+32)
=? 34(?+2) 33 ×32 ×33
12 19. (a): 4(?+2) = 3 2 3
⇒ ? = 88 10 10 ×10 ×10
4(?+2) 38
(0.3) = 108 = (0.3)8
7. (a): √256 + √784 =?× √121
16 + 28 = ? × 11 ⇒ 4(? + 2) = 8 ⇒ ? = 0
44
? = 11 = 4 20. (b): ? = 12 + 28 + 36 – 8 = 76 – 8 = 68

8. (d): 80% of 350 + 45% of 800 = ? × 256 21. (b): Total number of girls students from A & C together
60
280 + 360 = ? × 256 = 360 + 490 × 100 = 654.
640 490
? = 256 = 2.5 Total number of students from A = × 24 = 840
14
654 545 6
9. (e): 115% of 360 + 180% of 270 = ? × 225 Required % = 840 × 100 = 7 % = 77 7 %
414 + 486 = ? × 225 490
900
? = 225 = 4 22. (e): Boys from institute D = 14
× 18 – 280 = 350
490 30
34 ×96
Boys from institute E = 14 × 22 × 100 = 231
10. (b): 9? × 729 = 81 Required difference = 350 – 231 = 119.
? 92 ×96 3
9 = 92 ×93
=9 490 1
23. (a): Number of girls from institute B = 14
× 22 × 2
⇒?=3
= 385.
1 1 1 1 1
11. (d): 5 4 + 7 3 + 4 2 = 3 6 +? +5 6 Number of boys from institute D
490
1 1 1 1 1 = × 18 – 280 = 350
(5 + 7 + 4) + ( + + ) – 3– 5 – – =? 14
4 3 2 6 6 385
1 1 1 1 Required ratio = = 11 : 10
8 + 4 + 2 + 3–3 = ? 350
2+4 3
?= 8 + 8
= 84 24. (c): Required average
1 490 490 1
1 1 = 2 [24 × + 18 × ]=2 × [1470] = 735.
12. (b): 37 2 %of 300 + 62 2 % of 460 =? 14 14
75 125 25. (d): Total number of girls from B and C together
? = 200 × 300 + 200 × 460
490 1 60
225 575 = 22 × × + 490 ×
= 2
+ 2 = 400 14 2 100
= 385 + 294 = 679
13. (d): 75% × 160 + 45% × 300 = ? × 17 Total number of girls from D and E together
3 9 490 70
? × 17 = 4 × 160 + 20 × 300 = 280 + 22 × × 100 = 819.
120+135 255 14
?= = = 15 Required difference = 819 – 679 = 140.
17 17

108 www.bankersadda.com | www.sscadda.com | www.careerpower.in | Adda247 App


The IBPS RRB PO & Clerk 2021 | SUCCESS GUIDE
26. (c): 120 70 1000
36. (b): 100 × 120 + 100 × 80 ≈ ?× 100
144 + 56 ≈ ? × 10
? ≈20
37. (c): 2 + 5 × 7 – 432 + 532 ≈ ?
137 ≈?
27. (a):
42×22 20
38. (e): 7
+ 100 × 530 – 43 ≈ ?
132 + 106 – 43 ≈ ?
? ≈ 195
28. (d):
39. (b): 23 × 23 + 12 × 8 ≈ ?²
? ≈ 25
25×2001 25×2001 1
29. (e): 40. (a): 100
+ 100 – 3 × 100 ≈ ? +50
1000.5 – 33 – 50 ≈ ?
? ≈ 917

30. (b): 41. (a): 342 + 576 – ?² = (12)² – √100


918 – ?² = 144 – 10
?² = 918 – 134
?² = 784
? = 28
12
31. (c): Let pass percentage = x % 42. (e): 24 × ? + 100 × 225 = (19)² + √676
60
Total strength of school S = × 100 24 × ? + 27 = 361 + 26
𝑥
90
Total strength of school Q = × 100 24 × ? = 387 – 27
𝑥
9000 6000
– 24 × ? = 360
𝑥 𝑥
Required% = 6000 × 100 = 50% ? = 15
𝑥
36
32. (d): Number of failed student from school P 43. (b): (12)³ + ?² = × 5200
100
70 36
= 35 × 65 1728 + ?² = 100 × 5200
= 130 1728 + ?² = 1872
Pass student from school T = 100 ?² = 1872 – 1728
130
Required% = 100 × 100 = 130% ?² = 144
? = 12
33. (a): Total number of passed students
= 70 + 90 + 85 + 60 + 100 + 120 = 525 44. (a): 472 + 548 – ?² = (18)² + √400
525
Total Number of failed students = × 3 = 225 1020 – ?² = 324 + 20
7
?² = 1020 – 344
34. (e): Required difference
?² = 676
= (70 + 90 + 120 + 100 – 85 – 60) = 235
? = 26
35. (c): Let failed student in school R = y 96
So failed student in school U = y + 15 45. (e): 121 + ? = 100 × 500
ATQ, ? = 480 – 121
120+𝑦+15 3
=2 ? = 359
𝑦+85
On Solving 46. (b): Required% =
16+24 –8–20
× 100
y = 15 (16+24)
12
So, Total failed students = 45 = × 100 = 30%
40

109 www.bankersadda.com | www.sscadda.com | www.careerpower.in | Adda247 App


The IBPS RRB PO & Clerk 2021 | SUCCESS GUIDE
47. (d): Required difference 54. (e): I. x² + 2x – 35 = 0
[20+24 –12 –16] ⇒ x² + 7x – 5x – 35 = 0
= × 15000
100
= 2400 ⇒ x(x + 7) – 5 (x + 7) = 0
⇒ x = 5, –7
48. (a): It can be seen easily from the pie-chart that II. y² + 3y – 10 = 0
February month shows the highest percent
⇒ y² + 5y – 2y– 10 = 0
increase in income as compare to previous month
⇒ (y + 5) (y – 2) = 0
which is equal to
12 –8 ⇒ y= –5, 2
= × 100 ⇒ no relation can be established between x and y
8
4
= 8 × 100
55. (a): (i) 4x + 8y = 16
= 50% increment. 11
(ii) x + y = 4
49. (e): Required central angle Solving (i) and (ii)
18
= (20 + 16) × 5 ⇒ x = 3/2, y = 5/4
= 129.6° ⇒x>y
50. (c): Sandeep’s average income in starting four months 56. (e): I. 𝑥 2 + 32𝑥 − 273 = 0
(8+12+16+20)
= 4×100
× 15000 ⇒ 𝑥 2 + 39𝑥 − 7𝑥 − 273 = 0
= 2100 ⇒ 𝑥(𝑥 + 39) − 7(𝑥 + 39) = 0
Sandeep’s average income in Last four months ⇒ 𝑥 = 7 or –39
=
(16+20+20+24)
× 15000 = 3000 II. 3𝑦 2 − 44𝑦 + 161 = 0
4×100
Required difference = 3000 – 2100 = 900 ⇒ 3𝑦 2 − 21𝑦 − 23𝑦 + 161 = 0
⇒ 3𝑦(𝑦 − 7) − 23(𝑦 − 7) = 0
51. (d): I. x² + 13x +42 = 0 ⇒ (3𝑦 − 23)(𝑦 − 7) = 0
⇒ x² + 7x + 6x +42 = 0 23
⇒ 𝑦 = 7 or 3
⇒ 𝑥 = −7 𝑜𝑟 − 6
II. y² + 11y +30 = 0 𝑦≥𝑥
⇒ y² +6y + 5y +30 = 0 57. (d): I. 100x²–20x+1=0
⇒ 𝑦 = −5 𝑜𝑟 − 6 ⇒ 100x²–10x–10x+1=0
⇒y≥x ⇒ 10x(10x–1)–1(10x–1)=0
1
52. (b): I. x² – 9x – 90=0 ⇒ x = 10
⇒ x² – 15x + 6x – 90 = 0 II. 400𝑦 2 − 4 = 0
⇒ x = +15, –6 4
⇒ 𝑦 2 = 400
II. y² + 14y + 48 = 0
1
⇒ y² + 8y + 6y + 48 = 0 ⇒ 𝑦 = ± 10
⇒ y = –8, – 6 𝑥≥𝑦
x≥y
58. (a): I. 3𝑥 2 + 2𝑥 − 5 = 0
53. (c): I. 4x² + 16x + 15 = 0 ⇒ 3𝑥 2 + 5𝑥 − 3𝑥 − 5 = 0
⇒ 4x² + 10x + 6x + 15 = 0 ⇒ 𝑥(3𝑥 + 5) − 1(3𝑥 + 5) = 0
⇒ (2x + 5) (2x + 3) = 0 −5
5 3 ⇒𝑥= 3
𝑜𝑟 1
⇒x=– ,–
2 2 II. 2𝑦 2 + 𝑦 − 6 = 0
II. 3y² + 4y + 1 = 0
⇒ 2𝑦 2 + 4𝑦 − 3𝑦 − 6 = 0
⇒ 3y² + 3y + y + 1 = 0
⇒ 2𝑦(𝑦 + 2) − 3(𝑦 + 2) = 0
⇒ (y + 1) (3y + 1) = 0 3
⇒ y = –1, –⅓ ⇒ 𝑦 = −2 𝑜𝑟 2
⇒y > x Relationship can’t be established.

110 www.bankersadda.com | www.sscadda.com | www.careerpower.in | Adda247 App


The IBPS RRB PO & Clerk 2021 | SUCCESS GUIDE
59. (e): I. 2𝑥 2 + 31𝑥 + 120 = 0 66. (c):
2
⇒ 2𝑥 + 16𝑥 + 15𝑥 + 120 = 0
⇒ 2𝑥(𝑥 + 8) + 15(𝑥 + 8) = 0
−15
⇒ 𝑥 = 2 𝑜𝑟 − 8
II. 2𝑦 2 + 3𝑦 − 90 = 0
⇒ 2𝑦 2 + 15𝑦 − 12𝑦 − 90 = 0 67. (b):
⇒ 𝑦(2𝑦 + 15) − 6(2𝑦 + 15) = 0
−15
⇒ 𝑦 = 6 𝑜𝑟 2
𝑦≥𝑥
60. (d): I. 2𝑥 2 − 13𝑥 + 21 = 0
68. (c):
⇒ 2𝑥 2 − 6𝑥 − 7𝑥 + 21 = 0
⇒ 2𝑥(𝑥 − 3) − 7(𝑥 − 3) = 0
7
⇒ 𝑥 = 3 𝑜𝑟 2
II. 2𝑦 2 − 7𝑦 + 3 = 0
⇒ 2𝑦 2 − 6𝑦 − 𝑦 + 3 = 0 69. (d):
⇒ 2𝑦(𝑦 − 3) − 1(𝑦 − 3) = 0
1
⇒ 𝑦 = 2 𝑜𝑟 3.
𝑥 ≥ 𝑦.
70. (b):
61. (c): Series is

Alternate,
Solutions (71-75):
21
Population of sector A= 26000 × 52 = 10,500
16
Population of sector B= 26000 × 52 = 8,000
21
Population of sector C= 26000 × 52 = 7,500
62. (d): Series is Let number of females in sector A and number of males in
sector C be 9x and 8x respectively
Number of males in sector A= (10,500−9𝑥)
Number of females in sector C= (7,500-8x)
63. (b): Series is ATQ
(10,500−9𝑥)−(7,500 − 8x) = 2500
⇒𝑥 = 500
Number of females in sector A = 4,500
Number of males in sector A= 10,500-4,500= 6,000
Number of males in sector C= 4,000
64. (e): Series is Number of females in sector C= 7,500-4,000= 3,500
70
Number of females in sector B= 100 × 6,000 = 4,200
Number of males in sector B = 8000 - 4200= 3,800

65. (a): Series is Sector Males Female Totals


A 6,000 4,500 10,500
B 3,800 4,200 8,000
C 4,000 3,500 7,500

111 www.bankersadda.com | www.sscadda.com | www.careerpower.in | Adda247 App


The IBPS RRB PO & Clerk 2021 | SUCCESS GUIDE
6000+3800 Now,
71. (c):Required difference = 2
– 4,500 = 400
Required CI
68×4500
72. (e): Working females of sector A = 100 = 3060 30 2
= (18000 + 14000) [(1 + 100) − 1]
85×4200
Working females of sector B = 100 = 3570
= Rs 22,080
3570–3060
Required % = 3060 × 100 = 16 ⅔%
80. (c): Let P, Q and R have Rs 5x, 6x and 8x selectively
(6000+4000) 5 3
73. (a): Required % = × 100 = 55 9 % P = 5x + 8x × 8 = 8𝑥
18,000
(4500+4200+3500) Q = 6x
74. (d): Required ratio = (6000+3800+4000) R = 5x
12,200
= 13,800 = 61 : 69 ATQ,
7x – 5x = 560
75. (b): Required difference ⇒ x = 280
(4000+3800) (4200+3500)
=( 2
– 2
) = 50 Required ratio = (5×280+1000) : (6×280 + 1000) : (8
× 280 + 1000)
76. (b): Let amount invested by B = 𝑅𝑠. ′𝑥′
= 2400 : 2680 : 3240
Amount invested by ‘A’ = Rs. (𝑥 + 5000)
= 60 : 67 : 81
Ratio of profit sharing of A and B
= (𝑥 + 5000) × 2 ∶ 𝑥 × 1 81. (a): Difference of speed = 36km/h = 10m/s.
ATQ, Sum of length of both trains = 10×31.5 = 315m
1750 2𝑥+10,000
= 2𝑥+10,000+𝑥 When, running in opposite direction,
2450
5 2𝑥+10,000 315
⇒ = Sum of speed = = 35m/s.
7 3𝑥+10,000 9
⇒ 5 × (3𝑥 + 10,000) = 7 × (2𝑥 + 10,000) ATQ,
5 5
⇒ 15𝑥 + 50,000 = 14𝑥 + 70,000 𝑥× + (x + 36) × = 35
18 18
⇒ 𝑥 = 20,000 35×18
⇒ 2x + 36 = 5 = 126
Total amount invested by A and B together
= 20,000 + 20,000 + 5,000 = 𝑅𝑠 45,000 ⇒ x = 45 km/h.
and (x+36) = 81 km/h.
77. (c): Let Satish’s income = Rs. x
sum of speed of both trains= 81+45 = 126 km/h.
Then Veer’s income =Rs. (x+6000) 315
And Neeraj’s income = Rs. (1.3x + 7800) Or sum of speed = = 35m/s = 126 km/hr.
9
ATQ, 82. (c): Interest in first two years
1.3𝑥+7800 39
𝑥
= 20 20 2
= 10,000 [1 + 100] − 10000 = Rs. 4400
⇒ 26𝑥 + 1,56,000 = 39𝑥
1,56,000 Interest on third year
⇒ 𝑥 = 13 = 𝑅𝑠 12,000
10 2
Veer’s income = Rs. 12,000+6000 = Rs. 18,000 = Rs 14,400 × (1 + 100) − 14,400 = Rs 3024

78. (e): Let the speed of boat in still water be 5x km/hr and Total interest = Rs. 7424
that of stream be 3x km/hr. 83. (c): Let number is N
ATQ, So,
48 48
+ =12
8𝑥 2𝑥
ATQ
48+192 3
⇒ 8𝑥 =12 2
𝑁 = Natural …(i)
⇒ x = 2.5 𝑁
⇒ 4 = Natural …(ii)
Speed of boat in still water = 5x = 12.5 km/hr 1
⇒ 𝑁= Natural …(iii)
3
79. (a): ATQ, From these three equation we can conclude that
15 2
5805 = x[1 + 100
] −𝑥 the least number contain 3 × 4
⇒ x = Rs 18,000 Least number = 12

112 www.bankersadda.com | www.sscadda.com | www.careerpower.in | Adda247 App


The IBPS RRB PO & Clerk 2021 | SUCCESS GUIDE
84. (d): Let first term = 16x = a Diagonal of square = 𝑎√2 = diameter of circle
16x×6.25
So common ratio = =x=r = 2 × 12√2 = 24√2
100
So, ⇒ a = 24 m (where a = side of square)
4th term → 16x × x³ = 16x⁴
Area of square = (a)² = (24)² = 576 m²
and 5th term → 16x⁵
ATQ, 90. (c): Let, length, breadth and height be x, 2x and 3x
16𝑥 5
× 100 = 50 respectively
16𝑥 4
1
x=2 Then,
⇒a = 8 2(x×2x + 2x×3x + x×3x) = 88
1
⇒r = 2 or, 2(2x² + 6x² + 3x²) = 88
8
Sum of G.P. = 1 = 16 or, 22x² = 88
1–
2
or, x² = 4
85. (e): Let number of 20 paise coins = x ⇒x=2
And number of 50 paise coins = y
Volume of cube = (2x)³ = (4)³ = 64 cm³
Now,
x + y = 153 …(i) 1 3 2 2
91. (a): Required probability = 3 × 5 + 3 × 5
20x + 50y = 5460 …(ii)
7
Solving (i) and (ii) = 15
x = 73 and y = 80
86. (b): ATQ, 92. (c): On keeping the vowels together, we get BNTUUU
112
= y ⇒ 112 = xy Number of arrangements
x
Now x and y can be written as = 4!
(2, 56) (4, 28) (8, 14) (16, 7) or = 24
(56, 2) (28, 4) (14, 8) (7, 16)
Hence two possible co-prime pairs are possible, 93. (a): ATQ,
i.e. (7, 16) or (16, 7) (𝑋)(𝑋 + 16) = (2𝑋 + 32)(8)
Hence it can’t be determined 𝑋 = 16
87. (b): Let, he buys 1000 gm rice at Rs. 1000 i.e. Rs. 1/gm. Total work = (𝑋)(𝑋 + 16) = 16 × 32 = 512
70 512
CP for shopkeeper = 100 × 1000 = Rs. 700 Required time = 2(16−8) = 32 𝑑𝑎𝑦𝑠
80
SP for shopkeeper = 100 × 1000 = Rs. 800
(800 –700) 2 94. (a): Let age of Bhagat be ‘b’ year and Rahu be ‘a’ year
Profit % = × 100 = 147%
700 ATQ,
88. (d): 6SP = 6CP – SP 𝑏 × 𝑎 = 240 …(i)
or, 7SP = 6 CP 2a = b + 4 …(ii)
𝑆𝑃 6
or, = From (i) & (ii)
𝐶𝑃 7
1 2
% loss = 7 × 100 = 147% (2a- 4) ×a = 240
89. (b): a = 12 year

95. (d): By using alligation,

Area of circle = 288 π m²


⇒ πr² = 288 π
⇒ r = 12 √2 m.
113 www.bankersadda.com | www.sscadda.com | www.careerpower.in | Adda247 App
The IBPS RRB PO & Clerk 2021 | SUCCESS GUIDE
96. (b): I = A – P A : B : C = 12 : 15 : 10
𝑃×5×6 10
= 2613 – P Amount of C = × 8880 = 2400
100 37
30P 130P Quantity I = Quantity II
or, 2613 = P + 100 = 100
or, P = 2010 99. (b): Let, initial amount of acid = 20 ℓ
Interest when amount becomes 3015 Initial amount of water = 80 ℓ
= 3015 – 2010 = 1005 For this 20 ℓ to be 50% of solution,
5 Total solution must be 40ℓ
1005 = 2010 × 100 × n
Final amount of water = 20ℓ
or n = 10 years. (80–20)
Quantity I < Quantity II Required% = 80
× 100
60
= 80 × 100 = 75%
97. (a): Quantity I:
4 1 Quantity I < Quantity II
× 𝜋 × 6.7 × 6.7 × 6.7 = × 𝜋 × 𝑟 2 × 26.8
3 3
⇒ r = 6.7 cm 100. (a): Let, total money be x.
𝑥 15 2𝑥 18
Quantity II: 5.95 cm Then, total interest = 3 × 100 × 1 + 3
× 100 × 1
Quantity I > Quantity II 5
= 𝑥 (100 + 100)
12

98. (e): ATQ, 17


= 𝑥
100
5A = 4B = 6C
Hence, 17%
5A 5A
A:B:C=A ∶ ∶ Quantity I > quantity II
4 6

114 www.bankersadda.com | www.sscadda.com | www.careerpower.in | Adda247 App


The IBPS RRB PO & Clerk 2021 | SUCCESS GUIDE

Most Important Questions | Reasoning Ability | IBPS RRB PO &


CLERK PRELIMS
IBPS RRB PO PRELIMS

Directions (1-5): Study the following information carefully The one who likes pink colour sits second to the right of U.
and answer the given questions: One person sit between R and the one who likes Pink
Twelve players are sitting in two parallel rows each colour. The one who sits immediate right of R sits opposite
containing six players in such way that they are equidistant to the one who likes Green colour. The one who likes white
from each other. In row 1- S, T, U, V, W and X are sitting colour sits opposite to the one who likes Orange colour.
and facing North. In row 2- J, K, L, M, N and O are sitting Immediate neighbors of R do not like white. The one who
and facing South, but not necessarily in the same order. likes grey colour is immediate neighbor of the one who
Each player who sits in first row faces another player of the likes green colour. Q sits second to the right of V, who sits
second row. immediate left of T. Q does not like orange. P and W sits
Three players sit between U and V. The one who faces V immediate left to each other. W is not immediate neighbor
sits third to the right of L. M faces the one who sits third to of Q. One person sit between P and the one who likes blue
the right of S but does not face U. Immediate neighbor of
colour. W likes yellow colour and faces outside.
K faces S. Only one player sits between J and N, who faces
the one who sits at the immediate left of W. Neither W nor 6. Who among the following person likes orange colour?
X faces L. O does not sit at any of the extreme ends. (a) Q (b) T (c) S
1. Who among the following sits between N and J? (d) U (e) None of these
(a) L (b) K (c) O 7. Who among the following sits second to the right of T?
(d) M (e) None of these (a) U (b) R (c) V
2. Who among the following faces W? (d) W (e) None of these
(a) J (b) M (c) N
(d) K (e) O 8. Who among the following person sits immediate right
of the one who likes green colour?
3. Who among the following sits to the immediate left of (a) T (b) V (c) S
O? (d) Q (e) None of these
(a) N (b) M (c) J
(d) K (e) None of these 9. How many persons sits between T and W when
4. Four of the following five belong to a group in a certain counted to the left of T?
way, which among the following does not belong to (a) One (b) Two (c) Four
that group? (d) Three (e) None
(a) V (b) N (c) U 10. Four of the following five are alike in certain way based
(d) M (e) X
from a group, find the one that does not belong to that
5. How many players sit to the right of T? group?
(a) Five (b) Two (c) One (a) R (b) T (c) U
(d) Four (e) Three (d) Q (e) P
Directions (6-10): Study the information carefully and Directions (11-15): Study the information carefully and
answer the questions given below.
answer the questions given below.
Eight persons P, Q, R, S, T, U, V and W are sitting in a
Eight persons A, B, C, D, E, F, G and H are sitting in a
square table in such a way that the persons sitting at
corner face towards the center and the persons sitting at circular table in such a way that all of them face towards
middle of side face outside the center. Each of them likes center. All of them likes different colours i.e Red, Green,
different colour i.e. Red, Blue, Green, Pink, Yellow, Blue, Yellow, Orange, Pink, White and Grey but not
Orange, Grey and White but not necessary in same order. necessary in the same order.
115 www.bankersadda.com | www.sscadda.com | www.careerpower.in | Adda247 App
The IBPS RRB PO & Clerk 2021 | SUCCESS GUIDE
F likes orange colour sits opposite to the one who likes 20. What is the code for ‘poet’?
Pink colour. H sits second to the right of the one who likes (a) tu (b) ea (c) is
pink colour. G and D are the immediate neighbours of each (d) il (e) None of these
other. The one who likes grey colour sits immediate right
Directions (21-25): Study the following information
of D. The one who likes green colour sits second to the
carefully and answer the questions given below:
right of the one who likes Red colour. A sits immediate left
In a certain code language,
of the one who likes blue colour. E likes red colour and sits
‘forest guide case cliff’ is written as ‘mo zn si lf’,
immediate left of the one who sits opposite to D. A does
‘instant guide incident present’ is written as ‘gn iy oy si’,
not like the white colour. C does not like Pink. E is not an
‘cliff case key product’ is written as ‘vw mo zn gi’,
immediate neighbour of F.
‘domestic case present instant’ is written as ‘gn oy mo wn’.
11. Who among the following person likes Pink colour?
21. What is the code for ‘Domestic’?
(a) H (b) E (c) G
(a) wn (b) mo (c) oy
(d) C (e) None of these
(d) gn (e) None of these
12. Who among the following person sits second to the
22. Which of the following may be the code for ‘product
right of the one who likes green colour?
guide case’?
(a) A (b) E (c) D
(d) F (e) None of these (a) vw si iy (b) gi mo si (c) iy si gi
(d) vw mo si (e) Can’t be determined
13. Who among the following person likes white colour?
(a) C (b) H (c) B 23. What is the code for ‘Key’?
(d) G (e) None of these (a) gi (b) zn (c) mo
(d) vw (e) Can’t be determined
14. Who among the following person sits opposite to A?
(a) B (b) E (c) C 24. What does ‘iy’ stand for?
(d) F (e) None of these (a) Guide (b) Instant (c) Incident
(d) Present (e) None of these
15. How many people sit between G and F when counted
to right of G? 25. What is the code for ‘Case’?
(a) One (b) Two (c) Four (a) zn (b) si (c) oy
(d) Three (e) More than Four (d) mo (e) Can’t be determined

Directions (16–20): Study the following information Directions (26-30): In each question some statements are
carefully and answer the questions given below: followed by some conclusions. You have to take the given
In a certain code language, statements to be true and then decide which of the
‘dozen poet police internet’ is coded as ‘im il is tm, conclusions logically follow from the give statements,
‘poet global summit year’ is coded as ‘ac tu il ea’ disregarding the commonly known facts. Given answer-
‘year decline global release’ is coded as ‘ac tu at mn’ 26. Statements:
‘internet migrate year decline’ is coded as ‘ho ac mn is’ Only a few Grapes are Mango
16. How is ‘year’ coded in that code language? Some Mango are Orange
(a) at (b) tu (c) is No Orange is Guava
(d) ac (e) None of these Conclusions:
I. All Orange is Grapes is a possibility
17. What does ‘is’ stand for? II. Some Mango is Guava
(a) internet (b) poet (c) year (a) Only conclusion I follows.
(d) police (e) None of these (b) Only conclusion II follows.
18. ‘im’ is the code for which of the following? (c) Either conclusion I or II follows.
(a) dozen (b) police (c) migrate (d) Neither conclusion I nor II follows.
(d) poet (e) Can’t be determined (e) Both conclusion I and II follow.
19. Which of the following is the code for ‘dozen police 27. Statements:
release decline’? Only Parrot is Bird.
(a) tm tu mn at (b) im tm at mn (c) im ho is tu Some Parrot is Swan.
(d) im at mn tu (e) None of these No Swan is a White.

116 www.bankersadda.com | www.sscadda.com | www.careerpower.in | Adda247 App


The IBPS RRB PO & Clerk 2021 | SUCCESS GUIDE
Conclusions: 31. Statements:
I. Some White is Bird is a possibility Only Book are Paper.
II. All Swan is Parrot. No Book are Subject
(a) Only conclusion I follows. All Subject are Exam.
(b) Only conclusion II follows. Conclusions:
(c) Either conclusion I or II follows. I. Some Exam are Book is a possibility
(d) Neither conclusion I nor II follows. II. No Subject is Paper is a possibility
(e) Both conclusion I and II follow. (a) Only conclusion I follows.
28. Statements: (b) Only conclusion II follows.
All Word are Letter. (c) Either conclusion I or II follows.
Some Vowel are Letter. (d) Neither conclusion I nor II follows.
No Word are Sound. (e) Both conclusion I and II follow.
Conclusions:
I. Some Vowel is Word is a possibility 32. Statements:
II. All Sound is Letter is a possibility Some Money are Coin
(a) Only conclusion I follows. Some Coin are Bank.
(b) Only conclusion II follows. All Bank are Policy.
(c) Either conclusion I or II follows. Conclusions:
(d) Neither conclusion I nor II follows. I. Some Money is Policy
(e) Both conclusion I and II follow. II. No Policy is Money
29. Statements: (a) Only conclusion I follows.
All White are Brown. (b) Only conclusion II follows.
Only a few Yellow are White. (c) Either conclusion I or II follows.
Some Purple are Yellow. (d) Neither conclusion I nor II follows.
Conclusions: (e) Both conclusion I and II follow.
I. No Purple is White 33. Statements:
II. All Brown is Yellow is a possibility Only a few Ring are Music
(a) Only conclusion I follows.
Some Music are Learn.
(b) Only conclusion II follows.
No Learn is Hobby.
(c) Either conclusion I or II follows.
Conclusions:
(d) Neither conclusion I nor II follows.
(e) Both conclusion I and II follow. I. No Music is Hobby
II. All Ring is Learn
30. Statements: (a) Only conclusion I follows.
All Coffee are Sugar. (b) Only conclusion II follows.
Only a few Milk are Sugar. (c) Either conclusion I or II follows.
No Sugar is Tea (d) Neither conclusion I nor II follows.
Conclusions:
(e) Both conclusion I and II follow.
I. All Sugar is milk is a possibility
II. All Milk is Coffee is a possibility
(a) Only conclusion I follows.
(b) Only conclusion II follows.
(c) Either conclusion I or II follows.
(d) Neither conclusion I nor II follows.
(e) Both conclusion I and II follow.
Directions (31-35): In each of the questions below are
given some statements followed by some conclusions.
You have to take the given statements to be true even if
they seem to be at variance from commonly known facts.
Read all the conclusions and then decide which of the
given conclusions logically follows from the given
statements disregarding commonly known facts. Give
Answer

117 www.bankersadda.com | www.sscadda.com | www.careerpower.in | Adda247 App


The IBPS RRB PO & Clerk 2021 | SUCCESS GUIDE
34. Statements: 37. Statements:
Only a Few Google are Yahoo. Only a few Long is Short.
All Mail is Google All Long is Heavy.
No Mail are Inbox. Only a few Short is Weight.
Conclusions:
Conclusions:
I. All Weight is Heavy is a possibility
I. Some Google is not Inbox
II. All Heavy is Short is a possibility
II. All Yahoo is mail is a possibility (a) None follows
(a) Only conclusion I follows. (b) Only I follows
(b) Only conclusion II follows. (c) Only II follows
(c) Either conclusion I or II follows. (d) Either I or II follow
(d) Neither conclusion I nor II follows. (e) Both I and II follow
(e) Both conclusion I and II follow. 38. Statements:
Some Glass is Water.
35. Statements:
No Water is Milk.
Some Silver are White.
All Milk is Drink.
All White are Yellow. Conclusions:
No Yellow is Gold I. No Water is Drink
Conclusions: II. Some Glass is Milk
I. No White is Gold. (a) None follows
II. Some Silver is Yellow is a Possibility (b) Only I follows
(a) Only conclusion I follows. (c) Only II follows
(b) Only conclusion II follows. (d) Either I or II follow
(e) Both I and II follow
(c) Either conclusion I or II follows.
(d) Neither conclusion I nor II follows. 39. Statements:
(e) Both conclusion I and II follow. All Door is Gate.
All Gate is Brown.
Directions (36-40): In each of the questions below is given No Gate is Pink.
three statements followed by two conclusions numbered I Conclusions:
and II. You have to take the given statements to be true I. No Door is Pink.
even if they seem to be at variance with commonly known II. Some Pink is Brown is a possibility.
facts. Read all the conclusions and then decide which of (a) None follows
the given conclusions logically follows from the given (b) Only I follows
(c) Only II follows
statements disregarding commonly known facts.
(d) Either I or II follow
36. Statements: (e) Both I and II follow
Only Green is Grey. 40. Statements:
Some Green is Black. Some Morning is Night.
No Black is White. Only a few Night is Day.
Conclusions: No Night is Sun.
I. Some White is Grey is a possibility. Conclusions:
II. All Black is Green is a possibility I. Some Day is not Sun
II. All Night is Day is a possibility
(a) None follows
(a) None follows
(b) Only I follows
(b) Only I follows
(c) Only II follows (c) Only II follows
(d) Either I or II follow (d) Either I or II follow
(e) Both I and II follow (e) Both I and II follow

118 www.bankersadda.com | www.sscadda.com | www.careerpower.in | Adda247 App


The IBPS RRB PO & Clerk 2021 | SUCCESS GUIDE
Directions (41-45): Study the following information 47. Which of the following box is placed in the bottom of
carefully and answer the questions given below: the stack?
A person purchased different tests i.e. P, Q, R, S, T and U (a) B (b) K (c) H
during a period from Monday to Saturday, he purchased (d) C (e) None of these
only one test on each day. On each day different games 48. How Many boxes are placed between box A and Box
were played i.e. Ludo, Cricket, Badminton, Hockey, H?
Tennis, Rugby. The test R was purchased at least before (a) Four (b) Three (c) More than four
three tests and Ludo was played on Tuesday. Badminton (d) One (e) None
was played on Friday. The test U was purchased on
Tuesday. Both the tests Q and T were purchased at least 49. Which of the following statement is false about box G?
before one test. The test S was purchased immediately (a) Box G is placed just above box E
(b) Not more than two boxes are placed between box
after the test R. Hockey was not played on Monday. At
A and G
least four tests were purchased after the test T. Tennis is
(c) One box is placed between C and G
played the day on which S was purchased. Cricket is played
(d) More than three boxes are placed above box G
just after the day on which Q was purchased.
(e) Both (b) and (d)
41. How many tests were purchased after the test S?
50. Which of the following box is placed just below box C?
(a) 1 (b) 3 (c) 2
(a) D (b) H (c) G
(d) 4 (e) None of these
(d) A (e) None of these
42. Cricket was played on which day? Directions (51-55): Study the information carefully and
(a) Saturday (b) Friday (c) Monday answer the questions given below.
(d) Tuesday (e) None of these
Eight people P, Q, R, S, T, U, V, and W are going on holiday
43. The test P was purchased on which day? on two different dates i.e. 5 and 8 in four different months
(a) Monday (b) Tuesday (c) Wednesday i.e. January, March, April and May but not necessarily in
(d) Friday (e) Saturday the same order.
44. Rugby is played on which of the following day? V goes on the 5th of April. Two persons are going in
(a) Saturday (b) Friday (c) Tuesday between V and P. More than two persons are going
(d) Monday (e) None of these between U and S. U goes just after R but not in the same
month. More than Four persons are going between S and
45. Which of the following combination is not true? W. None of them goes in the month which has an even
(a) P- Cricket (b) R- Hockey (c) U- Ludo number of days. More than one person goes between W
(d) Q-Tennis (e) None and Q. T does not go in March.
Directions (46-50): Study the information carefully and 51. Who among the following person goes on 8th march?
answer the questions given below. (a) U (b) Q (c) R
(d) T (e) None of these
Nine boxes are placed one above another in a stack. Four
boxes are placed between F and H. More than Four boxes 52. How many persons are going between T and S?
are placed above the box H. Two boxes are placed (a) Four (b) One (c) Two
between box B and C, which is placed above box H. There (d) None (e) Three
are three boxes are placed between box E and box D, 53. Who among the following person goes just before V?
which is placed just above box F. More than three boxes (a) T (b) R (c) W
are placed between box C and box A, which is placed above (d) U (e) None of these
box B. More than two boxes are placed between box G and
box K, which is not placed above box C. 54. Four of the following five are alike in a certain and form
a group, find the one that does not belong to that
46. Which of the following box is placed just above box B? group?
(a) G (b) H (c) A (a) R (b) Q (c) U
(d) F (e) None of these (d) T (e) P
119 www.bankersadda.com | www.sscadda.com | www.careerpower.in | Adda247 App
The IBPS RRB PO & Clerk 2021 | SUCCESS GUIDE
55. Which of the following pair of persons goes in May? (a) If only conclusion I follows.
(a) W, P (b) U, W (c) S, P (b) If only conclusion II follows.
(d) U, S (e) None of these (c) If either conclusion I or II follows
(d) If neither conclusion I nor II follows.
Directions (56-60): In each question, four statements
(e) If both conclusions I and II follow.
showing relationship have been given, which are followed
by three conclusions I, II and III. Assuming that the given 61. Statements: D > A, F = E ≤B, D>B, C<B
statements are true, find out which conclusion(s) is/are Conclusions I. F>A II. A≥ F
definitely true. 62. Statements: A > Z < B ≥ C > X < D
56. Statements: X=U>L≥C=N<T Conclusions I. D > Z II. B ≥ X
Conclusions: I. N<U II. T>C III. X≥L 63. Statements: A > B, C ≤B, D < E = B, E < F, G ≤ B
(a) All follow Conclusions I.F > G II. C < G
(b) Only I & III follow
(c) Only II and III follow 64. Statements: X > Y ≥ Z = M > A ≥B
(d) Only either I or II follows Conclusions I. X > A II. Y ≥ M
(e) None of these 65. Statements: P < Q > R = Z < S ≤ X < T
Conclusions I. T > Z II. Z = T
57. Statements: T>Y≥B=N>F<B≤G
Conclusions: I. F≤T II. G>B III. Y>F Directions (66-70): These questions are based on the
(a) All follow following arrangement. Study the arrangement carefully
(b) Only III follows to answer these questions.
(c) Only II and III follow A%T31H@7UOEL8GP#UJ$R2K*CMV9A6
(d) Only either I or II follows &SZ
(e) Only II follows
66. How many such symbols are there in the above
58. Statements: F ≥M> A>R>E arrangement, each of which is immediately followed
Conclusions: I. M>E II. F>E III. F< E by a vowel?
(a) Only I follows (a) One (b) None (c) Two
(b) Only I & II follow (d) Three (e) More than three
(c) Only II and III follow
(d) Only either II or III follows 67. Which of the following element is exactly in the middle
(e) All follow of 3rd element from the right end and the 10th
element from the left end of the arrangement?
59. Statements: A≥B>M>D=F (a) 2 (b) R (c) $
Conclusions: I. B>D II. B<A III. M>F (d) K (e) None of these
(a) All follow
68. How many such symbols are there in the above
(b) Only I & III follow
arrangement, each of which is immediately preceded
(c) Only II and III follow
by a number and also immediately followed by a
(d) Only either II or III follows
consonant?
(e) Only III follows
(a) None (b) One (c) Two
60. Statements: P≤Q =R<S≤T (d) Three (e) More than three
Conclusions: I. Q≤T II. T>Q III. P> S 69. How many such vowels are there in the above
(a) All follow arrangement each of which is immediately followed
(b) Only I & II follow by a number and also immediately preceded by a
(c) Only II and III follow consonant?
(d) Only either I or II follows (a) None (b) One (c) Two
(e) Only II follows (d) Three (e) None of these
Direction (61-70): In these questions, relationship 70. What should come in the place of question mark (?) in
between different elements is show in the statements. the following series based on above arrangement?
The statements are followed by conclusions. Study the OEU G8P J$U K*?
conclusions based on the given statements and select the (a) C (b) M (c) V
appropriate answer: (d) 2 (e) None of these
120 www.bankersadda.com | www.sscadda.com | www.careerpower.in | Adda247 App
The IBPS RRB PO & Clerk 2021 | SUCCESS GUIDE
Directions (71-75): These questions are based on the 78. In which direction point L with respect to point P?
following set of numbers. (a) North east (b) South west (c) North west
978 721 327 198 654 (d) South east (e) None of these

71. If all the digits in each number are arranged in 79. If point N is 5m northeast from point B then what is the
ascending order within the number, then which of the shortest distance between point N and point P?
following will form the third highest in the new (a) 3m (b) 6m (c) 8m
arrangement? (d) 5m (e) None of these
(a) 327 (b) 654 (c) 978
(d) 721 (e) 198 80. If point K is north of point C and west of point F, then
what is the shortest distance between point K and
72. If first digit in each number is interchanged with the
point F?
second digit then second digit is interchanged with
third digit of number, then which of the following will (a) 12m (b) 10m (c) 14m
form the highest number in the new arrangement? (d) 13m (e) None of these
(a) 327 (b) 654 (c) 978
Directions (81-83): Study the information carefully and
(d) 721 (e) 198
answer the questions given below.
73. What will be the difference between the second digit
of the highest number and third digit of the second Point J is 10m west of point K. Point K is 8m west of point
lowest number? Y. Point Y is 6m south of point L. Point L is 4m east of point
(a) 4 (b) 5 (c) 6 H. Point H is 3m north of point V. Point V is 7m east of point
(d) 3 (e) None of these G. Point G is 3m north of point U.
74. If ‘2’ is added to all the given numbers, then the 81. What is the shortest distance between point J and
resultant of how many numbers will be completely point U?
divisible by 7? (a) 10m (b) 8m (c) 5m
(a) One (b) Two (c) Three
(d) 7m (e) None of these
(d) Four (e) More than four
82. In which direction point G with respect to point K?
75. What will be the difference of the twice of lowest
number and half of the highest number? (a) North east (b) North west (c) North
(a) 91 (b) 87 (c) 93 (d) South (e) None of these
(d) 95 (e) None of these
83. What is the shortest distance between point H and
Directions (76-80): Study the information carefully and point U?
answer the questions given below. (a) 9m (b) 5√17m (c) 8m
A person starts walking from point F and walks 6m south
(d) 7√15m (e) None of these
to reach at point L. from point L he starts walking in west
direction and walks 9m to reach at point H. Now he takes
a left turn and walks 11m to reach at point B. From point B,
he starts walking in north direction to reach at point P and
covers 4m distance. Now he takes two consecutive left
turns of 5m and 8m respectively and stopped at point C.
76. In which direction point C with respect to point H?
(a) North-east (b) South-west (c) North
(d) South (e) None of these
77. If point T is 1m east of point H the what is the shortest
distance between point T and Point F?
(a) 8m (b) 9m (c) 12m
(d) 10m (e) None of these

121 www.bankersadda.com | www.sscadda.com | www.careerpower.in | Adda247 App


The IBPS RRB PO & Clerk 2021 | SUCCESS GUIDE
Directions (84-87): Study the information carefully and 92. Which of the following is sixth to the left of eighteenth
answer the questions given below. element from the left?
There are nine persons in three generations of family. X is (a) % (b) L (c) D
the grandmother of T. D is the son in law of S. Z is the son (d) 8 (e) None of these
of Y, K is the brother in law of Y. T is the niece of Z. R is the 93. If the above sequence is written in reverse order, then
mother of M. T has no siblings. K is unmarried. which of the following will be 8th to the right of 24th
84. Who among the following person is the sister in law of element from the right end?
Z? (a) M (b) P (c) ∆
(a) D (b) M (c) T (d) # (e) None of these
(d) X (e) None of these 94. If the first fifteen elements in the above sequence are
85. How K is related to Z? written in reverse order, then which of the following
(a) Uncle (b) Aunt (c) Father element will be twenty-first from the right end?
(d) Grandfather (e) None of these (a) W (b) © (c) X
(d) R (e) None of these
86. If G is married to Z then who among the following is
the brother in law of G? 95. What should come in place of question mark in the
(a) M (b) Y (c) D following on the basis of above sequence?
(d) K (e) None of these K, O, X, D, ?
(a) 4 (b) E (c) 8
87. Who among the following is the son in law of R? (d) % (e) None of these
(a) Z (b) T (c) X
(d) M (e) None of these Directions (96-100): Study the following arrangement
carefully and answer these questions.
Directions (88-90): Study the information carefully and D9U2O3Z4EN8QT7WF61C5X2BMA4GPK
answer the questions given below.
Seven persons i.e. P, Q, R, S, T, U and V are in the family of 96. Four of the following five are alike in a certain way
four-generation with one married couple. There are three based on their positions in the above arrangement and
female members of the family. V is the mother of T, who so form a group. Which is the one that does not belong
is grandmother of S. R is son of the one who is daughter in to that group?
law of P. R has two children. S is female and she is not (a) XM4 (b) U34 (c) QW6
sister of Q. (d) NTF (e) FCX

88. Who among the following is grandson of V? 97. How many such numbers are there in the above
arrangement, each of which is immediately preceded
(a) U (b) T (c) R
by a consonant?
(d) Q (e) None of these
(a) Six (b) Three (c) Five
89. Who among the following is mother-in-law of Q? (d) Four (e) More than Six
(a) P (b) V (c) R
98. How many such numbers are there in the above
(d) S (e) U
arrangement, each of which is immediately preceded
90. How is U related to T? by a number and immediately followed by a
(a) Grandson (b) Granddaughter (c) Father consonant?
(d) Mother (e) None of these (a) One (b) Two (c) Three
(d) Four (e) More than four
Directions (91-95): Study the following digit-letter-
symbol sequence carefully and answer the questions given 99. Which of the following element is exactly between Z
below: and C in the above arrangement?
H$K1&OW©3XRL2∞D∆M#8C%P4E+Q@7 (a) 7 (b) T (c) W
F (d) 8 (e) None of these
91. How many such numbers are there in the above 100. If all the vowels in the above arrangement are
sequence, each of which is immediately preceded by a dropped, which of the following element will be the
consonant and immediately followed by a symbol? 11th to the right of D?
(a) None (b) One (c) Three (a) C (b) 1 (c) W
(d) Two (e) None of these (d) 6 (e) None of these
122 www.bankersadda.com | www.sscadda.com | www.careerpower.in | Adda247 App
The IBPS RRB PO & Clerk 2021 | SUCCESS GUIDE
IBPS RRB CLERK PRELIMS

Directions (1-5): Study the information carefully and 9. S is the only sister of O. H is the mother of S. K is
answer the questions given below. married to O. H has only two sons. M is the niece of S
Nine boxes are placed one above others in a stack. Four and N. K has no sibling. S is unmarried. Then how is N
boxes are placed between box H and K. Box K is placed related to K?
above box H. One box is placed between Box H and L. (a) Sister (b) Son (c) Brother
Three boxes are placed between box M and G. Two boxes (d) Brother-in-law (e) None of these
are placed between Box U and T. Box Z is placed above box
B. Box M is not placed just above or just below box K. Box 10. T's mother is the sister-in-law of O and O is the only
M is placed above box H. Box K is not placed at topmost of daughter of V. R is T's brother. How is O related to R?
stack. Box B placed just above box M. Box T is not placed (a) Grandmother
at the bottom of the stack. (b) Aunt
(c) Sister
1. Which of the following box is placed just above box L? (d) Cousin
(a) B (b) M (c) T (e) Mother
(d) H (e) None of these
11. Introducing a girl, a boy says, "She is the daughter of
2. How many boxes are placed in between B and G? my father's only brother-in-law". How is the girl
(a) Four (b) One (c) Two related to the boy?
(d) Three (e) None (a) Sister-in-law
3. Which of the following box is placed above box Z? (b) Grand-daughter
(a) One (b) Two (c) Four (c) Cousin
(d) Three (e) No one (d) Daughter
(e) Aunt
4. Four of the following five are alike in a certain way
based form a group, find the one that does not belong Directions (12-15): Study the following information
to that group? carefully and answer the question given below-
(a) K-M (b) B-L (c) T-U There are eight members in a family with three married
(d) Z-B (e) G-T couples. J is son-in-law of L, who is married to K. N has one
5. If box B and box T interchanged their position then son and one daughter. M is aunt of S. K is mother-in-law of
which of the following box is placed just below box B? G, who has no children. R is sister of S.
(a) M (b) H (c) L 12. How J is related to R?
(d) G (e) None of these (a) Brother (b) Uncle (c) Grandfather
Directions (6-8): Study the following information (d) Father (e) None of these
carefully and answer the question given below- 13. Who among the following is sister-in-law of G?
Seven members of a family are living in a house. There are (a) R (b) N (c) M
three married couples and three generations in this family. (d) J (e) None of these
D is grandmother of C, who is niece of L. A has only two
children. B is sister-in-law of K. F is mother of C, who has 14. Which of the following statement is not true?
no siblings. F is not sister of L. (a) R is daughter of N
(b) N is brother-in-law of G
6. If E is sister of B, then how L is related to E? (c) J is brother-in-law of M
(a) Sister (b) Brother (c) Brother-in-law (d) S is grandson of K
(d) Father (e) None of these
(e) Both (b) and (d)
7. How A is related to F?
15. How M is related to K?
(a) Father (b) Father-in-law (c) Son
(a) Daughter-in-law
(d) Brother-in-law (e) None of these
(b) Niece
8. If M is son of L, then how M is related to D? (c) Daughter
(a) Grandfather (b) Nephew (c) Grandson (d) Sister-in-law
(d) Son (e) None of these (e) None of these

123 www.bankersadda.com | www.sscadda.com | www.careerpower.in | Adda247 App


The IBPS RRB PO & Clerk 2021 | SUCCESS GUIDE
Direction (16-20): In these questions, relationship Directions (26-30): Study the information carefully and
between different elements is show in the statements. answer the questions given below.
The statements are followed by conclusions. Study the Seven persons A, B, C, D, E, F and G are going to attend a
conclusions based on the given statements and select the music concert on seven different days of the week
(Monday to Sunday) but not necessary in same order.
appropriate answer:
More than three persons goes before D, who does not go
(a) If only conclusion I follows. on Sunday. Not more than one person goes between D
(b) If only conclusion II follows. and F. F goes before D. A goes one of the day before G. B
(c) If either conclusion I or II follows goes one of the day after C but not just after. B and E do
(d) If neither conclusion I nor II follows. not go on Sunday and A goes just before C. C goes on
(e) If both conclusions I and II follow. Tuesday. Neither B nor D goes on second last day of week.
16. Statements: K<I>L≥Y>T=R<W≤D 26. Who among the following person goes just before B?
Conclusion I: L>R II: D≥T (a) D (b) E (c) F
(d) G (e) None of these
17. Statements: E=D≥C>V<U<N>M
27. Who among the flowing person goes on Saturday?
Conclusion I: C<E II: N>D (a) B (b) E (c) G
18. Statements: O<Z=Q=T≥L≥A ≤B (d) D (e) None of these
Conclusion I: A<Z II: Z=A 28. How many persons go between C and D?
(a) Two (b) Four (c) Three
19. Statements: P<H<J≤K>B>S≤X
(d) Five (e) None
Conclusion I: K≥P II: S>K
29. Who among the following person goes on Sunday?
20. Statements: W≤ C=Y≥N>F=J≤T (a) G (b) D (c) B
Conclusion I: N≥T II: C>J (d) F (e) C
Directions (21-25): Study the information carefully and 30. If A is related to F and B is related to E then which of
answer the questions given below. the following is related to D?
Eight persons are living in eight different floors of the (a) C (b) E (c) G
building in such a way ground floor is numbered as 1 and (d) B (e) A
top floor is numbered as 8. Y lives on an even number floor. Directions (31-35): Study the information carefully and
Two persons live between Y and K. T live one of the above answer the questions given below.
floor of H and both of them live at even number floor. O Seven persons are sitting in a row, in such a way some are
lives just above L floor. Two persons live between Y and X. facing south while some are facing north. More than three
persons are sits left of R, who does not sit at any extreme
M does not live above Y. X lives on of the floors below K.
end. T sits third to the right of V. R is facing north direction.
21. Who among the following live at 2nd floor? More than two persons are sits between V and R. Q sits
(a) L (b) H (c) M third to the left of P, who sits one of the extreme ends of
(d) K (e) Y the row. Three persons are sitting between S and U, who
is not an immediate neighbour of V. Immediate
22. How many floors are there in between M and X? neighbours of T facing opposite directions. U and T is
(a) Three (b) One (c) Two facing same direction of P but opposite to S.
(d) Four (e) None
23. Four of the following five are alike in certain way based
from a group, find the one that does not belong to that
group?
(a) K (b) L (c) Y
(d) M (e) X
24. How many persons live above O’s floor.
(a) Five (b) One (c) Three
(d) Four (e) Two
25. Who among the following live at topmost floor?
(a) Y (b) K (c) O
(d) T (e) L

124 www.bankersadda.com | www.sscadda.com | www.careerpower.in | Adda247 App


The IBPS RRB PO & Clerk 2021 | SUCCESS GUIDE
31. Who among the following sits immediate right of S? Directions (41-45): Study the information carefully and
(a) R (b) P (c) Q answer the questions given below.
(d) T (e) None of these Sumit starts his journey from point Z, walks 20m towards
west to reach at point H, then he starts walking in south
32. Who among the following sits third to the left of the
for 15m to reach point X. From point X he takes a left turn
one who sits second to the right of V?
and walks for 30m to reach at point S, From point S, he
(a) T (b) S (c) P start walking in north for 12m to reach at point V, then
(d) U (e) None of these takes a right turn and walks 11m to reach point Q. From
33. How many persons are sits between the one who sits point Q, He walks 9m in north direction and reach at point
immediate left of R and P? L.
(a) Four (b) Three (c) One 41. What is the shortest distance between point L and V?
(d) Two (e) None (a) 9m (b) 11m (c) √202m
34. Four of the following are alike in a certain way so form (d) 14m (e) None of these
a group, which among the following does not belong 42. If point O is midpoint between X and S then what is the
to that group? shortest distance between point V to point O?
(a) P (b) Q (c) T (a) 369m (b) 324m (c) 321m
(d) U (e) S (d) 359m (e) 3√41m
35. Who among the following third to the right of U? 43. In which direction point H with respect of Q?
(a) Q (b) T (c) V (a) North-east (b) South (c) North-West
(d) P (e) None of these (d) West (e) None of these
Directions (36-40): Study the information carefully and 44. In which direction point S with respect of L?
answer the questions given below. (a) North-east (b) South-west (c) North-west
Point W is 8m west of point R. Point B is 11m east of Point (d) West (e) South
V. Point U is 6m west of point H. Point C is 7m east of Point
45. If point M is 21m west of point L then what is the
L, which is 5m south of point V. Point R is 10m south of
shortest distance between point M and Z?
point U. Point H is 3m north of point B. (a) 9m (b) 10m (c) 8m
36. In which direction point L with respect to W? (d) 11m (e) None of these
(a) North (b) North East (c) South Directions (46-50): Study the following arrangement
(d) South west (e) None of these carefully and answer the questions given below-
37. What is the shortest distance between Point B and D&1KB6U3%95$Rπ7S©3FO≠8Z4@A2T#E
point C? 46. How many such prime numbers are there in the given
(a) 5m (b) 6m (c) 7m arrangement each of which is immediately followed
(d) 4m (e) None of these by a consonant and preceded by a symbol?
38. In which direction point C with respect to U? (a) None (b) One (c) Two
(d) Three (e) More than three
(a) South (b) South west (c) South east
(d) North west (e) North east 47. If the places of those symbols which are immediately
followed by numbers are interchanged with those of
39. If point J is 7m north of point W then in which direction
the numbers in the given arrangement, then which of
point J with respect to B?
the following will be the 13th from the right end?
(a) East (b) West (c) North west
(a) R (b) © (c) 5
(d) North east (e) None of these
(d) S (e) None of these
40. What is the shortest distance between point H and 48. Which of the following element is 8th to the right of the
point L? element which is 12th from the left end?
(a) 5√37m (b) 17m (c) √185m (a) Z (b) O (c) ≠
(d) 16m (e) None of these (d) F (e) @

125 www.bankersadda.com | www.sscadda.com | www.careerpower.in | Adda247 App


The IBPS RRB PO & Clerk 2021 | SUCCESS GUIDE
49. What should come in place of the question mark (?) in 57. How many such consonants are there in the above
the following series based on the above arrangement? arrangement each of which is immediately preceded
&K6, %5R, S3O, ? by symbol but not immediately followed by a vowel?
(a) 8@E (b) 2#E (c) 4AT (a) None (b) One (c) Two
(d) Z@2 (e) None of these (d) Three (e) More than three
50. If the symbols and numbers are dropped from the 58. Which of the following element is the tenth to the left
given arrangement and then the letters are of the eighteenth element from the left end of the
rearranged in alphabetical order from left to right, above arrangement?
then which of the following will be the 10th from the (a) % (b) B (c) K
left end? (d) O (e) 5
(a) U (b) O (c) T
59. How many such number are there in the above
(d) K (e) None of these
arrangement each of which is immediately followed
Directions (51-55): Study the following alphanumeric by symbol but not immediately preceded by a vowel?
series carefully and answer the questions given below: (a) None (b) One (c) Two
9O2$NDU%M8E45*A76#R©WD&B3F¥ (d) Three (e) None of these
51. How many vowels are present in the above series 60. Which of the following is exactly in the middle
which are immediately followed or preceded by a between A and M in the above arrangement?
perfect square? (a) T (b) δ (c) U
(a) None (b) One (c) Two (d) 4 (e) None of these
(d) Three (e) More than three
Directions (61-65): Study the following information
52. Which element is exactly between the elements which carefully and answer the question given below:
are 14th from the left and 16th from the right end? Eight friends are sitting around a square table in such a
(a) E (b) A (c) 5 way that four of them sit at four corners of the square
(d) 6 (e) none of these while the other four sits in the middle of each sides. The
53. How many consonants are present in the above series ones who sit at the four corners face inside while those
which are immediately followed by an odd number who sit in the middle of the sides face outside.
and immediately preceded by a symbol? A sits third to the right of D. B sits second to the left of the
(a) None (b) One (c) Two one, who sits opposite to S. Q sits immediate left of the
(d) Three (e) More than three one, who sits immediate right of R. A sits opposite to R. P
sits opposite to B. C does not sits at the middle of the side.
54. If all the consonants are removed from the above
series, then which element is 10th to the right of the 61. Who is sitting exactly between P and B, when counted
element which is 8th from the left end? right of P?
(a) 3 (b) © (c) # (a) S (b) Q (c) C
(d) 6 (e) None of these (d) A (e) R

55. How many symbols are between the elements which 62. How many persons sit between D and Q when counted
are 7th from the left end and 9th from the right end? left of Q?
(a) None (b) One (c) Two (a) Four (b) Three (c) Two
(d) Three (e) More than three (d) One (e) None
Directions (56-60): Study the following arrangement 63. Who among the following sits immediate right of P?
carefully and answer the questions given below: (a) R (b) C (c) Q
F61M2*B%OC#5Uδ4TZ7π9$HKA8+LE3 (d) A (e) B
56. If all symbols in the above arrangement are dropped, 64. Four of the following are alike in a certain way so form
which of the following will be the 14th from the right a group, which among the following does not belong
end? to that group?
(a) O (b) C (c) 5 (a) S (b) Q (c) P
(d) 4 (e) None of these (d) D (e) B

126 www.bankersadda.com | www.sscadda.com | www.careerpower.in | Adda247 App


The IBPS RRB PO & Clerk 2021 | SUCCESS GUIDE
65. Who among the following sits third left of the one who Conclusions:
sits second to the right of Q? I: Some grapes are black
(a) S (b) C (c) R II: All black are pink is a possibility
(d) B (e) P Directions (71-73): In each of the questions below are
Directions (66-70): In each of the questions below are given three statements followed by three conclusions
given some statements followed by some Conclusions. number I, II and III. You have to take the given statements
You have to take the given statements to be true even, if to be true even if they seem to be at variance from
they seem to be at variance from commonly known facts. commonly known facts. Read the entire conclusion and
Read all the conclusions and then decide which of the then decide which of the given conclusions logically
given conclusions logically follows from the given follows from the given statements disregarding
statements disregarding commonly known facts. commonly known facts.
(a) If only conclusion I follows. 71. Statements:
(b) If only conclusion II follows. some graduate are IAS.
(c) If either conclusion I or II follows. Some IAS are honest.
(d) If neither conclusion I nor II follows. Some honest are clerk.
(e) If both conclusions I and II follow. Conclusions:
I: Some clerk are graduate.
66. Statements:
II: Some clerk are graduate is a possibility.
Some Ram are Rom
III. All honest can be graduate
No Rom is bag
(a) only I and II follow
All bag are Bottle
(b) only I and III follow
Conclusion:
(c) only II and III follows
I: All rom can never be bottle
(d) All I, II and III follows
II: Some bottle are not Rom (e) None of these
67. Statements: 72. Statements:
All Desk are table Some hills are stone.
Some table are wood Some stone are mountain.
All room are wood No mountain are water.
Conclusion: Conclusions:
I: Some room are table I: Some hills are water.
II: No table is room II: No water are hills.
68. Statements: III. some stone are water.
No Pencil are pen (a) None follows
No charger are mobile (b) only II follows
Some pen are Charger (c) only III follows
Conclusions: (d) only I follows
I: Some mobile can be pen (e) only either I or II follow
II: Some pencil is mobile 73. Statements:
Some roof are floor
69. Statements:
No floor are window
Only a few ratio is fraction
Some window are door.
Some fraction are Review
Conclusions:
All review are tool
I: Some roof are not window.
Conclusions:
II: All floor being door is a possibility.
I: Some fraction are tool III. Some door can be roof.
II: Some review are ratio is a possibility (a) only I and II follow
70. Statements: (b) only I and III follow
Some grapes are green (c) only II and either I or III follow
All green are black (d) All I, II and III follows
Only a few black are Pink (e) None of these

127 www.bankersadda.com | www.sscadda.com | www.careerpower.in | Adda247 App


The IBPS RRB PO & Clerk 2021 | SUCCESS GUIDE
Directions (74-75): In each of the questions below are Directions (81-85): In each of the questions below are
given some statements followed by some Conclusions. given some statements followed by some Conclusions.
You have to take the given statements to be true even, if You have to take the given statements to be true even, if
they seem to be at variance from commonly known facts. they seem to be at variance from commonly known facts.
Read all the conclusions and then decide which of the Read all the conclusions and then decide which of the
given conclusions logically follows from the given given conclusions logically follows from the given
statements disregarding commonly known facts. statements disregarding commonly known facts.
(a) If only conclusion I follows. (a) If only conclusion I follows.
(b) If only conclusion II follows. (b) If only conclusion II follows.
(c) If either conclusion I or II follows. (c) If either conclusion I or II follows.
(d) If neither conclusion I nor II follows. (d) If neither conclusion I nor II follows.
(e) If both conclusions I and II follow. (e) If both conclusions I and II follow.
74. Statements: 81. Statements:
No day is night. Some song are Music
All night is bright. All music is Paper
All bright are star. Only a few paper is exam
Conclusions: Conclusion:
I: All day being bright is a possibility. I: All exam can be song
II: Some star is not day. II: All song can be paper

75. Statements: 82. Statements:


Some girls are boys. Some Today are tomorrow
All boys are IPS. All tomorrow are yesterday
All IPS are intelligent. No yesterday are Laptop
Conclusions: Conclusion:
I: All boys are intelligent. I: Some today are not Laptop
II: Some girls are intelligent. II: No tomorrow is laptop

Directions (76-80): In each of the question, relationships 83. Statements:


between some elements are shown in the statements. Some eyes are ears
These statements are followed by conclusions numbered I Some ears are arm
and II. Read the statements and give the answer. No ears are legs
(a) If only conclusion I follows. Conclusions:
(b) If only conclusion II follows. I: Some eyes are not legs
(c) If either conclusion I or II follows. II: Some Arm are not legs
(d) If neither conclusion I nor II follows. 84. Statements:
(e) If both conclusions I and II follow. Only mobile are window
76. Statements: G>S≥J>T<P=B≤A Some mobile is Summit
Conclusions: I: S>P II: A>T No summit is Market
Conclusions:
77. Statements: E<N≤V=L≤O≤P=A I: All market can never be mobile
Conclusions: I: P≥N II: O<E II: All mobile can never be market
78. Statements: B≤L<O>G≥E=R≥S 85. Statements:
Conclusions: I: S>O II: G<B Some Mango are grapes
79. Statements: T≥R=B>L≥E=U>O Only grapes are Orange
Conclusions: I: E<T II: O<L No grapes is Red
Conclusions:
80. Statements: R=A≥F=I>C, T>R≤Z I: Some mango are orange
Conclusions: I: Z>F II: I=Z II: No orange is mango

128 www.bankersadda.com | www.sscadda.com | www.careerpower.in | Adda247 App


The IBPS RRB PO & Clerk 2021 | SUCCESS GUIDE
Directions (86-90): Study the information carefully and 91. Statements: W≤E<H=Q>N>L≥P
answer the questions given below. Conclusion I: H>L II: W<Q
Eight persons K, L, M, N, O, P, Q and R are sitting around 92. Statements: L≥ R>B=A>Z<C>W
a circular table. Some of them face inside and some of Conclusion I: C≥L II: L>C
them face outside. All information is not necessarily in
same order. 93. Statements: H>L≤W<M>J>X≥J=N
Two persons sit between L and R. L sits opposite to P and Conclusion I: J=M II: M>J
both of them face towards center. O sits immediate left of 94. Statements: A≥L=R≥ S>D=O≤U>V
K. M sits immediate left of R. Q sits second to the right of Conclusion I: A≥O II: U>D
N. N sits third to the left of M. Q doesn’t sit opposite to the
95. Statements: Z>Q>P=C=R≤ E=G
person who sits immediate right of L. One person sits
Conclusion I: R≤Q II: G≥P
between O and P. K faces same direction as O and Q. P is
not immediate neighbor of O. Immediate neighbor of K Directions (96-100): Study the information carefully and
face opposite direction to each other. answer the questions given below.
An uncertain number of persons sit in a linear row in such
86. Which of the following sits third to left of Q? a way that all of them faces towards north direction. Four
(a) P (b) K (c) O Persons sit between H and A. R sits second to the right of
(d) N (e) None of these A. I sits third to the left of R. As many persons sit between
87. Who among the following sits third to the right of L? S and B as many between B and H. Four persons sit
(a) Q (b) O (c) R between R and S. H is not immediate neighbor of B. M sits
(d) M (e) None of these seventh to the right of B and third from one of the extreme
ends of row. I sits fifth from the one of the extreme end of
88. How many persons sit between M and K when counted the row. Not more than three persons sit between I and H.
from left of K?
(a) Four (b) Two (c) More than four 96. How many persons sit in the row?
(d) Three (e) None (a) 16 (b) 17 (c) 15
(d) 14 (e)18
89. Who among the following sits second to the right of
97. Who among the following person sit second to the
O?
right of the one who sits eleventh from the right end
(a) The one who sits immediate left of N
of row?
(b) M
(a) A (b) B (c) I
(c) R
(d) R (e) S
(d) The one who sits second to right of M
(e) None of these 98. How many persons sit between I and S?
(a) Six (b) Seven (c) Eight
90. Four of the following five are alike in certain way based (d) Five (e) Nine
from a group, find the one that does not belong to that
group? 99. If As many persons sit between A and T as many
(a) R (b) P (c) L between T and M then how is T related to M?
(d) O (e) M (a) Third to the left
(b) Fourth to the left
Direction (91-95): In these questions, relationship (c) Fifth to the right
between different elements is show in the statements. (d) Fourth to the right
The statements are followed by conclusions. Study the (e) None of these
conclusions based on the given statements and select the
100. If Y sits exactly between H and I then how is B related
appropriate answer:
to Y?
(a) If only conclusion I follows.
(a) Third to the right
(b) If only conclusion II follows.
(b) Fourth to the left
(c) If either conclusion I or II follows
(c) Second to the right
(d) If neither conclusion I nor II follows.
(d) Immediate left
(e) If both conclusions I and II follow.
(e) None of these

129 www.bankersadda.com | www.sscadda.com | www.careerpower.in | Adda247 App


The IBPS RRB PO & Clerk 2021 | SUCCESS GUIDE

Solutions
IBPS RRB PO PRELIMS

Solutions (1-5) Directions (16–20):

1. (b): 2. (d): 3. (b):

4. (c): 5. (b):

Solution(6-10):

16. (d): 17. (a): 18. (e):


19. (b): 20. (d):
Solution (21-25):

6. (c): 7. (b): 8. (e): Word Code


Forest lf
9. (c): 10. (d):
Guide si
Solution (11-15): Case mo
Cliff zn
Instant oy/gn
Incident iy
Present gn/oy
Domestic wn
Key/product vw/gi

21. (a): 22. (e): 23. (e):


24. (c): 25. (d):
26. (a):

11. (e): 12. (a): 13. (b):

14. (b): 15. (c):

130 www.bankersadda.com | www.sscadda.com | www.careerpower.in | Adda247 App


The IBPS RRB PO & Clerk 2021 | SUCCESS GUIDE
27. (d): 36. (c):

28. (e):
37. (b):

38. (a):
29. (b):

39. (e):

30. (a):

40. (b):
31. (a):

Solutions(41-45):
32. (c):
Day Test Game
Monday T Rugby
Tuesday U Ludo
Wednesday R Hockey
33. (d): Thursday S Tennis
Friday Q Badminton
Saturday P Cricket
41. (c): 42. (a): 43. (e):
34. (e): 44. (d): 45. (d):
Solutions (46-50):
Boxes
A
D
35. (a): F
B
G
E
C
H
K

131 www.bankersadda.com | www.sscadda.com | www.careerpower.in | Adda247 App


The IBPS RRB PO & Clerk 2021 | SUCCESS GUIDE
46. (d): 47. (b): 48. (c): 79. (a): 80. (c):
49. (b): 50. (b): Solution (81-83):
Solutions (51-55):
Date 5th 8th
Month
January W R
March U Q
April V T
May S P 81. (d): 82. (b): 83. (e):

51. (b): 52. (d): 53. (e): Solution (84-87):

54. (c): 55. (c):


66. (e): I. N<U(true) II. T>C(true) III. X≥L(false)
57. (b): I. F≤T(false) II. G>B (false) III. Y>F(true)
58. (b): I. M>E (true) II. F>E(true) III. F<E (false)
59. (b): I. B>D (true) II. B<A (false) III. M>F (True)
60. ( e) I. Q ≤ T (False) II. T > Q (True) III. P > S ( False) 84. (b): 85. (a): 86. (c):
61. (c): I. F>A (False) II. A≥ F (False) 87. (e):
62. (d): I. D > Z (False) II. B ≥ X (False) Solution (88-90):
63. (a): I.F > G (True) II. C < G(False)
64. (e): I. X > A (True) II. Y ≥ M (True)
65. (a): I. T > Z (True) II. Z = T (False)
66. (a): # U
67. (b): 68. (b): 69. (a):
70. (d): 71. (a): 72. (e):
73. (e): 74. (b): 75. (c):
Solution (76-80):
88. (c): 89. (b): 90. (a):
Solutions (91-95):
91. (d): K1&, L 2 ∞
92. (b): 93. (c): 94. (a):
95. (d):
Solutions (96-100):
96. (d): 97. (e):
98. (a): 6 1 C
76. (b): 77. (d): 78. (a): 99. (b): 100. (c):

132 www.bankersadda.com | www.sscadda.com | www.careerpower.in | Adda247 App


The IBPS RRB PO & Clerk 2021 | SUCCESS GUIDE
IBPS RRB CLERK PRELIMS

Solution(1-5): 12. (d): 13. (b): 14. (b):


Box
15. (c):
Z
K Solution (16-20):

B 16. (a): I: L>R(True) II: D≥T (False)


M
17. (d): I: C<E(false) II: N>D (False)
L
T 18. (c): I: A<Z(False) II: Z=A (False)
H 19. (d): I: K≥P(False) II: S>K (False)
G
20. (b): I: N≥T (False) II: C>J (True)
U
Solution (21-25):
1. (b): 2. (a): 3. (e):
Floor Person
4. (c): 5. (b):
Solutions (6-8): 8 T
7 K
6 O

5 L

4 Y

3 M

6. (c): 7. (b): 8. (c): 2 H

9. (d): 1 X

21. (b): 22. (b): 23. (c):

24. (e): 25. (d):

10. (b): 11. (c):


Solutions (12-15):

133 www.bankersadda.com | www.sscadda.com | www.careerpower.in | Adda247 App


The IBPS RRB PO & Clerk 2021 | SUCCESS GUIDE
Solution (26-30): Solutions (46-50):
Week Person
46. (c): π7S, ©3F
Monday A
47. (b): 48. (b): 49. (d):
Tuesday C
Wednesday F 50. (c):
Thursday B
Solutions (51-55):
Friday D
Saturday E 51. (c): 52. (c):

Sunday G 53. (b): &B3

26. (c): 27. (b): 28. (a): 54. (a): 55. (d):
29. (a): 30. (c): Solutions (56-60):
Solutions (31-35): 56. (c):

57. (c): *B%, $HK

58. (a):
31. (c): 32. (d): 33. (b): 59. (d): M2*, Z7π, π9$
34. (e): 35. (a): 60. (b):
Solution (35-40):
Solutions (61-65):

61. (a): 62. (c): 63. (d):

36. (b): 37. (e): 38. (c): 64. (b): 15. (d):

39. (b): 40. (c): Solution (66-70):

Solution (41-45): 66. (b):

67. (c):

41. (c): 42. (e): 43. (c):

44. (b): 45. (e):

134 www.bankersadda.com | www.sscadda.com | www.careerpower.in | Adda247 App


The IBPS RRB PO & Clerk 2021 | SUCCESS GUIDE
68. (a): 78. (d): I. S>O (False) II. G< B (False)
79. (e): I. E <T (True) II. O<L (True)
80. (c): I. Z>F (False) II. I=Z (False)
69. (e):
81. (e):

70. (a):
82. (e):

71. (c): 83. (e):

73. (e):
84. (b):

73. (d):

85. (b):
74. (e):

Solution(86-90-):
75. (e):

Solutions (76-80):

76. (b): I. S> P (False) II. A> T (True)


86. (a): 87. (c): 88. (a):
77. (a): I. P≥ N (True) II. O< E (False)
89. (d): 90. (d):

135 www.bankersadda.com | www.sscadda.com | www.careerpower.in | Adda247 App


The IBPS RRB PO & Clerk 2021 | SUCCESS GUIDE
91. (e): I: H>L(True) II: W<Q(True)
92. (c): I: C≥L (False) II: L>C (False)
93. (b): I: J=M (False) II: M>J(True)
94. (d): I: A≥O (False) II: U>D (False)
95. (b): I: R≤Q(False) II: G≥P (True)
Solutions(96-100):

96. (a): 97. (d): 98. (b):


99. (b): 100. (e) :

136 www.bankersadda.com | www.sscadda.com | www.careerpower.in | Adda247 App

You might also like